Anda di halaman 1dari 101

Naskah Soal

SIMAKUI 2010
SELEKSI MASUK UNIVERSITAS INDONESIA

Kemampuan Dasar

(Matematika Dasar, Bahasa Indonesia, Bahasa Inggris)


Distributed by :

Pak Anang

SELEKSI MASUK
UNIVERSITAS INDONESIA

SIMAK UI
KEMAMPUAN DASAR
Matematika Dasar
Bahasa Indonesia
Bahasa Inggris

203
Universitas Indonesia
2010

PETUNJUK UMUM
1. Sebelum mengerjakan ujian, periksalah terlebih
dulu, jumlah soal dan nomor halaman yang terdapat
pada naskah soal.
Naskah soal ini terdiri dari 13 halaman.
2. Tulislah nomor peserta Anda pada lembar jawaban
di tempat yang disediakan.
3. Tulislah kode naskah soal ini, pada lembar jawaban
di tempat yang disediakan. Kode naskah soal ini:

203

4. Bacalah dengan cermat setiap petunjuk yang


menjelaskan cara menjawab soal.
5. Pikirkanlah sebaik-baiknya sebelum menjawab tiap
soal, karena setiap jawaban yang salah akan
mengakibatkan pengurangan nilai (penilaian: benar
+4, kosong 0, salah -1).
6. Jawablah lebih dulu soal-soal yang menurut Anda
mudah, kemudian lanjutkan dengan menjawab
soal-soal yang lebih sukar sehingga semua soal
terjawab.

7. Tulislah jawaban Anda pada lembar jawaban ujian


yang disediakan.
8. Untuk keperluan coret-mencoret, harap
menggunakan tempat yang kosong pada naskah soal
ini dan jangan pernah menggunakan lembar
jawaban karena akan mengakibatkan jawaban Anda
tidak dapat terbaca.
9. Selama ujian, Anda tidak diperkenankan bertanya
atau meminta penjelasan mengenai soal-soal yang
diujikan kepada siapapun, termasuk kepada
pengawas ujian.
10. Setelah ujian selesai, Anda diharapkan tetap duduk
di tempat Anda sampai pengawas ujian datang ke
tempat Anda untuk mengumpulkan lembar jawaban.
11. Perhatikan agar lembar jawaban ujian tidak kotor,
tidak basah, tidak terlipat, dan tidak sobek.

PETUNJUK KHUSUS
PETUNJUK A:
Pilih satu jawaban yang paling tepat.

PETUNJUK B:
Soal terdiri dari 3 bagian, yaitu PERNYATAAN, kata SEBAB, dan ALASAN yang disusun berurutan.
Pilihlah:
(A) Jika pernyataan benar, alasan benar, dan keduanya menunjukkan hubungan sebab dan akibat
(B) Jika pernyataan benar, alasan benar, tetapi keduanya tidak menunjukkan hubungan sebab dan
akibat
(C) Jika pernyataan benar dan alasan salah
(D) Jika pernyataan salah dan alasan benar
(E) Jika pernyataan dan alasan keduanya salah

PETUNJUK C:
Pilihlah:
(A) Jika (1), (2), dan (3) yang benar
(B) Jika (1) dan (3) yang benar
(C) Jika (2) dan (4) yang benar
(D) Jika hanya (4) yang benar
(E) Jika semuanya benar

Kode Naskah Soal:


MATA UJIAN
TANGGAL UJIAN
WAKTU
JUMLAH SOAL

:
:
:
:

Matematika Dasar, Bahasa Indonesia, dan Bahasa Inggris


11 APRIL 2010
120 MENIT
60

Keterangan

203

Mata Ujian MATEMATIKA DASAR nomor 1 sampai nomor 20


Mata Ujian BAHASA INDONESIA nomor 21 sampai nomor 40
Mata Ujian BAHASA INGGRIS
nomor 41 sampai nomor 60

MATEMATIKA DASAR
Gunakan Petunjuk A dalam menjawab soal nomor 1 sampai
nomor 16.
1. x1 dan x2 adalah bilangan bulat yang merupakan
akar-akar persamaan kuadrat
x2 (2p + 4)x + (3p + 4) = 0, di mana p adalah
suatu konstanta. Jika x1 , p, x2 merupakan tiga
suku pertama dari suatu deret geometri, maka
suku ke-12 dari deret geometri tersebut adalah ....

(A) 1
(B) 1

(C) 6 + 2 5

(D) 6 2 5
3
. Jika a and b
2
bilangan bulat positif, maka nilai a + b adalah ....

2. lim ( 64x2 + ax + 7 8x + b) =
x

(D) 16
(E) 24

3. Ketinggian roket setelah t menit diluncurkan


vertikal ke atas dari permukaan tanah memenuhi
hubungan h = 65t 5t2 , h dalam km dan t dalam
menit. Roket tersebut mencapai ketinggian tidak
kurang dari 150 km selama ... menit.

c Universitas Indonesia

(B)
(C)

(E)

(E) 4

(A) 3
(B) 5
(C) 7

(A)

(D)

(A) 5
(B) 9
(C) 12

4. Jika x + y + 2z = k, x + 2y + z = k dan
2x + y + z = k , k = 0, maka x2 + y 2 + z 2 jika
dinyatakan dalam k adalah ....
k2
16
3k 2
16
4k 2
17
3k 2
8
2k 2
3

5. Jika (p, q) merupakan penyelesaian dari sistem


berikut:
3
log x + 2 log y = 4
3
log (x2 ) 4 log (4y 2 ) = 1,
maka nilai p q = ....

(A) 2
(B) 4
(C) 5

(D) 9
(E) 13

(D) 10
(E) 13

Downloaded from http://pak-anang.blogspot.com

Halaman 1 dari 13 halaman

Kode Naskah Soal:


2

6.

10. Nilai

log 5. 6 log 5 + 3 log 5. 6 log 5


= ....
2 log 5. 3 log 5

(D) 5

(A) 0
(B) 1
(C) 2

Nilai minimum fungsi f (x, y) = 500x + 1000y pada


daerah yang diarsir adalah ....

(D) 5.000

(A) 8.000
(B) 6.000
(C) 5.750

(E) 6

11. Dalam suatu penerbangan, penumpangnya terdiri


atas 9 anak laki-laki, 5 anak Indonesia, 9 orang
laki-laki dewasa, 7 anak laki-laki warga negara
asing, 14 warga Indonesia, 6 laki-laki Amerika
dewasa, dan 7 perempuan warga negara asing.
Jumlah penumpang penerbangan tersebut adalah
....

(A) 39
(B) 34
(C) 33

(E) 4.500

7.

203

(D) 29
(E) 26

12. Persamaan garis l yang menyinggung lingkaran


x2 + y 2 = 8 pada titik x = 2 dan memiliki gradien
positif adalah ....

(A) y = x 4
(B) y = x + 4
(C) y = 2x + 4

(B)

pertidaksamaan
....

(A) 9
(B) 8
(C) 7

<

2
2x5

(E) 7

log (2x + 4)

log 2x

(E)

(D) 6

log (2x)3

(D)
1
adalah
8

(C)

8. Bilangan bulat terkecil yang memenuhi


2x

(E) y = x + 8

13. Fungsi f : dan g : didenisikan


sebagai f (x) = 23x1 dan g(x) = 4(x + 2)3 . Jika
f 1 adalah invers dari f , maka (f 1 g)(x) = ....

(A) 2 log 3 2x

Luas segitiga pada gambar adalah .... cm2

(A) 4(1 3)

(B) 4( 3 1)

(C) 4( 3 + 1)

(D) 2( 3 + 1)

(E) 2(1 3)
1
32

(D) y = x 8

log (2x + 2)

, nilai
2
2 sin x + sin 2x cos x + sin 2x cos3 x + .... = ....

14. Untuk 0 < x <

(A) sin x
(B) cos x
(C) 2 sin x

(D) 2 sec x
(E) 2 csc x

9. Diketahui sistem persamaan berikut:


2x + y = 3
(3x 2y 1)(x + y 6) = 0.
Jika (x1 , y1 ) dan (x2 , y2 ) adalah penyelesaian dari
sistem persamaan tersebut, maka nilai dari
x1 + x2 + y1 + y2 = ....

(A) 6
(B) 5
(C) 4
c Universitas Indonesia

(D) 5
(E) 6

Downloaded from http://pak-anang.blogspot.com

Halaman 2 dari 13 halaman

Kode Naskah Soal:


15. Persamaan kuadrat x2 (a2 + 7)x + 4 = 0
mempunyai akar-akar x1 dan x2 . Jika nilai dari

x1 x2 + x2 x1 = 8, maka hasil kali dari nilai-nilai


a yang memenuhi adalah ....

(A) 5

(B) 5

(C) 5

10
(2) C4 .5!.3!
10
(3) C2
10
(4) C6 .5!.3!

(E) 5

19. 10 orang yang mengunjungi restoran akan


menempati 2 meja bundar. Meja bundar A
berukuran besar untuk 6 orang dan meja bundar B
untuk 4 orang. Banyaknya cara mereka
menggunakan kedua meja tersebut adalah ....
(1) 5!.3!

(D) 4

16. lim

203

1
1 3x
1 x x2
( ) +( )
= ...
2
2

(A) 4
(B) 2
(C) 1

(D) 2
(E) 3

20. Diberikan sepasang persamaan 2x 3y = 13 dan


3x + 2y = b dengan 1 b 100, dan b bilangan
bulat. Misalkan n2 = x + y , dengan x dan y adalah
solusi dari persamaan di atas, yang berupa
bilangan bulat, maka nilai n yang memenuhi
adalah ....
(1) 4
(2) 3

Gunakan Petunjuk C dalam menjawab soal nomor 17 sampai


nomor 20.

(3) 1
(4) 2

17. Panitia Perayaan Hari Kemerdekaan RI 17 Agustus


yang terdiri dari 4 orang akan dipilih dari 4 pasang
suami istri. Banyaknya cara pemilihan panitia
tersebut jika ....
(1) semua orang dapat dipilih = 70 cara
(2) terdiri dari 2 pria dan 2 wanita = 36 cara
(3) terdiri dari 3 pria dan 1 wanita = 16 cara
(4) semua panitia harus pria = 1 cara

<x< ,
2
2
maka pernyataan berikut ini yang benar adalah ....

18. Jika 2 sin2 x 7 sin x + 3 < 0 dengan

<x<
6
2
1
(2)
3 < tan x <
3
1
(3) 0 < cos x <
3
2
1
(4)
< sin x < 3
2
(1)

c Universitas Indonesia

Downloaded from http://pak-anang.blogspot.com

Halaman 3 dari 13 halaman

Kode Naskah Soal:

203

BAHASA INDONESIA
Gunakan Petunjuk A dalam menjawab soal nomor 21 sampai
nomor 40.
21. Penggunaan ejaan pada kalimat berikut salah,
KECUALI ...

(A) Pengacara
terdakwa,
Parlindungan
Simanungkalit, SH, akan mengajukan eksepsi
pada sidang minggu depan.
(B) Mungkin dr.
Brajadenta, MA, SH
berkepentingan
dengan
perkara
yang
menghebohkan dunia kedokteran dan hukum
itu.
(C) Prof. Dr. Basa Situmeang memang seorang
Profesor dan dia menamatkan pendidikan
S3-nya di Bandung setelah terlebih dahulu
menekuni bahasa Jerman selama enam bulan di
Berlin.
(D) Saya katakan bahwa Joni Agus Setyarto, S.H.
sengaja datang ke sini untuk melamar anak
saya, Ayu Minarti, S.Hum., yang baru saja
diwisuda kemarin.
(E) Ungkapan "Mangan ora mangan kumpul"
perlu ditinjau lagi relevansinya untuk jaman
sekarang, jaman Reformasi yang penuh dengan
kebebasan itu.
22. Secara sosial bahasa mempunyai makna yang dapat
menimbulkan berbagai penafsiran. Membelokkan
makna simbolis ke arah pengertian tertentu justru
bisa berdampak negatif yang berjangka panjang
dan turun-temurun. Pembelokan makna simbolis
dari agama, budaya, dan tradisi melalui bahasa,
misalnya, akan bisa mendatangkan malapetaka
turun-temurun yang menyandera generasi. Bisa
dibayangkan dampaknya jika penafsiran yang
salah disebarluaskan ke masyarakat.
Gagasan utama teks di atas adalah ...

(A) Dampak penafsiran makna bahasa yang salah


dapat dibayangkan.
(B) Membelokkan makna simbolis berdampak
negatif.
(C) Makna bahasa dapat menimbulkan banyak
penafsiran.
(D) Pembelokan makna simbolis mendatangkan
malapetaka.
(E) Secara sosial bahasa dapat menimbulkan
penafsiran.

c Universitas Indonesia

23. Karat sering muncul pada sepeda kesayangan kita


manakala kita kurang rajin merawatnya. Terlebih
pada bagian pelek yang berlapis krom. Alhasil,
penampilan sepeda jadi kurang sip. Bila sudah
melekat di sepeda kita, karat akan sulit
dihilangkan. Jika terus menyikat atau
mengampelasnya, lapisan asli besi bisa rusak.
Tanpa sengaja saya memperoleh kiat mengusir
karat tanpa merusak lapisan besinya asalkan
lapisan asli besi belum mengelupas. Jadi, ketika
karat sepeda masih tipis dan baru, langsung
gunakan kiat ini. Jangan karena malas, sepeda
disimpan di gudang. Karat justru akan senang.
Pesan yang ingin disampaikan melalui tulisan di
atas adalah ...

(A) Sepeda akan rusak jika sudah terkena karat.


(B) Karat akan mengganggu penampilan sepeda
kita.
(C) Jangan menganggap remeh karat sepeda kita
meskipun masih tipis dan baru.
(D) Bila karat sudah di sepeda kita, karat akan sulit
dihilangkan.
(E) Tanpa sengaja seseorang memperoleh kiat
mengusir karat tanpa merusak lapisan besinya
asalkan lapisan asli besi belum mengelupas.
24. Para maniak sepak bola dibuat lupa daratan oleh
permainan bola ajaib yang telah menjadi salah satu
ikon terbesar budaya olahraga tontonan manusia
abad XXXXI.
Makna ungkapan lupa daratan dalam kalimat di
atas adalah ...

(A) lupa diri.


(B) tidak peduli pada hal yang lain.
(C) bertindak sesuai dengan harga diri.
(D) terlalu asyik.
(E) bersorak-sorai.

Downloaded from http://pak-anang.blogspot.com

Halaman 4 dari 13 halaman

Kode Naskah Soal:


25. Cady dan para koleganya meneliti 6.997 pasien
kanker payudara. Sebagian dari mereka rutin
menjalani mamogra pada periode 19901999 dan
sebagian lainnya tidak. Perkembangan para pasien
tersebut lalu dicatat hingga tahun 2007. Setelah
sekitar 12,5 tahun periode follow-up, 461 dari
mereka meninggal karena kanker payudara dan
hampir 75% dari mereka tidak menjalani
pemeriksaan mamogra secara teratur.
Simpulan yang dapat diambil dari paragraf di atas
adalah ...

(A) Pemeriksaan mamogra yang teratur dapat


menghindari risiko kematian pada pasien
kanker payudara.
(B) Mamogra adalah metode pengobatan kanker
payudara bagi perempuan guna menghindari
kematian.
(C) Perempuan pada umumnya mendapat
gangguan kanker payudara karena tidak
menjalani pengobatan secara dini.
(D) Pada tahun 19901999 Cady dan rekan-rekan
mengadakan penelitian terhadap perempuan
yang terkena kanker payudara.
(E) Tidak semua pasien perempuan terkena
penyakit kanker payudara karena rutin
menjalani mamogra.
26. Banyak orang yang berniat menambah asupan
sayurnya dengan menyantap salad. Namun, jangan
percaya bahwa semua salad itu menyehatkan. Ada
banyak jenis lemak yang dikandung oleh berbagai
sajian salad. Anda harus tahu mana yang memberi
manfaat gizi bagi Anda dan mana yang tidak.
Tukar beberapa sayuran yang disajikan dengan
jenis sayur yang lebih sehat.
Maksud kutipan di atas terdapat pada kalimat
berikut, KECUALI ...

203

27. Penulisan gabungan kata dalam kalimat berikut


benar, KECUALI ...

(A) Saya tidak memberi tahu siapa pun soal


kedatangannya.
(B) Buku karya Dewi Lestari dipromosikan secara
getoktular.
(C) Astronot sedang berlatih beradaptasi di wilayah
antigravitasi.
(D) Orang utan dapat kalian lihat di kebun
binatang.
(E) "Sebar luaskan informasi penting ini ke segenap
karyawan!" perintah manajer itu.
28. Kalimat yang tidak efektif adalah ...

(A) Tindak kekejaman, kekerasan, dan menindas


orang kecil merupakan perbuatan tidak terpuji.
(B) Ketika saya datang, mereka sudah berkumpul
di halaman sekolah untuk menanti inspektur
upacara.
(C) Tata tertib ini tidak boleh diubah sampai ada
tata tertib baru yang disahkan oleh pimpinan.
(D) Sebagai mahasiswa, Anda diharapkan dapat
memberi keteladanan yang baik.
(E) Diperlukan orang yang sanggup berpikir kritis
dan tidak ekstrem.
29. Cara penulisan kata berikut sesuai dengan EYD,
KECUALI ...

(A) khazanah.
(B) konkret.
(C) karier.

(D) jadwal.
(E) sejarahwan.

(A) Makanan salad ternyata tidak selamanya aman


untuk kesehatan.
(B) Makanan salad jadi berbahaya jika disajikan
dengan varian yang berlemak.
(C) Makanan salad baik untuk kesehatan jika
sayuran yang disajikan bergizi.
(D) Makanan salad menjadi sehat jika tidak diberi
mayonais.
(E) Makanan salad tidak baik bagi kesehatan jika
pengelolaannya tidak tepat.

c Universitas Indonesia

Downloaded from http://pak-anang.blogspot.com

Halaman 5 dari 13 halaman

Kode Naskah Soal:


30. (1) Pembayaran dengan kartu kredit sangat praktis.
(2) Kartu kredit mempunyai kelebihan
dibandingkan dengan uang tunai. (3) Kartu kredit
lebih praktis daripada uang tunai karena dapat
dibawa ke mana-mana, sedangkan uang tunai
dalam jumlah yang besar sulit atau tidak dapat
dibawa ke mana-mana. (4) Kartu kredit juga dapat
digunakan untuk pembayaran jarak jauh,
sedangkan uang tunai sulit atau tidak dapat
digunakan untuk pembayaran jarak jauh. (5) Oleh
karena itu, kita perlu memiliki kartu kredit.
Kalimat yang mengganggu kepaduan paragraf di
atas adalah ...

(A) kalimat (1).


(B) kalimat (2).
(C) kalimat (2) dan (5).
(D) kalimat (5).
(E) kalimat (1) dan (5).
31. Kata berimbuhan yang dipakai pada konteks yang
tidak tepat ditemukan dalam kalimat ...

(A) Burung gereja beterbangan mendengar suara


lonceng berdentang.
(B) Anak-anak kejar-mengejar di halaman sekolah.
(C) Para penonton berlari-larian turun ke tengah
lapangan tatkala melihat api berkobar.
(D) Amran dan Rio selalu surat-menyurat melalui
e-mail.
(E) Pemandangan indah terhalang karena lembah
berselimutkan kabut.

203

32. Pada saat ini masyarakat semakin dipermudah


untuk memiliki kendaraan pribadi. Adanya
kemudahan ini tidak lepas dari persaingan di
antara perusahaan pembayaran yang sering
mengadakan perang harga.
Ide kedua kalimat di atas dapat digabung menjadi
satu. Kalimat yang paling tepat sebagai
penggabungan ide kedua kalimat itu adalah ...

(A) Pada saat ini masyarakat semakin dipermudah


untuk memiliki kendaraan pribadi dan
kemudahan ini tidak lepas dari persaingan
di antara perusahaan pembayaran yang sering
mengadakan perang harga.
(B) Pada saat ini masyarakat semakin dipermudah
untuk memiliki kendaraan pribadi karena
adanya persaingan di antara perusahaan
pembayaran yang sering mengadakan perang
harga.
(C) Pada saat ini persaingan di antara perusahaan
pembayaran dalam bentuk perang harga
mempermudah masyarakat untuk memiliki
kendaraan pribadi.
(D) Makin dipermudahnya masyarakat untuk
memiliki kendaraan pribadi pada saat ini tidak
lepas dari persaingan di antara perusahaan
pembayaran yang sering mengadakan perang
harga.
(E) Perang harga di antara perusahaan pembayaran
yang memudahkan masyarakat untuk semakin
memiliki kendaraan pribadi sendiri tidak lepas
dari persaingan pada saat ini.
33. Banyak perempuan menguatkan diri, lalu
meninggalkan keluarga dan kampung halaman
untuk bekerja ke luar negeri dengan tawaran gaji
yang lumayan besar bagi mereka yang
berpendidikan rendah.
Kalimat di atas kurang padu karena penggunaan
kata tugas yang tidak tepat. Kata tugas yang tidak
tepat dalam kalimat di atas adalah ...

(A) dan.
(B) yang.
(C) dengan.

c Universitas Indonesia

Downloaded from http://pak-anang.blogspot.com

(D) untuk.
(E) lalu.

Halaman 6 dari 13 halaman

Kode Naskah Soal:


34. Usaha pembangunan berencana yang pada
umumnya dilakukan oleh hampir semua negara
yang sedang berkembang itu memang bertujuan
untuk meningkatkan kemakmuran serta mengejar
kemajuan teknologi dan ilmu pengetahuan.
Gagasan utama kalimat di atas adalah ...

(A) Usaha pembangunan berencana dilakukan oleh


hampir semua negara.
(B) Usaha pembangunan berencana memang
bertujuan.
(C) Usaha pembangunan berencana meningkatkan
kemakmuran.
(D) Usaha pembangunan berencana mengejar
kemajuan teknologi dan ilmu pengetahuan.
(E) Usaha pembangunan berencana di hampir
semua negara yang sedang berkembang.
35. Kalimat-kalimat berikut tidak efektif, KECUALI ...

(A) Tulisan ini membahas tentang peran dan fungsi


kepala desa dalam memimpin masyarakat.
(B) Dalam tulisan ini dikaji peran dan fungsi kepala
desa dalam memimpin desa.
(C) Supermal Karawaci beroperasi setiap hari Senin
hingga Minggu dari pukul 09.00-22.00.
(D) Makalah ini akan membuktikan bagaimana
sosietas memengaruhi kepribadian seseorang.
(E) Sejarah Kota Yogyakarta sampai berakhirnya
kekuasaan Belanda dan Jepang merupakan
daerah yang menganut sistem feodal.

203

36. Bayi yang dilahirkan tidak dengan berat badan


rendah pada umumnya mempunyai status gizi saat
lahir yang kurang lebih sama dengan status gizi
bayi di Amerika. Akan tetapi, seiring dengan
bertambahnya umur ditambah faktor-faktor
lainnya, sebagian besar bayi tersebut terus
mengalami penurunan status gizi. Puncak
penurunan terjadi pada umur kurang lebih 1824
bulan. Pada kelompok umur inilah prevalensi
balita kurus (wasting) dan balita pendek (stunting)
mencapai titik tertinggi (Hadi, 2001). Setelah
melewati umur 24 bulan, status gizi balita
umumnya mengalami perbaikan meskipun tidak
sempurna.
Topik yang dibicarakan dalam bacaan tersebut
adalah ...

(A) status gizi balita.


(B) perbandingan gizi balita di Indonesia dan
Amerika.
(C) kasus gizi buruk pada balita.
(D) prevalensi balita kurus dan balita pendek.
(E) perbaikan status gizi balita.
37. Formosa adalah nama lain Taiwan dalam bahasa
Portugis yang berarti pulau cantik. Nama itu
disematkan bukan tanpa alasan. Kecantikan
alamnya menggaung ke berbagai penjuru dunia.
Salah satunya adalah Alishan yang membuat
seseorang berada di atas awan ketika datang ke
sana. Selain itu, ada pula Danau Sun Moon. Danau
itu merupakan danau terbesar di Taiwan.
Kalimat yang dapat melanjutkan paragraf tersebut
adalah ...

(A) Tidak ada orang di Taiwan yang tidak


mengetahui danau yang terkenal indah itu.
(B) Banyak wisatawan mendatangi daerah
perkotaan Taiwan.
(C) Keindahan alam Taiwan memang mengundang
banyak wisatawan.
(D) Keindahan Taiwan dapat pula dijumpai pula di
daerah perkotaan, misalnya di Taipei.
(E) Pemerintah Taiwan berusaha meningkatkan
pendapatan dari sektor pariwisata.

c Universitas Indonesia

Downloaded from http://pak-anang.blogspot.com

Halaman 7 dari 13 halaman

Kode Naskah Soal:


38. Bagi mereka yang terjun langsung di lapangan
perlu juga diingatkan, jangan menambah
penderitaan para korban. Bantuan yang ada
hendaknya disalurkan dengan tepat sasaran.
Bantuan untuk bencana ini harus dikelola dengan
jujur dan transparan. Derita para korban jangan
ditambah lagi karena keinginan memuaskan nafsu
serakah. Melalui bencana ini, marilah kita
mengikat persaudaraan kita dengan suatu rasa
kebersamaan, kearifan, dan kecerdasan kita dalam
merajutnya. Tragedi yang terjadi di Sumatra Barat
ini adalah derita kita bersama. Derita anak bangsa.
Karena itu, segala daya, upaya, dan pengorbanan
harus dimaksimalkan demi meringankan
penderitaan para korban. Ini adalah derita kita
bersama, derita yang harus merangsang rasa
kemanusiaan kita untuk berbicara. Marilah kita
bantu dengan ikut meringankan beban mereka.
Pernyataan yang sesuai dengan isi teks di atas
adalah ...

(A) Pengelolaan bantuan untuk bencana Sumatra


Barat yang terjun langsung di lapangan
diharapkan tepat sasaran dan transparan.
(B) Diharapkan kita semua membantu korban
bencana Sumatra Barat dengan jujur dan
transparan untuk meringankan beban mereka.
(C) Dengan adanya bencana gempa bumi yang
melanda Provinsi Sumatra Barat diharapkan
kita bahu-membahu membantu mereka.
(D) Kita bangsa Indonesia harus turut berduka atas
terjadinya musibah gempa bumi di Sumatra
Barat dengan memberikan bantuan.
(E) Marilah kita membantu korban bencana
gempa bumi di Sumatra Barat dengan rasa
kemanusiaan, baik moral maupun material.

203

39. (1) O, ibunya sayang, setua itu ia masih rajin pergi


ke pasar seperti dulu. (2) Ia tidak berkata lagi,
melangkah meninggalkan adiknya sendirian di
dapur. (3) Tanah masih basah bekas hujan
semalam, namun cuaca bersih sekali, langit biru
merata dan puncak Gunung Kelud jauh di timur
tersaput merah. (4) Pelan sekali seperti merayapi
puncaknya, matahari naik juga akhirnya. (5) Hari
masih pagi. (6) Burung jalak berlompatan di tanah
samping perigi, kutilang ramai beterbangan dari
dahan ke dahan. (7) Kandang sapi ... (Dikutip dari
novel Pulang karya Toha Mohtar. Jakarta: Pustaka
Jaya, 1994 [cet. ke-5], hlm. 22).
Makna kata tersaput merah pada kalimat (3) dalam
teks di atas adalah ...

(A) terhiasi/dihiasi warna merah dari sinar


matahari pagi.
(B) ternodai, terkotori/dinodai warna merah dari
sinar matahari pagi.
(C) tertutup/terlapisi warna merah dari sinar
matahari pagi.
(D) terdominasi warna merah dari sinar matahari
pagi.
(E) tersapu/tertelan warna merah dari sinar
matahari pagi.
40. "Peta bahasa ini yang merupakan peta yang
menunjukkan data inventarisasi bahasa daerah di
Indonesia serta wilayah persebarannya," kata Kepala
Pusat Bahasa Depdiknas di sela-sela gladi bersih
pameran pendidikan di Sasana Budaya Ganesha Institut
Teknologi Bandung.
Kalimat tersebut tidak efektif. Kalimat tersebut
dapat menjadi efektif jika diperbaiki dengan cara ...

(A) mengubah kata inventarisasi dengan inventaris.


(B) menambah -lah pada kata ini.
(C) mengubah
kata
persebarannya
dengan
penyebarannya.
(D) mengganti kata gladi bersih dengan gladi resik.
(E) mengganti kata
tengah-tengah.

c Universitas Indonesia

Downloaded from http://pak-anang.blogspot.com

di

sela-sela

dengan

di

Halaman 8 dari 13 halaman

Kode Naskah Soal:

203

BAHASA INGGRIS
(1) ____________________________ (2) Depending on what it resembles, a doll may seem to the little girl who plays
with it to be a baby, a child, or an older person. (3) The baby doll presumably appeals to the girls developing sense of
her own nature as a female, and by "mothering" the doll she strengthens her role identity. (4) Some dolls, on the other
hand, can be bought in the same size and "age" as the child, up to maxima usually of about 40 inches and 6 years of age;
these dolls can actually wear the same clothes the little girl herself wears. (5) Collecting dolls is a very popular hobby.
(6) Their appeal therefore seems to lie in a sort of companionship they provide in the girls imagination. (7) Finally, the
dolls that represent older people presumably provide a focus for the childs ego ideal, appealing to his or her sense of a
desired future identity. (8) In Japan, for instance, where dolls have been very important for millennia, both boys and
girls celebrate annual festivals during which they are presented with dolls that represent men and women outstanding
in Japanese history; during the festivals manly and womanly virtues are praised for the childrens edication.
Gunakan Petunjuk A dalam menjawab soal nomor 41 sampai
nomor 42.
41. The paragraph should begin with _______.

(A) Baby dolls, child dolls, older dolls are common


examples of dolls
(B) There seems to be three ways in which dolls are
valued
(C) Dolls attract children regardless their age and
sex
(D) There are several kinds of dolls that girls prefer
to buy
(E) The annual doll festivals are very important for
Japanese children
42. The sentence which is irrelevant to the topic of the
text is sentence number _______.

(A) three
(B) four
(C) ve

c Universitas Indonesia

(D) six
(E) seven

Downloaded from http://pak-anang.blogspot.com

Halaman 9 dari 13 halaman

Kode Naskah Soal:

203

Reality television is a genre of television programming which, it is claimed, presents unscripted dramatic or
humorous situations, documents actual events, and features ordinary people rather than professional actors. It could be
described as a form of articial or "heightened" documentary. Although the genre has existed in some form or another
since the early years of television, the current explosion of popularity dates from around 2000.
Reality television covers a wide range of television programming formats, from game or quiz shows which resemble
the frantic, often demeaning programmes produced in Japan in the 1980s and 1990s (a modern example is Gaki no
tsukai), to surveillance- or voyeurism- focused productions such as Big Brother .
Critics say that the term "reality television" is somewhat of a misnomer and that such shows frequently portray a
modied and highly inuenced form of reality, with participants put in exotic locations or abnormal situations,
sometimes coached to act in certain ways by o-screen handlers, and with events on screen manipulated through
editing and other post-production techniques.
Part of reality televisions appeal is due to its ability to place ordinary people in extraordinary situations. For
example, on the ABC show, The Bachelor , an eligible male dates a dozen women simultaneously, travelling on
extraordinary dates to scenic locales. Reality television also has the potential to turn its participants into national
celebrities, in talent and performance programs such as Pop Idol, though frequently Survivor and Big Brother participants
also reach some degree of celebrity.
Some commentators have said that the name "reality television" is an inaccurate description for several styles of
program included in the genre. In competition-based programs such as Big Brother and Survivor, and other
special-living-environment shows like The Real World, the producers design the format of the show and control the
day-to-day activities and the environment, creating a completely fabricated world in which the competition plays out.
Producers specically select the participants, and use carefully designed scenarios, challenges, events, and settings to
encourage particular behaviours and conicts. Mark Burnett, creator of Survivor and other reality shows, has agreed
with this assessment, and avoids the word "reality" to describe his shows; he has said, "I tell good stories. It really is not
reality TV. It really is unscripted drama."
Gunakan Petunjuk A dalam menjawab soal nomor 43 sampai
nomor 47.
43. The topic of this text is _______.

(A) critics on reality television programs


(B) what reality television programs really are
(C) the popularity of reality television programs
(D) manipulation in reality television programs
(E) the appeal of reality television programs
44. In the rst line, the writer says "it is claimed" to
show that _______ in the rst sentence.

(A) the writer agrees with the statement


(B) everyone agrees with the statement
(C) no one agrees with the statement
(D) the writer gives his approval to the statement
(E) the writer somewhat disagrees with the
statement

45. The words "heightened documentary" in paragraph


1 means that _______.

(A) the story in the reality television program is


made up
(B) the story in the reality television program is of
a real event
(C) the reality television is able to locate people in
an extraordinary situation
(D) the reality television focuses on games and
quizzes
(E) the production of the reality television
programs is well-managed
46. Which statement is NOT true about Survivor?

(A) It is a popular reality TV show.


(B) Its creator does not think it shows reality.
(C) Its participants can become celebrities like those
of pop idol.
(D) It involves an important element of competition.
(E) It is considered to be in the same category of
shows as The Real World.

c Universitas Indonesia

Downloaded from http://pak-anang.blogspot.com

Halaman 10 dari 13 halaman

Kode Naskah Soal:

203

47. The purpose of the writer is to show _______.

(A) that the term "reality television program" is not


precise
(B) some criticisms on reality television programs
(C) various formats that the reality television
programs have
(D) the maneuvers used in the reality television
programs
(E) screen manipulation in the reality television
programs

c Universitas Indonesia

Downloaded from http://pak-anang.blogspot.com

Halaman 11 dari 13 halaman

Kode Naskah Soal:

203

Gastronomy is the study of the relationship between culture and food. It is often thought ____(48)____ that the term
gastronomy refers exclusively to the art of cooking, but this is only a small part of this discipline; it cannot always be
said that a cook is also gourmet. Gastronomy studies various ____(49)____ components with food as its central axis.
Thus, it is related to the Fine Arts and Social Sciences, and even to the Natural Sciences in terms of the digestive system
of the human body. A gourmets principal activities involve discovering, tasting, experiencing, researching,
understanding and writing about foods. Gastronomy is, ____(50)____, an interdisciplinary activity. Good observation
will ____(51)____ that around the food, there exist dance, dramatic arts, painting, sculpture, literature, architecture, and
music; in other words, the Fine Arts. ____(52)____, it also involves physics, mathematics, chemistry, biology, geology,
agronomy, and also anthropology, history, philosophy, psychology, and sociology. The application of scientic
knowledge to cooking and gastronomy has become known as molecular gastronomy.
Gunakan Petunjuk A dalam menjawab soal nomor 48 sampai
nomor 52.
48. ....

(A) accurately
(B) erroneously
(C) surprisingly

(D) strangely
(E) seriously

49. ....

(A) culture
(B) cultural
(C) culturally

(D) culturalist
(E) culturing

50. ....

(A) in addition
(B) in contrast
(C) therefore

(D) nevertheless
(E) furthermore

51. ....

(A) convince
(B) propose
(C) dierentiate
(D) refuse
(E) reveal
52. ....

(A) However
(B) Consequently
(C) Hence

c Universitas Indonesia

(D) Thus
(E) Therefore

Downloaded from http://pak-anang.blogspot.com

Halaman 12 dari 13 halaman

Kode Naskah Soal:


Gunakan Petunjuk A dalam menjawab soal nomor 53 sampai
nomor 60.
53. "Im afraid you cant get away with this kind of
work."
"_______"

(A) I guess youre right. I didnt realize it was that


bad.
(B) Dont worry, Im not going away.

203

57. "Wheres the report, Ben? You told me it would be


ready by now. I need it for the board meeting this
afternoon."
"Dont worry, Jim. _______ before the board
meeting."

(A) I have it nished


(B) I have had it nished
(C) I will have it nished

(C) Im sorry, I didnt mean to frighten you.

(D) The report will nish

(D) Thats the only way I can do anyway.

(E) The report has nished

(E) Dont be afraid. Im going to be away.


54. "The earthquake happening at about 3 oclock
yesterday caused many people to panic."
"Including me, as I _______ on the third-oor of my
oce at that time."

(A) worked

58. "This rock concert is boring and too loud for me."
"I agree. _______ leave?"

(A) Will we
(B) Shall we
(C) Must we

(D) Would we
(E) Dont we

59. _______ as the author of Faust, Goethe, made


important contributions to every branch of writing,
and from the 1770s until his death all major
developments in German literature reected his
inuence.

(B) have worked


(C) have been working
(D) was working
(E) had worked
55. "Did you receive our inquiry? When will we receive
your conrmation?"
"My apology. It seems that _______. Could you
possibly resend it?"

(A) we mislay your letter

(A) Knowing most widely


(B) To be most widely known
(C) He was most widely known
(D) Having known most widely
(E) Most widely known

(B) your letter is mislaid


(C) we had mislaid your letter
(D) your letter has been mislaid
(E) your letter was mislaid
56. In the treatment of people _______, the diet must
include more calories than the body needs for
energy so that the excess calories can be stored in
the body as fat.

60. "I wouldnt have bought a new laptop if I had not


needed one."
This means that I _______ a new laptop.

(A) did not need


(B) would buy
(C) would need

(D) bought
(E) didnt buy

(A) whose underweight


(B) which are underweight
(C) that are to be underweight
(D) who are underweight
(E) whom are underweight

c Universitas Indonesia

Downloaded from http://pak-anang.blogspot.com

Halaman 13 dari 13 halaman

SELEKSI MASUK
UNIVERSITAS INDONESIA

SIMAK UI
KEMAMPUAN DASAR
Matematika Dasar
Bahasa Indonesia
Bahasa Inggris

204
Universitas Indonesia
2010

PETUNJUK UMUM
1. Sebelum mengerjakan ujian, periksalah terlebih
dulu, jumlah soal dan nomor halaman yang terdapat
pada naskah soal.
Naskah soal ini terdiri dari 13 halaman.
2. Tulislah nomor peserta Anda pada lembar jawaban
di tempat yang disediakan.
3. Tulislah kode naskah soal ini, pada lembar jawaban
di tempat yang disediakan. Kode naskah soal ini:

204

4. Bacalah dengan cermat setiap petunjuk yang


menjelaskan cara menjawab soal.
5. Pikirkanlah sebaik-baiknya sebelum menjawab tiap
soal, karena setiap jawaban yang salah akan
mengakibatkan pengurangan nilai (penilaian: benar
+4, kosong 0, salah -1).
6. Jawablah lebih dulu soal-soal yang menurut Anda
mudah, kemudian lanjutkan dengan menjawab
soal-soal yang lebih sukar sehingga semua soal
terjawab.

7. Tulislah jawaban Anda pada lembar jawaban ujian


yang disediakan.
8. Untuk keperluan coret-mencoret, harap
menggunakan tempat yang kosong pada naskah soal
ini dan jangan pernah menggunakan lembar
jawaban karena akan mengakibatkan jawaban Anda
tidak dapat terbaca.
9. Selama ujian, Anda tidak diperkenankan bertanya
atau meminta penjelasan mengenai soal-soal yang
diujikan kepada siapapun, termasuk kepada
pengawas ujian.
10. Setelah ujian selesai, Anda diharapkan tetap duduk
di tempat Anda sampai pengawas ujian datang ke
tempat Anda untuk mengumpulkan lembar jawaban.
11. Perhatikan agar lembar jawaban ujian tidak kotor,
tidak basah, tidak terlipat, dan tidak sobek.

PETUNJUK KHUSUS
PETUNJUK A:
Pilih satu jawaban yang paling tepat.

PETUNJUK B:
Soal terdiri dari 3 bagian, yaitu PERNYATAAN, kata SEBAB, dan ALASAN yang disusun berurutan.
Pilihlah:
(A) Jika pernyataan benar, alasan benar, dan keduanya menunjukkan hubungan sebab dan akibat
(B) Jika pernyataan benar, alasan benar, tetapi keduanya tidak menunjukkan hubungan sebab dan
akibat
(C) Jika pernyataan benar dan alasan salah
(D) Jika pernyataan salah dan alasan benar
(E) Jika pernyataan dan alasan keduanya salah

PETUNJUK C:
Pilihlah:
(A) Jika (1), (2), dan (3) yang benar
(B) Jika (1) dan (3) yang benar
(C) Jika (2) dan (4) yang benar
(D) Jika hanya (4) yang benar
(E) Jika semuanya benar

Kode Naskah Soal:


MATA UJIAN
TANGGAL UJIAN
WAKTU
JUMLAH SOAL

:
:
:
:

Matematika Dasar, Bahasa Indonesia, dan Bahasa Inggris


11 APRIL 2010
120 MENIT
60

Keterangan

204

Mata Ujian MATEMATIKA DASAR nomor 1 sampai nomor 20


Mata Ujian BAHASA INDONESIA nomor 21 sampai nomor 40
Mata Ujian BAHASA INGGRIS
nomor 41 sampai nomor 60

MATEMATIKA DASAR
Gunakan Petunjuk A dalam menjawab soal nomor 1 sampai
nomor 18.
x
1. Jika f (x) =
, x = 2 maka f 1 (x) adalah
x2 4
....
2x
x2 + 2
2x

, x = 1
x2 1
2x

,
x2 + 1

2x

, x= 2
22
x
x

, x = 1
21
x

(A) f (x) =
(B) f (x) =
(C) f (x) =
(D) f (x) =
(E) f (x) =

2. Jika A dan B adalah dua kejadian dengan


1
1
11
P (A) = dan P (B) = serta P (A B) =
,
8
2
16
maka kejadian A dan B adalah ....

(A) saling bebas


(B) saling lepas
(C) tidak saling bebas
(D) saling lepas dan tidak bebas
(E) tidak dapat ditentukan hubungannya
3. x1 dan x2 adalah bilangan bulat yang merupakan
akar-akar persamaan kuadrat
x2 (2p + 4)x + (3p + 4) = 0, di mana p adalah
suatu konstanta. Jika x1 , p, x2 merupakan tiga
suku pertama dari suatu deret geometri, maka
suku ke-12 dari deret geometri tersebut adalah ....

(A) 1
(B) 1

(C) 6 + 2 5

(D) 6 2 5

1 3c 3
4 3
3

=
2 1
6 21
2b
3
maka nilai dari a + b + c + d adalah ....

4. Jika 3

(A) 47
(B) 37
(C) 27

4a
d

(D) 17
(E) 7

5. Agar pertidaksamaan 2x2 + 4x + a2 > 6 dipenuhi


oleh semua bilangan riil x, maka ....

(A) a > 2 atau a < 2


(B) 2 < a < 2

(C) 2 2 < a < 2 2

(D) a < 2 2 atau a > 2 2


(E) a < 3 atau a > 3
6. Garis y = mx + 5 memotong parabola
y = x2 4mx + 4n di titik P dan Q. Jika P = (1,6),
maka koordinat Q adalah ....
3 13
(A) ( , )
2 2

5 + 21 15 + 21
(B) (
,
)
2
2

5 21 15 21
(C) (
,
)
2
2
9 29
(D) ( , )
4 4
(E) (4,9)
7. Jika f (x) =
f 1

log (x + 1) +

log

1
, maka
x2

log 2 = ....

(A) 3
(B) 4
(C) 5

(D) 6
(E) 7

(E) 4
c Universitas Indonesia

Downloaded from http://pak-anang.blogspot.com

Halaman 1 dari 13 halaman

Kode Naskah Soal:


8. Jika x + y + 2z = k, x + 2y + z = k dan
2x + y + z = k , k = 0, maka x2 + y 2 + z 2 jika
dinyatakan dalam k adalah ....

(A)
(B)
(C)
(D)
(E)

k2
16
3k 2
16
4k 2
17
3k 2
8
2k 2
3

204

11. Sebuah menara dan gedung masing-masing


mempunyai tinggi 50 m dan 62 m. Pada saat sudut
elevasi matahari mencapai 60o , selisih bayangan
menara dan gedung adalah ....

(A)
(B)
(C)

2 3

3 3

(D) 4 3

(E) 8 3

12. Koesien suku tengah dari (3 2x)6 adalah ....

(A) 4320
(B) 2160
(C) 160

(D) 2160
(E) 4320

13.

9.

Luas segitiga pada gambar adalah .... cm2

(A) 4(1 3)

(B) 4( 3 1)

(C) 4( 3 + 1)

(D) 2( 3 + 1)

(E) 2(1 3)
10. Sigma membeli 5 kg jeruk impor berlabel diskon
10%, dan 7 kg jeruk lokal berlabel diskon 5%.
Sigma membayar dengan pecahan Rp100.000,00
dan menerima uang kembali Rp26.350,00. Kasir
menyatakan bahwa jumlah potongan harga sesuai
dengan label diskon adalah Rp5.850,00. Jika pada
waktu dan di toko yang sama Prima membeli 2 kg
jeruk impor dan 3 kg jeruk lokal sejenis dengan
yang dibeli Sigma, maka Prima harus membayar
sebesar ....

(A) Rp30.600,00
(B) Rp31.650,00
(C) Rp33.000,00

c Universitas Indonesia

Sebuah tempat air terbuat dari plat baja yang


berbentuk separuh tabung (sesuai gambar). Bagian
atas terbuka dan kapasitasnya 125 liter. Agar
bahan pembuatannya sehemat mungkin,
nilai h = ... meter.

(A) 1
(B) 5
(C) 10

(D) 50
(E) 100

14. Persamaan 3 cos x sin x = 2 p dapat dicari


penyelesaiannya apabila p memenuhi ....

(A) 4 p 0
(B) 0 p 4
(C) 4 p 2
(D) p 2 atau p 2
(E) 2 p 2

(D) Rp34.500,00
(E) Rp35.150,00

Downloaded from http://pak-anang.blogspot.com

Halaman 2 dari 13 halaman

Kode Naskah Soal:


15. Segitiga OAB adalah segitiga sama kaki
(OA = OB). Titik O merupakan titik asal, dan B
terletak di sumbu X positif. Jika koordinat titik
A(3, 4), maka koordinat titik berat segitiga OAB
adalah ....

(A)

4 2
,
3 3

(C)

2 8
,
3 3

(D)

8 4
,
3 3

(E)

8 2
,
3 3

18.

4 8
,
3 3

(B)

204

16. Jika 4 log(2 log x) + 2 log(4 log x) = 2, maka

5
log x + x + 5 = ....

(A) 1
(B) 2
(C) 4

(D) 5
(E) 16

17. Jumlah nilai x dan y yang merupakan bilangan


bulat dari sistem persamaan berikut:
2x + 3y 1 = 0
x2 xy 2y 2 x 4y 2 = 0,
adalah ....

(A) 7
(B) 1
(C) 1

Jarak dari titik R ke garis horizontal adalah ....

(A) 3 + 3 cm

(B) 3 3 cm

(C) 3 + 2 3 cm

(D) 6 + 2 3 cm

(E) 6 2 3 cm

(D) 3
(E) 7

Gunakan Petunjuk C dalam menjawab soal nomor 19 sampai


nomor 20.

3
3 sin x
1
19. Jika A =
untuk < x <
2
sin x
cos x
2
2
dan det ( A ) = 1, maka x mempunyai nilai ....

2
5
(2)
6
3
(3)
2
7
(4)
6
(1)

20. Diberikan sepasang persamaan 2x 3y = 13 dan


3x + 2y = b dengan 1 b 100, dan b bilangan
bulat. Misalkan n2 = x + y , dengan x dan y adalah
solusi dari persamaan di atas, yang berupa
bilangan bulat, maka nilai n yang memenuhi
adalah ....
(1) 4
(2) 3
(3) 1
(4) 2

c Universitas Indonesia

Downloaded from http://pak-anang.blogspot.com

Halaman 3 dari 13 halaman

Kode Naskah Soal:

204

BAHASA INDONESIA
Gunakan Petunjuk A dalam menjawab soal nomor 21 sampai
nomor 40.
21. Jika data perusahaan Anda diakses lebih dari satu
orang, dari setiap orang dapat diberi hak akses data
yang berbeda-beda, password ini akan sangat
berguna.
Kalimat inti dari kalimat luas tersebut adalah ...

(A) Jika data perusahaan Anda diakses lebih dari


satu orang.
(B) Setiap orang dapat diberi hak akses data yang
berbeda-beda.
(C) Password ini sangat berguna.
(D) Data perusahaan Anda diakses.
(E) Setiap orang dapat diberi hak akses data.
22. Untuk menanggulangi pencemaran tanah akibat
penumpukan sampah itu dapat dilakukan melalui
berbagai cara seperti melalui program 3 R, yaitu
reduce, reuse, dan recycle.
Perbaikan kalimat yang tepat untuk kalimat
tersebut adalah ...

(A) Penanggulangan pencemaran tanah akibat


penumpukan sampah itu dapat dilakukan
melalui program 3 R, yaitu reduce, reuse, dan
recycle.
(B) Menanggulangi pencemaran tanah akibat
penumpukan sampah itu dapat dilakukan
melalui berbagai cara.
(C) Menanggulangi pencemaran tanah akibat
penumpukan sampah itu dapat dilakukan
melalui program 3 R, yaitu reduce, reuse, dan
recycle.
(D) Untuk menanggulangi pencemaran tanah
akibat penumpukan sampah itu melalui
berbagai cara dapat dilakukan seperti program
3 R, yaitu reduce, reuse, dan recycle.
(E) Penanggulangan pencemaran tanah akibat
penumpukan sampah itu berbagai cara dapat
dilakukan seperti program 3 R, yaitu reduce,
reuse, dan recycle.

c Universitas Indonesia

23. Sejumlah badan usaha setelah bencana gempa


menimpa Padang, Sumatera Barat, telah
melakukan PHK (Pemutusan Hubungan Kerja).
Hotel Ambacang melakukan PHK terhadap 120
karyawan dengan pesangon gaji 1 bulan gaji. Hotel
Rocky melakukan PHK terhadap 130 karyawan
dengan pesangon 3 bulan gaji. Selain itu, Hotel
Bima Minang juga akan mem-PHK 200
karyawannya. Sebaliknya, 200 karyawan RS BMC
diminta pihak manajemen untuk membuat surat
pengunduran diri dengan alasan RS tersebut tidak
mampu menggaji karyawan karena bangunan dan
peralatan RS rusak berat.
Gagasan utama paragraf tersebut adalah ...

(A) Pascagempa di Padang banyak hotel yang


mem-PHK-kan karyawannya.
(B) Pengusaha dengan terpaksa harus mem-PHK
karyawan.
(C) Sejumlah badan usaha melakukan PHK.
(D) PHK membuat para pekerja tidak memiliki
pekerjaan.
(E) PHK terjadi karena perusahaan tersebut
bangkrut.
24. Terlepas dari kontroversi mengenai autisme, para
peneliti tidak menjumpai hubungan yang jelas
antara autisme dan vaksinasi. Beberapa kasus
memang memperlihatkan bahwa gejala autisme
muncul setelah si anak mendapatkan suntikan
vaksin tertentu. Vaksin yang sering dicurigai dapat
memicu autisme ialah MMR. Namun, para peneliti
yakin ini hanya sebuah kebetulan.
Gagasan pokok paragraf di atas adalah ...

(A) Para peneliti tidak menjumpai hubungan yang


jelas antara autisme dan vaksinasi.
(B) Beberapa kasus memperlihatkan bahwa gejala
autisme muncul setelah si anak mendapatkan
suntikan vaksin tertentu.
(C) Vaksin yang sering dicurigai dapat memicu
autisme ialah MMR.
(D) Para peneliti yakin ini hanya sebuah kebetulan.
(E) Gejala autisme yang muncul setelah si
anak mendapatkan suntikan vaksin tertentu
merupakan sebuah kebetulan.

Downloaded from http://pak-anang.blogspot.com

Halaman 4 dari 13 halaman

Kode Naskah Soal:


25. Di antara lima kalimat berikut, kalimat yang
penulisannya benar dari segi kaidah adalah ...

(A) "Siapa dia, Mas?" tanyanya sambil menunjuk


perempuan molek berambut panjang yang dari
tadi berdiri mematung di dekat kios Pak Udin.
(B) Makna kata eksibel adalah "luwes".
(C) "Sewaktu pintu kelas kubuka, kudengar suara
serentak dari murid-muridku, "Selamat pagi,
Bu Guru," kisah Bu Kiki.
(D) Kata depan "di", "ke", dan "dari" dituliskan
terpisah dari kata yang mengikutinya, misalnya
di kamar, ke atas, dan dari belakang.
(E) Harga kaos ini tidak lebih dari Rp. 30.000,- .
26. Kalimat-kalimat berikut tidak efektif, KECUALI ...

(A) Topeng adalah benda dengan bentuk tertentu


yang dapat dikenakan seseorang sehingga
pada saat seseorang mengenakan topeng
tersebut, ia dapat menarik sifat dari topeng
yang dikenakannya.
(B) Dengan memiliki sepasang mata dan alisnya,
dua daun telinga yang dihiasi anting, satu
hidung dengan kumis bapang, jambang dan
jenggot yang lebat, serta mulut yang terbuka
sehingga terlihat gigi yang besar dengan
taringnya yang tajam.
(C) Sedangkan menurut Kardji, topeng merupakan
kata lainnya dari tapel sebagai penutup muka.
(D) Tidak dapat diketahui dengan pasti apakah
sejak dulu Ondel-ondel digunakan sepasang
(lelaki-perempuan),
bahkan
sekarang
diikutsertakan pula anak-anaknya yang lebih
kecil.
(E) Bila merujuk pada pendapat orang bahwa
Ondel-ondel merupakan penjelmaan danyang
(setan), maka bentuk yang seram lebih
representatif dibanding yang cantik.
27. Di antara kalimat-kalimat berikut, kalimat yang
bukan kalimat baku adalah ...

(A) Permintaan para langganan telkomsel segera


ditanggapi dengan cepat.
(B) Pengendara mobil dilarang melewati jalan itu,
kecuali pada hari libur.
(C) Sehubungan dengan itu, dikemukakannya
saran-saran yang positif.
(D) Kita
memerlukan
pemikiran
untuk
memecahkan masalah itu.
(E) Satu cara terbaik hidup sehat adalah mengatur
diet yang seimbang.
c Universitas Indonesia

204

28. Perlu kita sadari bahwa tidak semua tayangan


televisi dapat berpengaruh negatif terhadap
pembentukan perilaku anak. Ada tayangan yang
justru mengajarkan anak berperilaku baik,
menghargai pendapat teman, dan hormat terhadap
orangtua. Sayangnya, tayangan televisi yang
seperti itu sangat sedikit jumlahnya. Hasil
penelitian Yayasan Kesejahteraan Anak Indonesia
menyebutkan persentase acara televisi khusus
anak-anak hanya sekitar 2,7%4,5% dari total
tayangan. Yang lebih mengkhawatirkan, materi
yang disajikan sangat mengganggu perkembangan
perilaku anak-anak karena kebanyakan bertemakan
cerita kurang bermutu.
... , diperlukan perhatian yang cukup serius untuk
mengantisipasi pengaruh negatif tayangan televisi
bagi anak karena anak adalah generasi penerus
bangsa dan melalui tangan merekalah
pembangunan bangsa ini akan diwariskan. Jadi,
sangat tidak mungkin apabila bangsa yang besar
ini dibangun oleh generasi muda yang memiliki
perilaku buruk.
Kata sambung antarparagraf yang harus
digunakan pada awal paragraf kedua adalah ...

(A) akan tetapi.


(B) meskipun demikian.
(C) karena.
(D) oleh karena itu.
(E) padahal.
29. Di antara lima kalimat berikut, kalimat yang
penulisannya benar dari segi kaidah penulisan
angka adalah ...

(A) "Kalau ingin memahami lebih lanjut topik yang


sedang kita bahas, Saudara dapat membaca Bab
III, Pasal 8, halaman 67," katanya.
(B) 15 orang dinyatakan hilang dalam gempa tahun
lalu.
(C) Dari 33 peserta, delapan belas di antaranya
perempuan dan sisanya laki-laki.
(D) Saya telah menyerahkan 7 karung beras kepada
fakir miskin.
(E) Dia dilahirkan tahun 50-an.

Downloaded from http://pak-anang.blogspot.com

Halaman 5 dari 13 halaman

Kode Naskah Soal:


30. Peningkatan jumlah penumpang kereta rel listrik
Jabotabek pada Oktober 2007 tercatat sebanyak
182.528 orang dan meningkat menjadi 195.588
orang pada November 2007.
Jika kita memperbaiki kalimat di atas, kata yang
harus dibuang adalah ...

(A) sebanyak.
(B) meningkat menjadi.
(C) peningkatan.

32. (1) Investasi yang baik adalah investasi emas dalam


bentuk batangan ... investasi dalam bentuk tanah.
(2) Emas batangan mudah dijual kembali. (3) ...,
emas batangan tidak memerlukan ongkos
pembuatan seperti halnya emas perhiasan. (4) ...,
tidak ada salahnya jika Anda mempertimbangkan
investasi emas dalam bentuk batangan.
Kata hubung yang paling sesuai untuk mengisi
rumpang di atas adalah ...

(A) dibandingkan, lagi pula, dengan demikian

(D) tercatat.

(B) daripada, selain itu, jadi

(E) jumlah.

(C) dari, sebagai tambahan, jadi

31. Baterai nano memiliki kelebihan bisa "tidur


panjang" alias tahan lama meski tidak dipakai
selama 15 tahun. Dengan begitu, baterai dapat
digunakan sebagai power untuk alat sensor
monitor radioaktif atau sensor monitor bahan
kimiawi beracun lain. Baterai nano ini bisa
langsung "hidup" dan dengan waktu singkat
memberi energi tinggi saat dibutuhkan. Baterai
nano juga baterai pertama yang mampu
membersihkan dirinya sendiri setelah digunakan.
Ia akan menetralkan cairan kimia beracun yang
timbul di dalamnya.
Yang dinyatakan mengenai baterai nano dalam teks
di atas adalah ...

(A) kelebihan dan kekurangan baterai nano.


(B) kelebihan dan kekurangan baterai nano
dibandingkan dengan baterai lain.
(C) penggunaan baterai nano sensor monitor bahan
kimiawi beracun.
(D) kelebihan baterai nano sebagai alat sensor
monitor radioaktif.
(E) kelebihan-kelebihan baterai nano.

c Universitas Indonesia

204

(D) dibandingkan dengan, selain itu, dengan


demikian
(E) daripada, sementara itu, jadi
33. Banyak atlet yang dulunya berprestasi, tetapi
sekarang hidupnya susah. Hal ini disebabkan oleh
kecenderungan untuk beralih profesi bagi seorang
atlet masih sulit. Selain minimnya keterampilan,
atlet segan menanggalkan titelnya sebagai "sang
juara". Banyak atlet yang masih gengsi untuk
menjadi pedagang atau pegawai kantoran. Jadi,
mereka memilih menjadi pelatih. Ketika mereka
pensiun dari pelatih, mereka tidak mempunyai
penghasilan. Akhirnya, hidup mereka menjadi
susah. Situasi ini sebenarnya dapat dihindari kalau
saja mereka mau berusaha meninggalkan ego
mereka sebagai "sang juara".
Pesan yang paling tepat yang ingin disampaikan
melalui teks di atas adalah ...

(A) Alih profesi bagi seorang atlet masih sulit.


(B) Atlet ingin tetap menjadi sang juara.
(C) Atlet harus meninggalkan ego "sang juara".
(D) Atlet masih gengsi untuk menjadi pedagang
atau pegawai kantoran.
(E) Atlet lebih senang menjadi pelatih.

Downloaded from http://pak-anang.blogspot.com

Halaman 6 dari 13 halaman

Kode Naskah Soal:


34. Saat ini, sebagian burung di Indonesia berada di
tepi jurang kepunahan. Dalam hal ini, tercatat 71
jenis burung endemik berstatus kritis, genting, dan
rentan. Di antaranya adalah burung yang
merupakan lambang nasional Indonesia, elang
jawa.
Kalimat yang dapat melanjutkan kutipan di atas
adalah ...

(A) Burung endemik lain yang berstatus kritis


adalah cenderawasih.
(B) Burung paok hujan memiliki panjang 18 cm.
(C) Peneliti Indonesia tidak memiliki data komplet
mengenai migrasi burung daratan.
(D) Jenis burung ini patut dilindungi.
(E) Habitat burung-burung di Indonesia itu rusak.
35. Kalimat yang tidak efektif adalah ...

(A) Tindak kekejaman, kekerasan, dan menindas


orang kecil merupakan perbuatan tidak terpuji.
(B) Ketika saya datang, mereka sudah berkumpul
di halaman sekolah untuk menanti inspektur
upacara.
(C) Tata tertib ini tidak boleh diubah sampai ada
tata tertib baru yang disahkan oleh pimpinan.
(D) Sebagai mahasiswa, Anda diharapkan dapat
memberi keteladanan yang baik.
(E) Diperlukan orang yang sanggup berpikir kritis
dan tidak ekstrem.
36. Ulama itu mempertemukan Hasan dan Azizah.
Proses pembentukan kata mempertemukan dalam
kalimat di atas sejalan dengan proses pembentukan
kata berimbuhan memper-kan dalam kalimat ...

204

37. ... Batik kraton adalah batik yang pada awalnya


berkembang di dalam kraton. Batik ini dulu hanya
boleh dipakai oleh keturunan bangsawan yang
tinggal di lingkungan Kraton Solo atau Puro
Mangkunegaran, tetapi sekarang boleh dipakai
siapa saja. Batik nonkraton adalah batik yang
berkembang di luar kraton, yaitu di kalangan para
saudagar. Batik ini sejak dulu boleh dipakai siapa
saja.
Kalimat yang tepat mengawali paragraf di atas
adalah ...

(A) Pemakai batik kraton dan batik nonkraton.


(B) Pemakaian batik kraton terbatas, sedangkan
batik nonkraton tidak terbatas.
(C) Batik kraton dan batik nonkraton adalah batik
yang digemari orang.
(D) Ada dua jenis batik solo, yaitu batik kraton dan
batik nonkraton.
(E) Pada awalnya, pemakaian batik kraton dan
batik nonkraton berbeda.
38. Kata berimbuhan yang dipakai pada konteks yang
tidak tepat ditemukan dalam kalimat ...

(A) Burung gereja beterbangan mendengar suara


lonceng berdentang.
(B) Anak-anak kejar-mengejar di halaman sekolah.
(C) Para penonton berlari-larian turun ke tengah
lapangan tatkala melihat api berkobar.
(D) Amran dan Rio selalu surat-menyurat melalui
e-mail.
(E) Pemandangan indah terhalang karena lembah
berselimutkan kabut.

(A) Patih Gadjah Mada berupaya mempersatukan


kerajaan-kerajaan Nusantara.
(B) Atasan Anda tidak mempersalahkan Anda atas
peristiwa yang terjadi kemarin.
(C) Mengapa Andi selalu mempertanyakan
nilai-nilai kuliahnya?
(D) Saya mohon Anda tidak mempersamakan saya
dengan dia.
(E) Wartawan memperjuangkan orang yang
tertindas dengan kata-kata.

c Universitas Indonesia

Downloaded from http://pak-anang.blogspot.com

Halaman 7 dari 13 halaman

Kode Naskah Soal:

204

39. (1) O, ibunya sayang, setua itu ia masih rajin pergi


ke pasar seperti dulu. (2) Ia tidak berkata lagi,
melangkah meninggalkan adiknya sendirian di
dapur. (3) Tanah masih basah bekas hujan
semalam, namun cuaca bersih sekali, langit biru
merata dan puncak Gunung Kelud jauh di timur
tersaput merah. (4) Pelan sekali seperti merayapi
puncaknya, matahari naik juga akhirnya. (5) Hari
masih pagi. (6) Burung jalak berlompatan di tanah
samping perigi, kutilang ramai beterbangan dari
dahan ke dahan. (7) Kandang sapi .... (Dikutip dari
novel Pulang karya Toha Mohtar. Jakarta: Pustaka
Jaya, 1994 (cet. ke-5), hlm. 22).
Makna kata perigi pada kalimat 6 dalam teks di atas
adalah ...

(A) balai-balai.
(B) gudang tua.
(C) empang.
(D) tanggul.
(E) sumur.
40. Perlindungan terhadap kesehatan dan keselamatan
kerja sesuai dengan Undang-undang Tenaga Kerja
diberlakukan terhadap semua orang di tempat
kerja. Anjuran kesehatan dan keselamatan kerja
merupakan tugas penting manajemen yang baik di
semua wilayah. Perusahaan, bagaimanapun,
berkewajiban melindungi semua karyawan di
seluruh tempat kerja. Karyawan harus menerima
salinan tertulis undang-undang tersebut dari
perusahaan agar mereka mengetahui hak mereka.
Dari kalimat berikut ini, kalimat yang sesuai
dengan kutipan di atas adalah ...

(A) Tidak ada peraturan kesehatan dan


keselamatan kerja sebelum sebelum adanya
Undang-Undang Tenaga Kerja.
(B) Undang-undang kesehatan dan keselamatan
kerja hanyalah tanggung jawab karyawan.
(C) Karyawan
perlu
mengetahui
isi
Undang-Undang Tenaga Kerja karena berkaitan
dengan hak kesehatan dan keselamatan mereka
selama bekerja.
(D) Undang-undang menyatakan bahwa semua
karyawan harus menerima salinan tertulis.
(E) Perlindungan
terhadap
kesehatan
dan
keselamatan kerja berlaku untuk karyawan
tetap.

c Universitas Indonesia

Downloaded from http://pak-anang.blogspot.com

Halaman 8 dari 13 halaman

Kode Naskah Soal:

204

BAHASA INGGRIS
European society in the 18th century was a broad pyramid, with the few of the nobility at the top and the masses of
the peasantry at the bottom. In western Europe there was an increasing split between the wealthy nobility, who spent
much of their time practicing exquisite etiquette at court, and those lower nobles who stayed in the countryside, hunting
and running their estates with little concern for either social niceties or abstract ideas. Only a few at the social peak lived
the life portrayed in 20th century historical novelsamid bright chandeliers, powdered wigs, and beautiful womenbut
it was in this milieu that the century earned its reputation for licentiousness and decadence. Every monarch had his
mistresses, and the ways of Versailles were mimicked across Europe, most absurdly in the courts of the tiny German
principalities.
Gunakan Petunjuk A dalam menjawab soal nomor 41 sampai
nomor 42.
41. The topic of this paragraph is _______.

(A) the wealthy and the poor


(B) the European pyramid
(C) the wealthy nobility in Europe
(D) the European luxurious life in the 18th century
(E) social life in the 18th century European society
42. The paragraph which precedes this one most likely
discusses _______.

(A) European lower nobles


(B) wealthy people in Germany
(C) family disputes in Europe
(D) social class in European society
(E) social problems

c Universitas Indonesia

Downloaded from http://pak-anang.blogspot.com

Halaman 9 dari 13 halaman

Kode Naskah Soal:

204

A monkey that has acquired the sole power to hand out apples is generously rewarded with grooming sessions by
the other monkeys in its group. However, as soon as another monkey can hand out apples as well, the market value of
the rst monkey is halved. The monkeys therefore unerringly obey the law of supply and demand. In the experiment
researchers placed food containers with highly-desired pieces of apple in two groups of South African vervet monkeys.
For the monkeys there was just one problem: only one in each group could open the food container. This monkey had a
low position in the rank order and was therefore scarcely groomed. However, as soon as she acquired the power to hand
out apples she was valued more and was groomed a lot by the rest of the group. Yet she could only enjoy that privilege
briey; the researchers placed a second food container that could be opened by another low-ranking female. From that
moment onwards the market value of the rst monkey was halved, and she was therefore groomed half as often.
The experiments revealed that the female monkeys that could open the food containers were groomed more than
when they exerted no power over the food production. The females concerned also did not have to groom the other
monkeys as long. They were therefore paid for their services as food suppliers. Biological market theory predicts that
the market value of these female monkeys should vary according to the law of supply and demand. The fact that the
grooming time of the rst monkey was halved as soon as the second monkey gained the power to distribute apples,
conrms this idea; the price of goods - in this case the female monkeys who could open the containers - was
instantaneously adjusted to the market. Immediately after the opening of the food containers, the researchers registered
how long the females were groomed for. The next occasion on which the females could open a container was, however,
several days later. The fact that the females were still groomed more indicates that the vervet monkeys apply a strategy
that works in the long term. The choice of partners is also inuenced by long-term attitudes; the monkeys can value one
monkey relatively more than the others.
A change in price grooming for less long if there is another monkey that supplies apples is only possible if a
negotiation process takes place. Many economists assume that such negotiations can only take place if they are
concluded with a contract. However, the vervet monkeys do not have the possibility to conclude such binding contracts
and yet they still succeed in agreeing to a change in price for a service.
Gunakan Petunjuk A dalam menjawab soal nomor 43 sampai
nomor 47.
43. The most appropriate title for this passage is
_______.

(A) Vervet Monkeys Grooming Habits


(B) Why Monkeys Groom
(C) Monkeys Reward for Distributing Food
(D) Economic Rules of Supply and Demand for
Monkeys
(E) The Application of Economic Principles
44. The word "unerringly" in paragraph 1 is closest in
meaning to _______.

(A) equally
(B) precisely
(C) progressively
(D) abruptly
(E) unexpectedly

c Universitas Indonesia

45. According to the text, the grooming time of the


female monkey that can open a food container
_______.

(A) is decided through negotiations with other


monkeys
(B) is not aected when another female can supply
apples
(C) is reduced if there is another monkey that
supplies apples
(D) is determined by the number of apples they can
provide
(E) is always the same for every monkey
46. Which of the following statements is NOT TRUE,
according to the text?

(A) Vervet monkeys only apply a short term


strategy.
(B) Female monkeys supplying apples received
rewards from other groups.
(C) There were two food containers involved in the
experiment.
(D) Vervet monkeys behave according to the law of
supply and demand.
(E) Vervet monkeys could have negotiations
without binding contracts.

Downloaded from http://pak-anang.blogspot.com

Halaman 10 dari 13 halaman

Kode Naskah Soal:

204

47. This passage would probably be an assigned


reading in which of the following course?

(A) Economics
(B) Science
(C) Sociology

c Universitas Indonesia

(D) Statistics
(E) Psychology

Downloaded from http://pak-anang.blogspot.com

Halaman 11 dari 13 halaman

Kode Naskah Soal:

204

Major developments have taken place in the eld of agriculture during the last century, one of the most important of
which has been the introduction and extensive use of machinery. This has had great eects on the environment and on
the lives of millions of people around the world. ____(48)____ , consideration of some of the eorts of agricultural
mechanization , both positive and negative, is ____(49)____ for any country currently experiencing an increase in the use
of such machines. The ____(50)____ increase in output that has been made possible by more use of mechanization is
probably the most important positive eect of this process. The speed of planting crops, spreading fertilizers and
pesticides, and harvesting, is phenomenal. All three of these processes contribute to equally enormous increases in
production. ____(51)____, mechanization has improved food production during this century and has helped to feed the
larger world ____(52)____.
Gunakan Petunjuk A dalam menjawab soal nomor 48 sampai
nomor 52.
48. ....

(A) Besides
(B) Therefore
(C) Otherwise

(D) In addition
(E) However

49. ....

(A) serious
(B) thorough
(C) ecient

(D) sensitive
(E) essential

50. ....

(A) whole
(B) absolute
(C) vast

(D) urgent
(E) gradual

51. ....

(A) However
(B) In addition
(C) Besides

(D) Thus
(E) Moreover

52. ....

(A) population
(B) populated
(C) populous

c Universitas Indonesia

(D) populate
(E) populist

Downloaded from http://pak-anang.blogspot.com

Halaman 12 dari 13 halaman

Kode Naskah Soal:


Gunakan Petunjuk A dalam menjawab soal nomor 53 sampai
nomor 60.
53. "Did you complete your term paper last night?"
"_______, but I was too tired"

(A) I was lucky

204

57. "Did you receive our inquiry? When will we receive


your conrmation?"
"My apology. It seems that _______. Could you
possibly resend it?"

(A) we mislay your letter


(B) your letter is mislaid

(B) I should have done it

(C) we had mislaid your letter

(C) Im terribly sorry

(D) your letter has been mislaid

(D) I couldnt make it

(E) your letter was mislaid

(E) I managed to
54. "The earthquake happening at about 3 oclock
yesterday caused many people to panic."
"Including me, as I _______ on the third-oor of my
oce at that time."

(A) worked
(B) have worked

58. Ive just received an email from the bookstore


_______ that the newest Harry Potter novel will be
available next week.

(A) to say
(B) saying
(C) which said

(D) to be saying
(E) said

59. "Your hair looks great."


"Thanks. I ______________ at that new
hairdressers."

(C) have been working


(D) was working
(E) had worked
55. "Wheres the report, Ben? You told me it would be
ready by now. I need it for the board meeting this
afternoon."
"Dont worry, Jim. _______ before the board
meeting."

(A) I have it nished

(A) cut my hair


(B) have cut my hair
(C) had my hair cut
(D) had to cut my hair
(E) was cutting my hair
60. "Ive called him several times to no avail. He must
have forgotten to bring his mobile phone."
This means that he _______.

(B) I have had it nished


(C) I will have it nished
(D) The report will nish

(A) didnt have a mobile phone

(E) The report has nished

(B) didnt remember to bring his mobile phone

56. _______ his parents had wanted him to be a


government administrator when he graduated
from college, he ended up to be the most wanted
terrorist of the country.

(A) When
(B) While
(C) Until

c Universitas Indonesia

(C) remembered bringing his mobile phone


(D) could not remember his mobile phone
(E) didnt forget to bring his mobile phone

(D) Since
(E) Unless

Downloaded from http://pak-anang.blogspot.com

Halaman 13 dari 13 halaman

SELEKSI MASUK
UNIVERSITAS INDONESIA

SIMAK UI
KEMAMPUAN DASAR
Matematika Dasar
Bahasa Indonesia
Bahasa Inggris

205
Universitas Indonesia
2010

PETUNJUK UMUM
1. Sebelum mengerjakan ujian, periksalah terlebih
dulu, jumlah soal dan nomor halaman yang terdapat
pada naskah soal.
Naskah soal ini terdiri dari 11 halaman.
2. Tulislah nomor peserta Anda pada lembar jawaban
di tempat yang disediakan.
3. Tulislah kode naskah soal ini, pada lembar jawaban
di tempat yang disediakan. Kode naskah soal ini:

205

4. Bacalah dengan cermat setiap petunjuk yang


menjelaskan cara menjawab soal.
5. Pikirkanlah sebaik-baiknya sebelum menjawab tiap
soal, karena setiap jawaban yang salah akan
mengakibatkan pengurangan nilai (penilaian: benar
+4, kosong 0, salah -1).
6. Jawablah lebih dulu soal-soal yang menurut Anda
mudah, kemudian lanjutkan dengan menjawab
soal-soal yang lebih sukar sehingga semua soal
terjawab.

7. Tulislah jawaban Anda pada lembar jawaban ujian


yang disediakan.
8. Untuk keperluan coret-mencoret, harap
menggunakan tempat yang kosong pada naskah soal
ini dan jangan pernah menggunakan lembar
jawaban karena akan mengakibatkan jawaban Anda
tidak dapat terbaca.
9. Selama ujian, Anda tidak diperkenankan bertanya
atau meminta penjelasan mengenai soal-soal yang
diujikan kepada siapapun, termasuk kepada
pengawas ujian.
10. Setelah ujian selesai, Anda diharapkan tetap duduk
di tempat Anda sampai pengawas ujian datang ke
tempat Anda untuk mengumpulkan lembar jawaban.
11. Perhatikan agar lembar jawaban ujian tidak kotor,
tidak basah, tidak terlipat, dan tidak sobek.

PETUNJUK KHUSUS
PETUNJUK A:
Pilih satu jawaban yang paling tepat.

PETUNJUK B:
Soal terdiri dari 3 bagian, yaitu PERNYATAAN, kata SEBAB, dan ALASAN yang disusun berurutan.
Pilihlah:
(A) Jika pernyataan benar, alasan benar, dan keduanya menunjukkan hubungan sebab dan akibat
(B) Jika pernyataan benar, alasan benar, tetapi keduanya tidak menunjukkan hubungan sebab dan
akibat
(C) Jika pernyataan benar dan alasan salah
(D) Jika pernyataan salah dan alasan benar
(E) Jika pernyataan dan alasan keduanya salah

PETUNJUK C:
Pilihlah:
(A) Jika (1), (2), dan (3) yang benar
(B) Jika (1) dan (3) yang benar
(C) Jika (2) dan (4) yang benar
(D) Jika hanya (4) yang benar
(E) Jika semuanya benar

Kode Naskah Soal:


MATA UJIAN
TANGGAL UJIAN
WAKTU
JUMLAH SOAL

:
:
:
:

Matematika Dasar, Bahasa Indonesia, dan Bahasa Inggris


11 APRIL 2010
120 MENIT
60

Keterangan

205

Mata Ujian MATEMATIKA DASAR nomor 1 sampai nomor 20


Mata Ujian BAHASA INDONESIA nomor 21 sampai nomor 40
Mata Ujian BAHASA INGGRIS
nomor 41 sampai nomor 60

MATEMATIKA DASAR
Gunakan Petunjuk A dalam menjawab soal nomor 1 sampai
nomor 17.
x
x1
, g(x) =
dengan
x+1
x
1
x = 1, x = 0 maka (f g) (x) adalah ....

1. Jika f (x) =

(A)
(B)
(C)
(D)
(E)

(B) 1

(C) 6 + 2 5

(D) 6 2 5

(E) 4

c Universitas Indonesia

33
19

33 54
19 31

(E)

2
1

33 19
54 31

(D)

7
5

2
1
41 19

(C)

2. x1 dan x2 adalah bilangan bulat yang merupakan


akar-akar persamaan kuadrat
x2 (2p + 4)x + (3p + 4) = 0, di mana p adalah
suatu konstanta. Jika x1 , p, x2 merupakan tiga
suku pertama dari suatu deret geometri, maka
suku ke-12 dari deret geometri tersebut adalah ....

(A)
(B)

1x
1
, x=
1 2x
2
1
, x=0
x
1 2x
, x=1
1x
1
, x=0
x
1+x
1
, x=
1 + 2x
2

(A) 1

3. Diketahui AX = B, BC = D. Jika
1
2
3 2
A=
,C =
,D=
3 5
1 1
maka X adalah ....

41 2
19
1

54
31

4. Jika x + y + 2z = k, x + 2y + z = k dan
2x + y + z = k , k = 0, maka x2 + y 2 + z 2 jika
dinyatakan dalam k adalah ....

(A)
(B)
(C)
(D)
(E)

k2
16
3k 2
16
4k 2
17
3k 2
8
2k 2
3

Downloaded from http://pak-anang.blogspot.com

Halaman 1 dari 11 halaman

Kode Naskah Soal:


5.

Luas segitiga pada gambar adalah .... cm2

(A) 4(1 3)

(B) 4( 3 1)

(C) 4( 3 + 1)

(D) 2( 3 + 1)

(E) 2(1 3)
6. Jika y = 3x 1 , maka nilai y untuk x yang
+
memenuhi x2 2x + 1 2 |x + 1| 0 adalah ....

(A) y 8 atau y 2

(D) 10 y 0
(E) y 8 atau y 0
7. Kota A memiliki 500 pemilih yang semuanya
menentukan dua hal pada suatu referendum. Hal
pertama menunjukkan bahwa terdapat 375 yang
setuju, sementara hal kedua menunjukkan 275
yang setuju. Jika ternyata tepat ada 40 pemilih yang
tidak setuju dengan kedua hal tersebut, maka
banyaknya pemilih yang setuju tehadap kedua hal
tersebut adalah ....

c Universitas Indonesia

(B) 75 < x < 345


(C) x < 75 atau x > 285

(E) 460
2

log x

11. Jika x dan y adalah bilangan bulat yang memenuhi


sistem berikut:
3x + 4y = 32
y>x
3
y < x,
2
maka x + y = ....

(A) 5
(B) 7
(C) 8

(D) 9
(E) 12

12. Jika 1 dan 2 memenuhi persamaan


10 cos 2x + sin x 9 = 0 dengan 1 > 2 , maka
csc2 1 csc2 2 = ....

(D) 350

8. Jika x memenuhi persamaan x


4
log x2 = ....

(D) 4 atau 4
1
(E) 2 atau
2

(A) 75 < x < 285

(E) x < 15 atau x > 75

(C) y 10 atau y 0

(A) 2 atau 2
1
(B) 4 atau
4
(C) 1 atau 1

9. Segitiga ABC mempunyai sisi-sisi a, b dan c, dengan


perbandingan a : b : c = 2 : 3 : 4, maka tan(A + B) =
....

(A) 15

15
(B)
4
2
(C)
3

15
(D)
4

(E) 15

10. Grak fungsi = 3 cos x , terletak di bawah


f (x)
grak g(x) = 2 sin x pada interval ....

(D) x < 75 atau x > 345

(B) 8 y 0

(A) 190
(B) 210
(C) 310

205

= 16, maka

(A) 41
(B) 9
(C) 1

(D) 9
(E) 41

13. Jika diketahui f (x) = |tan(x)| , maka laju


perubahan f (x) pada saat x = k, di mana

< k < akan sama dengan ....


2

(A) sin(k)
(B) cos(k)
(C) sec2 (k)

Downloaded from http://pak-anang.blogspot.com

(D) sec2 (k)


(E) cot(k)

Halaman 2 dari 11 halaman

Kode Naskah Soal:


3 sin 2x cos 2a 3 cos 2x sin 2a
,
sin(x a)
nilai maksimum dari fungsi tersebut adalah ....

14. Diketahui f (x) =

(D) 5

(A) 2
(B) 3
(C)

(E) 6

15. Zakiya membeli x tangkai bunga seharga y rupiah,


dengan x dan y adalah bilangan bulat, y dalam
ribuan (misalnya 2 adalah Rp2.000,00). Saat hendak
meninggalkan toko, pramuniaganya berkata, "Jika
Anda membeli lagi 18 tangkai bunga, saya akan
menjualnya dengan harga 6 sehingga Anda hemat
0,6 per lusin tangkai bunga". Nilai x dan y yang
memenuhi kondisi ini adalah ....

Gunakan Petunjuk C dalam menjawab soal nomor 18 sampai


nomor 20.
18. Diberikan sepasang persamaan 2x 3y = 13 dan
3x + 2y = b dengan 1 b 100, dan b bilangan
bulat. Misalkan n2 = x + y , dengan x dan y adalah
solusi dari persamaan di atas, yang berupa
bilangan bulat, maka nilai n yang memenuhi
adalah ....
(1) 4
(2) 3
(3) 1
(4) 2
19. Daerah hasil dari fungsi f (x) =

(A) x = 10, y= 4
(B) x= 12, y= 3

205

x
, x=0
|x|
0, x = 0

adalah ....

(C) x = 12, y= 4

(1) 1

(D) x = 10, y= 3

(2) 0

(E) x = 15, y = 5
16. Jika a dan b adalah bilangan bulat positif dan b
bukan kuadrat dari suatu bilangan bulat, relasi dari

a dan b sehingga jumlah dari a + b dan


kebalikannya merupakan bilangan bulat adalah ....

(A) a2 = b 1
(B) a2 = b + 1

(3) 1
(4) (, )
20. Nilai x yang memenuhi persamaan
x2 px + 20 = 0 dan x2 20x + p = 0 adalah ....

(1) 10 4 5
(2) 1

(C) a = b + 1

(3) 10 + 4 5

(D) a2 = b2 + 1

(4) 1

(E) a = b + 1
17. Jika (x1 , y1 ) dan (x2 , y2 ) adalah penyelesaian dari
sistem persamaan berikut:
1 5

( log x) + 3 log y = 4
2 x
log 25 y log 9 = 1
maka 5 log x1 x2 3 log y1 y2 = ....

(A) 4
(B) 6
(C) 8

c Universitas Indonesia

(D) 12
(E) 16

Downloaded from http://pak-anang.blogspot.com

Halaman 3 dari 11 halaman

Kode Naskah Soal:

205

BAHASA INDONESIA
Gunakan Petunjuk A dalam menjawab soal nomor 21 sampai
nomor 40.
21. Para pegawai negeri menerima gaji setiap awal
bulan dan dibelanjakan sebagian untuk keperluan
sehari-hari.
Kalimat tersebut dapat menjadi efektif jika
diperbaiki dengan cara ...

(A) menghilangkan kata para.


(B) menambahkan kata pada sebelum kata setiap.
(C) mengganti
kata
dibelanjakan
dengan
membelanjakannya.
(D) menambahkan nya pada kata keperluan.
(E) menghilangkan kata awal.
22. Pernahkah mengalami pada saat bangun tidur,
mata Anda tampak sembab dan kulit terlihat
kusam? Itu artinya Anda kurang tidur. Perlu
diketahui, selain asupan nutrisi dari dalam dan
luar tubuh, kualitas tidur juga bisa memengaruhi
kondisi kesehatan dan keindahan kulit kita.
Pernyataan berikut tepat sebagai jawaban untuk
pertanyaan dalam kutipan di atas, KECUALI ...

(A) Jam tidur yang tidak cukup dapat


mengakibatkan mata sembab dan kulit kusam.
(B) Pola makan yang tidak benar akan berdampak
bagi kesehatan kulit.
(C) Kualitas tidur yang tidak wajar dapat
memengaruhi mata sembab dan kulit kusam.
(D) Penggunaan krim malam sesaat sebelum tidur
akan membuat kulit cerah.
(E) Ruang tidur yang mewah dapat membuat tidur
nyaman sehingga mata tidak sembab.
23. Kalimat yang tidak efektif adalah ...

(A) Tindak kekejaman, kekerasan, dan menindas


orang kecil merupakan perbuatan tidak terpuji.
(B) Ketika saya datang, mereka sudah berkumpul
di halaman sekolah untuk menanti inspektur
upacara.
(C) Tata tertib ini tidak boleh diubah sampai ada
tata tertib baru yang disahkan oleh pimpinan.
(D) Sebagai mahasiswa, Anda diharapkan dapat
memberi keteladanan yang baik.
(E) Diperlukan orang yang sanggup berpikir kritis
dan tidak ekstrem.

24. Kalimat berikut yang efektif adalah ...

(A) Elizabeth Pahl, pemandu kami, bercerita, udara


pagi itu memang sangat luar biasa.
(B) Selain itu, karena bentuk topogranya, banyak
ditemui jurang-jurang, lidah glestser serta
celah-celah es yang berbahaya.
(C) Dengan menumpang Aeroot SU-787, dari
Moskow kami tiba tepat pukul 19.50 di
Miralnye Vody.
(D) Meskipun Rusia sudah berubah dan membuka
diri ala reformasi, namun sisa-sisa ciri negara
sosialis dari negeri tirai besi ini masih terlihat.
(E) Senin pagi, 17 Agustus 2009, di tengah
kebekuan udara yang mencapai suhu 25o C,
kami sudah bangun dan mempersiapkan diri
untuk mendaki puncak Gunung Elbrus.
25. Pada malam Minggu diselenggarakan pertunjukan
musik di panggung utama.
Kata pertunjukan mengalami pola pembentukan
yang sama dengan kata berimbuhan per-an dalam
kalimat berikut, KECUALI ...

(A) Peragaan busana kali ini menampilkan busana


pengantin.
(B) Saya selalu berupaya memberikan perhatian
kepada orang tua.
(C) Ani mengagumi perlakuan Ria kepada
sahabatnya yang cacat.
(D) Persamaan hak kaum perempuan dengan pria
terus diperjuangkan.
(E) Perbuatan Rio terhadap temannya sulit
dimaafkan.
26. (1) Tempe dan tahu buatan Mampang tidak
menggunakan formalin. (2) Agar tetap awet,
kuncinya adalah masa fermentasi yang sempurna.
(3) Biasanya setelah diberi ragi, tempe dibiarkan
selama tiga hari hingga benar-benar siap dijual. (4)
Tempe dengan proses pembuatan seperti itu
memiliki rasa yang enak. (5) Kedelai yang dipakai
biasanya dimpor dari Amerika Serikat.
Kalimat yang tidak mendukung gagasan utama
adalah ...
(A) kalimat 2.
(B) kalimat 3 dan 4.
(C) kalimat 4.
(D) kalimat 4 dan 5.
(E) kalimat 5.

c Universitas Indonesia

Downloaded from http://pak-anang.blogspot.com

Halaman 4 dari 11 halaman

Kode Naskah Soal:


27. (1) Penularan DBD terjadi melalui gigitan nyamuk
Aedes aegypti/Aedes albopictus betina. (2) Nyamuk ini
sebelumnya telah membawa virus dalam tubuhnya
dari penderita demam berdarah lain. (3) Nyamuk
tersebut berasal dari Brazil dan Ethiopia dan sering
menggigit manusia pada waktu pagi dan siang. (4)
Orang yang berisiko terkena demam berdarah
adalah anak-anak yang berusia di bawah 15 tahun.
(5) Orang yang tinggal di lingkungan lembap serta
daerah pinggiran kumuh juga berpotensi terkena
demam berdarah. (6) Penyakit DBD sering terjadi
di daerah tropis dan muncul pada musim
penghujan. (7) Virus ini kemungkinan muncul
akibat pengaruh musim/alam serta perilaku
manusia.
Perbaikan yang tepat untuk kalimat (3) adalah ...

(A) Nyamuk tersebut berasal dari Brazil dan


Ethiopia.
Nyamuk itu senang menggigit
manusia pada waktu pagi dan siang.
(B) Nyamuk tersebut berasal dari Brazil dan
Ethiopia, lalu sering menggigit manusia pada
waktu pagi dan siang.
(C) Nyamuk tersebut berasal dari Brazil dan
Ethiopia. Biasanya, nyamuk itu menggigit
manusia pada waktu pagi dan siang.
(D) Nyamuk tersebut berasal dari Brazil dan
Ethiopia.
(E) Nyamuk tersebut sering menggigit manusia
pada waktu pagi dan siang.
28. Kata berimbuhan yang dipakai pada konteks yang
tidak tepat ditemukan dalam kalimat ...

(A) Burung gereja beterbangan mendengar suara


lonceng berdentang.
(B) Anak-anak kejar-mengejar di halaman sekolah.
(C) Para penonton berlari-larian turun ke tengah
lapangan tatkala melihat api berkobar.
(D) Amran dan Rio selalu surat-menyurat melalui
e-mail.
(E) Pemandangan indah terhalang karena lembah
berselimutkan kabut.

205

29. Mikhael Novak (The Joy of Sports, 1976)


mengemukakan bahwa peristiwa olahraga
termasuk sepak bola, hadir seperti "liturgi" atau
upacara religius lainnya. Di sana ada tata cara yang
nyaris sakral dan mesti dihormati. Fungsinya pun
menyerupai fungsi liturgi: memberi manusia
perasaan kesatuan utuh dengan hal-hal di luar
dirinya, mengajari manusia untuk menghormati
"nasib" (kalah/menang), menampilkan model
kepahlawanan atau kepengecutan, dan akhirnya
merayakan kebersamaan itu sendiri.
Maksud pernyataan Mikhael Novak dalam teks di
atas adalah ...

(A) Olahraga
mempunyai
nilai
sebagaimana layaknya agama.
(B) Olahraga dan agama itu sama.

spiritual

(C) Kemiripan antara olahraga dan agama terletak


pada liturgi.
(D) Nilai sakral dalam olahraga harus dihormati.
(E) Nilai spiritual dalam
merayakan kebersamaan.

olahraga

adalah

30. Asap rokok yang baru mati di asbak mengandung


tiga kali lipat bahan pemicu kanker.
Kalimat di atas bukan kalimat yang efektif. Agar
menjadi efektif, kalimat tersebut dapat diperbaiki
menjadi ...

(A) Asap rokok yang baru mati terkandung tiga kali


lipat bahan pemicu kanker.
(B) Asap rokok yang baru mati di asbak
mengandung tiga kali lipat bahan yang
memicu kanker.
(C) Asap rokok yang baru dimatikan di asbak
mempunyai tiga kali lipat bahan pemicu
kanker.
(D) Asap rokok yang baru mati di asbak memiliki
tiga kali lipat bahan pemicu kanker.
(E) Asap dari rokok yang baru dimatikan di asbak
mengandung tiga kali lipat bahan pemicu
kanker.
31. Enam belas orang utan sumatra akan
dilepasliarkan di habitat aslinya di Taman nasional
Bukit Tigapuluh, Jambi. Makna dilepasliarkan dalam
kalimat tersebut adalah ...
(A) dilepas lalu dibiarkan hidup bebas.
(B) dijadikan binatang liar.
(C) dipelihara di alam liar.
(D) diberikan kandang di hutan.
(E) dilatih untuk hidup di alam bebas.

c Universitas Indonesia

Downloaded from http://pak-anang.blogspot.com

Halaman 5 dari 11 halaman

Kode Naskah Soal:


32. Kalimat berikut yang bebas dari kesalahan
penulisan kata adalah ...

(A) Prihatmoro ingin mengunjungi bapaknya di


Medan pasca nikah bulan ini.
(B) Semua yang tinggal di blok itu ikut berbela
sungkawa atas meninggalnya Pak Wir yang
dikenal sangat dekat dengan mahasiswa.
(C) Tuhan Yang Mahakuasa akan melindungi siapa
pun yang suka menolong orang lain tanpa
pamrih.
(D) Pesan saya hanya satu: sebarluaskan informasi
penting ini ke segenap masyarakat yang tinggal
di daerah kumuh itu.
(E) Jarang yang menyadari bahwa kita berangkat
dari sudut-pandang yang berbeda dalam
menyikapi rencana kedatangan Miyabi ke
Indonesia.
33. Pulau Bali tersohor di seantero dunia karena
keindahan pemandangannya dan lebih lagi karena
kekayaan tradisi kesenian dan religinya. Kita dapat
mengira bahwa reputasi yang terpuji itu
sepenuhnya disebabkan oleh daya sorgawi yang
kini melekat pada Pulau Bali lebih dari sekadar
sarana pemasaran usang brosur-brosur pariwisata.
Dalam teks tersebut kata yang penulisannya tidak
sesuai dengan kaidah EYD adalah ...

(A) seantero.
(B) sekadar.
(C) sorgawi.

(D) religi.
(E) tersohor.

34. Penulisan kata dalam kalimat-kalimat berikut ini


tidak mengikuti aturan EYD, KECUALI ...

(A) Ketidakberpihakan Saudara pada semua partai


sangat kami hargai.
(B) Kami akan menindak-lanjuti laporan yang
disampaikan Ketua Tim Empat.
(C) Para
ahli
bahasa
itu
berusaha
meng-Indonesia-kan sejumlah kata asing.
(D) Kami tidak bertanggungjawab atas kecelakaan
bus antarprovinsi itu.
(E) Gerakan anti pemerintah itu berusaha
menyebarluaskan ideologi mereka.

205

36. Burung elang sangat dekat dengan kehidupan


masyarakat Arab. Penyair Arab sering membuat
puisi yang mengidentikkan burung elang dan kelas
masyarakat di sana. ... Di berbagai tempat umum
di Abu Dhabi, banyak dijumpai gambar diri sheikh
dengan burung elang perkasa bertengger di lengan.
Elang bagi masyarakat badawi Arab merupakan
satwa yang membantu kelangsungan hidup
mereka selama mengembara di gurun pasir.
Kalimat yang dapat mengisi bagian yang kosong
adalah ...

(A) Elang menjadi simbol Jazirah Arab lebih dari


kawasan mana pun.
(B) Elang dapat ditemukan di mana pun di Jazirah
Arab.
(C) Elang adalah lambang keperkasaan.
(D) Selain itu, orang Arab juga suka memelihara
elang.
(E) Elang merupakan lambang kekuasaan bagi
mereka.
37. Adanya politisasi terlihat pada masalah investasi
riil dari luar negeri. Pihak asing digambarkan
"menguasai" perekonomian nasional. Hal ini
terlihat pada kasus penjualan Indosat ke Temasek
Holdings dan peningkatan peran perusahaan
minyak asing. ..., kecuali pada saham perbankan
dan transaksi di bursa, nilai investasi asing di
Indonesia sangat kecil dibandingkan dengan
negara lain.
Konsentrasi kepemilikan asing, justru ada di
wilayah "panas"(sektor nansial, khususnya bursa)
sehingga uang mereka gampang menguap. ...,
ketika terjadi krisis nansial, Indonesia tetap
terkena. Nilai tukar rupiah jatuh hingga lebih dari
Rp12.000,00 per dolar AS.
Kata sambung yang tepat untuk menghubungkan
kalimat pada paragraf tersebut adalah ...

(A) walaupun demikian; oleh karena itu.


(B) sebaliknya; oleh karena itu.
(C) oleh karena itu; walaupun demikian.
(D) padahal; akibatnya.
(E) dengan demikian; di samping itu.

35. Cara penulisan kata berikut sesuai dengan EYD,


KECUALI ...

(A) khazanah.
(B) konkret.
(C) karier.

c Universitas Indonesia

(D) jadwal.
(E) sejarahwan.

Downloaded from http://pak-anang.blogspot.com

Halaman 6 dari 11 halaman

Kode Naskah Soal:


38. Gambang semarang sebenarnya merupakan
turunan dari gambang kromong yang populer di
Jakarta. Irama gambang semarang rancak,
mengentak, dan terkesan humoristik. Menurut
dugaan, gambang semarang merupakan versi
sederhana dari gambang kromong. Adapun
gambang kromong adalah ekspresi rindu dendam
cina peranakan pada kesenian leluhurnya. Itu
adalah musik hibrida: akulturasi dari musik Cina,
Betawi, Melayu, dan Deli. Saat ini, gambang
kromong memiliki nasib seperti nasib musik
tradisional lainnya, yaitu jarang berpentas.
Demikian pula dengan nasib gambang semarang.
Topik pembicaraan bacaan tersebut adalah ...

(A) gambang semarang dan gambang kromong.


(B) gambang kromong.
(C) gambang semarang.
(D) nasib gambang kromong.
(E) nasib musik tradisional.
39. Di samping itu, alat-alat kelengkapan yang dimiliki
DPR seperti badan kehormatan, komisi-komisi,
serta fraksi-fraksi yang ada, harus difungsikan
dengan baik. Kepentingan rakyat harus menjadi
prioritas utama dibandingkan kepentingan partai
maupun kepentingan-kepentingan pihak tertentu.
Di pihak lain, untuk anggota DPD,
memperjuangkan kepentingan daerah di tingkat
pusat merupakan tugas pokok. Peran anggota DPD
sangat dibutuhkan mengingat banyaknya aspirasi
daerah, khususnya daerah di luar Pulau Jawa, yang
kerap tidak terperhatikan oleh pemerintah pusat.
Pesan yang terdapat dalam teks tersebut adalah ...

205

40. Situ Gunung adalah salah satu lokasi wisata tertua


di Jawa Barat. Pada zaman Belanda, para noni dan
sinyo kerap berlibur di lokasi ini. Pada era
keemasan kegiatan pecinta alam 1970-an hingga
awal 1990, Situ Gunung merupakan salah satu
tempat favorit berkemping. Letaknya yang
tersembunyi di lembah di antara bukit-bukit itulah
yang menarik perhatian orang. Situ atau danau ini
berada pada ketinggian 1.023 meter dari
permukaan laut. Danau yang terbentuk dari
letusan Gunung Pangrango ini masih menyimpan
potensi ora dan fauna yang cukup kaya. Di
pohon-pohon sekitar danau, siamang kerap
bergelayut dari pohon ke pohon. Hal ini
menambah daya tarik Situ Gunung.
Pernyataan yang tidak sesuai dengan bacaan
tersebut adalah ...

(A) Lokasi wisata Situ Gunung sudah dikenal sejak


zaman Belanda.
(B) Situ Gunung termasuk dalam kawasan Gunung
Gede Pangrango.
(C) Pada masa lalu, remaja Belanda sering
berwisata ke Situ Gunung.
(D) Situ Gunung memiliki kekayaan ora dan
fauna.
(E) Noni dan sinyo memilih Situ Gunung sebagai
tempat berkemping.

(A) Alat-alat kelengkapan yang dimiliki DPR


hendaknya difungsikan secara optimal
sehingga benar-benar dapat mewakili rakyat.
(B) Peran anggota DPR dan DPD hendaknya dapat
menyuarakan hati rakyat karena mereka dipilih
oleh rakyat.
(C) Alat kelengkapan DPR seperti badan
kehormatan, komisi-komisi, serta fraksi-fraksi
hendaknya jangan melupakan kepentingan
partai.
(D) DPR sebagai wakil rakyat hendaknya
memperhatikan aspirasi rakyat yang berada di
daerah, khususnya yang berada di luar Pulau
Jawa.
(E) Kepentingan rakyat hendaknya diperjuangkan
oleh DPR, sementara kepentingan daerah di
tingkat pusat merupakan tugas pokok bagi
DPD.
c Universitas Indonesia

Downloaded from http://pak-anang.blogspot.com

Halaman 7 dari 11 halaman

Kode Naskah Soal:

205

BAHASA INGGRIS
Humans have been using wind power for at least 5,500 years to propel sailboats, sailing ships, operating windmills,
and most recently, generating electricity, though its uses have been limited by the winds speed and strength instability.
However, a new study by scientists at the Carnegie Institution and California State University compiled the rst-ever
global survey of wind energy available at high altitudes in the atmosphere. The researchers assessed potential for wind
power in terms of wind power density, which takes into account both wind speed and air density at dierent altitudes in
the worlds ve largest cities: Tokyo, New York, Sao Paulo, Seoul, and Mexico City. They found that there is a huge
amount of energy available in high altitude winds, especially near the jet streams. Jet streams are meandering belts of
fast winds at altitudes between 20 and 50,000 feet that shift seasonally, but otherwise are persistent features in the
atmosphere. Jet stream winds are generally steadier and 10 times faster than winds near the ground, making them a
potentially vast and dependable source of energy. Several technological schemes have been proposed to harvest this
energy, including tethered, kite-like wind turbines that would be lofted to the altitude of the jet streams.
____________________________
Gunakan Petunjuk A dalam menjawab soal nomor 41 sampai
nomor 42.
41. The topic of the paragraph is _______.

(A) future uses of wind power


(B) best locations to exploit wind energy source
(C) wind power: a new form of renewable energy
(D) the potential of wind power at high altitude
(E) wind power turbines and generators
42. The sentence that best ends the paragraph is
_______

(A) Up to 40 megawatts of electricity could be


generated by current designs and transmitted to
the ground via the tether.
(B) So, while high-altitude wind may ultimately
prove to be a major energy source, it requires
substantial infrastructure.
(C) Good selection of a wind turbine site is critical
to economic development of wind power.
(D) Meteorologists also use the jet streams as an aid
to forecast weather.
(E) Will then wind power tapped by high-ying
kites light up our homes?

c Universitas Indonesia

Downloaded from http://pak-anang.blogspot.com

Halaman 8 dari 11 halaman

Kode Naskah Soal:

205

No one doubts that the words we write or speak are an expression of our inner thoughts and personalities. But
beyond the meaningful content of language, a wealth of unique insights into an authors mind are hidden in the style of
a text - in such elements as how often certain words and word categories are used, regardless of context. When people
try to present themselves a certain way, they tend to select what they think are appropriate nouns and verbs, but they
are unlikely to control their use of articles and pronouns. These small words create the style of a text, which is less
subject to conscious manipulation.
Social psychologist James W. Pennebaker of the University of Texas at Austin Pennebaker developed a computer
program that analyzes text, called Linguistic Inquiry and Word Count (LIWC, pronounced Luke). The software has
been used to examine other speech characteristics as well, tallying up nouns and verbs in hundreds of categories to
expose buried patterns.
Most recently, Pennebaker and his colleagues used LIWC to analyze the candidates speeches and interviews during
last falls presidential election. The software counts how many times a speaker or author uses words in specic
categories, such as emotion or perception, and words that indicate complex cognitive processes. It also tallies up
so-called function words such as pronouns, articles, numerals and conjunctions. Within each of these major categories
are subsets: Are there more mentions of sad or happy emotions? Does the speaker prefer "I" and "me" to "us" and "we"?
LIWC answers these quantitative questions; psychologists must then gure out what the numbers mean. Before LIWC
was developed in the mid-1990s, years of psychological research in which people counted words by hand established
robust connections between word usage and psychological states or character traits.
The political candidates, for example, showed clear dierences in their speaking styles. John McCain tended to
speak directly and personally to his constituency, using a vocabulary that was both emotionally loaded and impulsive.
Barack Obama, in contrast, made frequent use of causal relationships, which indicated more complex thought
processes. He also tended to be vaguer than his Republican rival.
Gunakan Petunjuk A dalam menjawab soal nomor 43 sampai
nomor 47.
43. An appropriate title for the text is _______.

(A) Linguistic Analysis Program: A New Invention


(B) Psychological Analysis in US Presidents
Speeches
(C) How the Use of Articles and Pronouns Reveals
Our Characters.
(D) What Your Choice of Words Says About Your
Personality
(E) Low-level Words That Give Clues to Large-Scale
Behavior
44. According to the text, how does LIWC help analyze
president candidates speeches?

(A) It does the quantitative analysis, which was


later used by psychologist.
(B) It helps the psychologist reveal the candidates
hidden motives.
(C) It makes analysis of nouns and verbs used by
the candidates during their speeches.
(D) It counts how many inappropriate words are
used by the candidates.
(E) It provides psychological research on the
connection of word usage and peoples
personality.

c Universitas Indonesia

45. According to the text, which of the following words


would be able to be used to tell about our
personality?

(A) Provide
(B) Examination
(C) The

(D) Analysis
(E) Test

46. The word robust in paragraph 3 can be best replaced


by _______.

(A) denite
(B) vigorous
(C) frail

(D) strong
(E) slight

47. According to the text, which of the following


statements about LIWC is FALSE?

(A) The software cannot detect unconsious use of


words.
(B) The software is able to count certain words and
analyze the numbers.
(C) By using LIWC, we learn about the
characteristics of the US presidential
candidates.
(D) LIWCs result conrms that peoples characters
aect their word usage.
(E) The software works on the basis of the
psychologists analysis design.

Downloaded from http://pak-anang.blogspot.com

Halaman 9 dari 11 halaman

Kode Naskah Soal:

205

Dog agility is a dog sport in which a handler directs a dog through an obstacle course in a race for both time and
accuracy. Dogs generally run o-leash with no food or toys as incentives. The handler can touch neither dog nor
obstacles.____(48)____, the handlers controls are limited to voice, movement, and various body signals, requiring
____(49)____ training of the animal. In its simplest form, an agility course consists of a set of standard obstacles, laid out
by an agility judge in a design of his own choosing on a roughly 100 by 100-foot (30 by 30 m) area, with numbers
indicating the order in which the dog must complete the obstacles. Courses are ____(50)____ enough that a dog could
not complete them correctly without human direction. In ____(51)____ , the handler must assess the course, decide on
handling strategies, and direct the dog through the course, with precision and speed equally important. Many strategies
exist to ____(52)____ for the inherent dierence in human and dog speeds and the strengths and weaknesses of the
various dogs and handlers.
Gunakan Petunjuk A dalam menjawab soal nomor 48 sampai
nomor 52.
48. ....

(A) In contrast
(B) Consequently
(C) However

(D) Still
(E) Besides

49. ....

(A) exceptional
(B) exceptionally
(C) excepting
(D) exception
(E) excepted
50. ....

(A) regular
(B) thorny
(C) complicated
(D) straightforward
(E) interesting
51. ....

(A) compete
(B) competitor
(C) competitive
(D) competitively
(E) competition
52. ....

(A) compensate
(B) estimate
(C) appreciate

c Universitas Indonesia

(D) examine
(E) investigate

Downloaded from http://pak-anang.blogspot.com

Halaman 10 dari 11 halaman

Kode Naskah Soal:


Gunakan Petunjuk A dalam menjawab soal nomor 53 sampai
nomor 60.
53. "May I be of any assistance, madam?"
"_______."

205

57. This old house is a mess. The owner should


_______.

(A) xing the ceiling


(B) have the ceiling xed

(A) Id love to do this

(C) have xed the ceiling

(B) Mind your own business

(D) get the ceiling xing

(C) Theres nothing you can do

(E) be xing the ceiling

(D) Its beyond your knowledge


(E) Youre very kind, but Im just looking
54. "The earthquake happening at about 3 oclock
yesterday caused many people to panic."
"Including me, as I _______ on the third-oor of my
oce at that time."

(A) worked

58. "I heard that Dona was awarded a scholarship to


continue her study."
"Oh? Thats great. She _______ very pleased."

(A) will be
(B) could be
(C) might be

(D) must be
(E) needs to be

59. _______ as Het achterhuis in 1947, Anne Franks


journal was successfully dramatized by Frances
Goodrich and Albert Hackett and presented in
New York as The Diary of Anne Frank in 1956.

(B) have worked


(C) have been working
(D) was working

(A) Originally published in Dutch

(E) had worked


55. "Wheres the report, Ben? You told me it would be
ready by now. I need it for the board meeting this
afternoon."
"Dont worry, Jim. _______ before the board
meeting."

(B) Having originally published in Dutch


(C) Having had originally published in Dutch
(D) It was originally published in Dutch
(E) It was published originally in Dutch
60. Had there been a guidance to write the report on
our survey of Dengue Fever in several parts of East
Java, there would not have been variant styles of
reports. From the above sentence, we may conclude
that _______.

(A) I have it nished


(B) I have had it nished
(C) I will have it nished
(D) The report will nish

(A) we agreed to one style only

(E) The report has nished


56. "Did you receive our inquiry? When will we receive
your conrmation?"
"My apology. It seems that _______. Could you
possibly resend it?"

(B) the guidance had good results


(C) there was no xed format for the report
(D) the content of our reports was the same
(E) there were dierences in the topics of our report

(A) we mislay your letter


(B) your letter is mislaid
(C) we had mislaid your letter
(D) your letter has been mislaid
(E) your letter was mislaid

c Universitas Indonesia

Downloaded from http://pak-anang.blogspot.com

Halaman 11 dari 11 halaman

SELEKSI MASUK
UNIVERSITAS INDONESIA

SIMAK UI
KEMAMPUAN DASAR
Matematika Dasar
Bahasa Indonesia
Bahasa Inggris

206
Universitas Indonesia
2010

PETUNJUK UMUM
1. Sebelum mengerjakan ujian, periksalah terlebih
dulu, jumlah soal dan nomor halaman yang terdapat
pada naskah soal.
Naskah soal ini terdiri dari 12 halaman.
2. Tulislah nomor peserta Anda pada lembar jawaban
di tempat yang disediakan.
3. Tulislah kode naskah soal ini, pada lembar jawaban
di tempat yang disediakan. Kode naskah soal ini:

206

4. Bacalah dengan cermat setiap petunjuk yang


menjelaskan cara menjawab soal.
5. Pikirkanlah sebaik-baiknya sebelum menjawab tiap
soal, karena setiap jawaban yang salah akan
mengakibatkan pengurangan nilai (penilaian: benar
+4, kosong 0, salah -1).
6. Jawablah lebih dulu soal-soal yang menurut Anda
mudah, kemudian lanjutkan dengan menjawab
soal-soal yang lebih sukar sehingga semua soal
terjawab.

7. Tulislah jawaban Anda pada lembar jawaban ujian


yang disediakan.
8. Untuk keperluan coret-mencoret, harap
menggunakan tempat yang kosong pada naskah soal
ini dan jangan pernah menggunakan lembar
jawaban karena akan mengakibatkan jawaban Anda
tidak dapat terbaca.
9. Selama ujian, Anda tidak diperkenankan bertanya
atau meminta penjelasan mengenai soal-soal yang
diujikan kepada siapapun, termasuk kepada
pengawas ujian.
10. Setelah ujian selesai, Anda diharapkan tetap duduk
di tempat Anda sampai pengawas ujian datang ke
tempat Anda untuk mengumpulkan lembar jawaban.
11. Perhatikan agar lembar jawaban ujian tidak kotor,
tidak basah, tidak terlipat, dan tidak sobek.

PETUNJUK KHUSUS
PETUNJUK A:
Pilih satu jawaban yang paling tepat.

PETUNJUK B:
Soal terdiri dari 3 bagian, yaitu PERNYATAAN, kata SEBAB, dan ALASAN yang disusun berurutan.
Pilihlah:
(A) Jika pernyataan benar, alasan benar, dan keduanya menunjukkan hubungan sebab dan akibat
(B) Jika pernyataan benar, alasan benar, tetapi keduanya tidak menunjukkan hubungan sebab dan
akibat
(C) Jika pernyataan benar dan alasan salah
(D) Jika pernyataan salah dan alasan benar
(E) Jika pernyataan dan alasan keduanya salah

PETUNJUK C:
Pilihlah:
(A) Jika (1), (2), dan (3) yang benar
(B) Jika (1) dan (3) yang benar
(C) Jika (2) dan (4) yang benar
(D) Jika hanya (4) yang benar
(E) Jika semuanya benar

Kode Naskah Soal:


MATA UJIAN
TANGGAL UJIAN
WAKTU
JUMLAH SOAL

:
:
:
:

Matematika Dasar, Bahasa Indonesia, dan Bahasa Inggris


11 APRIL 2010
120 MENIT
60

Keterangan

206

Mata Ujian MATEMATIKA DASAR nomor 1 sampai nomor 20


Mata Ujian BAHASA INDONESIA nomor 21 sampai nomor 40
Mata Ujian BAHASA INGGRIS
nomor 41 sampai nomor 60

MATEMATIKA DASAR
Gunakan Petunjuk A dalam menjawab soal nomor 1 sampai
nomor 17.
1. lim

x2 + 3x x = ...

(A) 81

(A) 1

(B) 9

(B) 0
3
(C)
2
(D) 3

(C) 1
1
(D)
9
1
(E)
81

(E)
2. Pada acara peringatan HUT RI di suatu universitas,
diadakan acara pembagian hadiah. Saat
pendaftaran, setiap peserta diberi sebuah kupon
yang nantinya bisa ditukar dengan hadiah apabila
nomor yang dimilikinya sesuai dengan nomor yang
terambil pada saat pengocokan. Nomor-nomor
yang dibagikan terdiri dari suatu huruf yang
diikuti oleh dua angka yang berbeda dan angka
kedua haruslah bilangan ganjil. Banyaknya nomor
undian yang dibuat adalah ....

(A) 71
(B) 260
(C) 520

(D) 650
(E) 1.170

3. x1 dan x2 adalah bilangan bulat yang merupakan


akar-akar persamaan kuadrat
x2 (2p + 4)x + (3p + 4) = 0, di mana p adalah
suatu konstanta. Jika x1 , p, x2 merupakan tiga
suku pertama dari suatu deret geometri, maka
suku ke-12 dari deret geometri tersebut adalah ....

(A) 1
(B) 1

1 2
. Nilai k yang
2 5
memenuhi k. det(AT ) = det(A1 ) adalah ....

4. Diketahui matriks A =

(C) 6 + 2 5

(D) 6 2 5

5.
Himpunan penyelesaian dari pertidaksamaan
x2 2x + 1 |x + 1|, adalah ....

(A) {x R| x = 0}
(B) {x R| x > 0}
(C) {x R| x 0}
(D) {x R| x < 0}
(E) {x R| x 0}
6. Jika x + y + 2z = k, x + 2y + z = k dan
2x + y + z = k , k = 0, maka x2 + y 2 + z 2 jika
dinyatakan dalam k adalah ....

(A)
(B)
(C)
(D)
(E)

k2
16
3k 2
16
4k 2
17
3k 2
8
2k 2
3

(E) 4

c Universitas Indonesia

Downloaded from http://pak-anang.blogspot.com

Halaman 1 dari 12 halaman

Kode Naskah Soal:

10. Nilai dari lim (2 x + 1)

7.

4x 3 x + 2 = ....

Luas segitiga pada gambar adalah .... cm2

(A) 4(1 3)

(B) 4( 3 1)

(C) 4( 3 + 1)

(D) 2( 3 + 1)

(E) 2(1 3)
8. Seorang pengamat melihat kapal laut yang sedang
berlabuh dengan kedudukan menghadap menara.
Sudut deviasi pengamat terhadap bagian depan
kapal adalah 30 dan sudut deviasi terhadap
bagian belakang kapal adalah 60 . Tinggi
pengamat adalah 1,8 meter; ia berada di puncak
menara yang tingginya 22 m, sedangkan menara
tersebut dibangun pada ketinggian 21,2 m di atas
permukaan laut. Panjang kapal adalah ....

(A) 45 3

45 3
(B)
3

(C) 30 3
30
(D)
3

(E) 30 + 2 3
9. Diketahui segitiga ABC dengan panjang

AC = 4 + 2 3 dan BAC = 30 . Titik D berada


pada AB sedemikian sehingga CD tegak lurus
dengan AB dan panjang BD = 1. Nilai sin ABC =
....

(A) 3 + 2 3

(B) 2 + 3

1
(C)
1+ 3
2

1
(D)
2+ 6
4

2+ 6
(E)

(A)
3
(B)
4
(C) 1
7
(D)
4
(E)
11. Jika sin x = a, maka

1
2a2 3 +

1
a2

= ....

(A) sin2 x tan2 x


(B) sec2 x cos x
(C) cos 2x csc x
(D) sec 2x tan2 x
1 + cot x
(E)
sin x cos x
12. Seorang siswa diminta untuk menyelesaikan
persamaan x2 + bx + c = 0 , tetapi justru
menyelesaikan persamaan x2 + cx + b = 0, b dan c
bilangan bulat. Salah satu akar yang diperoleh
adalah sama dengan akar dari persamaan semula,
namun akar yang lain m kurangnya dari akar
kedua persamaan semula. b dan c jika dinyatakan
dalam m adalah ....

(A) b =
(B) b =
(C) b =
(D) b =
(E) b =

m 1
m1
, c=
2
2
m1
m1
, c=
2
2
m+1
m 1
, c=
2
2
m + 1
m1
, c=
2
2
m1
m+1
, c=
2
2

13. Sebuah akuarium memiliki dasar dan sisi-sisi yang


berbentuk persegi panjang dan tidak memiliki
tutup. Volume dari akuarium adalah 4 m3 . Lebar
dari dasar akuarium adalah 1 m. Untuk pembuatan
dasar akuarium diperlukan biaya sebesar
Rp10.000,00 per m2 , sedangkan untuk sisi-sisinya
diperlukan biaya sebesar Rp5.000,00 per m2 . Biaya
minimal yang diperlukan untuk membuat sebuah
akuarium adalah ....

(A) Rp20.000,00
(B) Rp40.000,00
(C) Rp50.000,00
c Universitas Indonesia

206

Downloaded from http://pak-anang.blogspot.com

(D) Rp60.000,00
(E) Rp80.000,00

Halaman 2 dari 12 halaman

Kode Naskah Soal:


14. Grak fungsi f (x) = mx2 + (n + 3)x + m + 2n + 4
memotong sumbu Y di titik A(0, 5) dan memotong
sumbu X di titik B dan C. Jika absis titik tengah B
dan C adalah 2 maka titik puncak grak tersebut
adalah ....

(A) (2,9)
(B) (9,2)
(C) (2,11)

Gunakan Petunjuk C dalam menjawab soal nomor 18 sampai


nomor 20.
18. Nilai x yang memenuhi
23x
log (x2 2x + 2) =

3
2
1
(B)
2
(C) 1

(A)

(2) 4x 4y
(3) 3x + 3y
(4) 8x + 2y

2
2 sin 4
17. Diketahui y(x) = 2 cos( x) cos 2 + sin( x)
,

cos 2
maka periode dari fungsi tersebut adalah ....

(C) 2

(D)
2
(E) 4

c Universitas Indonesia

Daerah yang diarsir adalah gambar himpunan


penyelesaian pembatasan suatu soal program
linier. Bentuk-bentuk di bawah ini yang mencapai
maksimum di A adalah ....
(1) 100x + 50y

1
(D)
2
3
(E)
2

(B)

19.

(E) 8x + 2

16. Jika sistem:


x + 2y = 3k
3x ky = 1
dan sistem:
kx y = 1
x2 + y = 1
mempunyai satu penyelesaian yang sama, maka
hasil kali semua nilai k yang memenuhi adalah ....

(A) 2

log 4 adalah ....

(1)

(E) (2,8)

(D) 4x + 1

1
3
(2) 1
2
(3)
3
1
(4)
4

(D) (11,2)

15. Misalkan fungsi f : didenisikan dengan


f (2x 3) = 4x2 + 2x 5 dan f adalah turunan
pertama dari f . Hasil dari f (2x 3) = ....

(A) 2x 7
(B) 2x 1
(C) 2x + 7

206

20. Diberikan sepasang persamaan 2x 3y = 13 dan


3x + 2y = b dengan 1 b 100, dan b bilangan
bulat. Misalkan n2 = x + y , dengan x dan y adalah
solusi dari persamaan di atas, yang berupa
bilangan bulat, maka nilai n yang memenuhi
adalah ....
(1) 4
(2) 3
(3) 1
(4) 2

Downloaded from http://pak-anang.blogspot.com

Halaman 3 dari 12 halaman

Kode Naskah Soal:

206

BAHASA INDONESIA
Gunakan Petunjuk A dalam menjawab soal nomor 21 sampai
nomor 40.
21. Kata berimbuhan yang dipakai pada konteks yang
tidak tepat ditemukan dalam kalimat ...

(A) Burung gereja beterbangan mendengar suara


lonceng berdentang.
(B) Anak-anak kejar-mengejar di halaman sekolah.
(C) Para penonton berlari-larian turun ke tengah
lapangan tatkala melihat api berkobar.
(D) Amran dan Rio selalu surat-menyurat melalui
e-mail.
(E) Pemandangan indah terhalang karena lembah
berselimutkan kabut.
22. Pemerintah harus menyadari riset bioteknologi
tidak akan berjalan tanpa dukungan dana. Untuk
meningkatkan hasil pertanian dan mencapai
ketersediaan pangan yang cukup, pemerintah perlu
memberikan bantuan dana untuk melakukan riset
mengenai bioteknologi. Riset bioteknologi ini
bukan hal yang murah. Pemerintah juga harus
memberikan dana. Kalau dana yang ada terbatas,
penelitian yang dilakukan juga jalan di tempat.
Selain mendukung dalam masalah dana,
pemerintah juga harus mendukung dalam
peraturan. Peraturan pemerintah mengenai
bioteknologi ini sudah ada, tetapi ada kendala,
yaitu komisi yang seharusnya terbentuk dua tahun
yang lalu sampai sekarang belum terbentuk.
Pertanyaan berikut tidak sesuai dengan isi kutipan
di atas, KECUALI ...

(A) Mengapa pemerintah perlu melakukan riset


bioteknologi?
(B) Apakah peraturan pemerintah mengenai
bioteknologi sudah dilaksanakan?
(C) Bagaimanakah cara meningkatkan hasil
pertanian?
(D) Kapankah
sebaiknya
melakukan
riset
bioteknologi?
(E) Bagaimanakah peranan pemerintah dalam
pelaksanaan riset bioteknologi?

23. Anda sering mengenakan earphone saat bekerja, di


jalan sambil menunggu bus, atau menjelang tidur?
Hati-hati, riset mengatakan bahwa penggunaan
earphone bisa berbahaya untuk kesehatan telinga
Anda. Berdasarkan sebuah penelitian baru-baru ini
di Amerika Serikat, sekitar 5,2 juta anak berusia
619 tahun terganggu pendengarannya gara-gara
terlalu sering terpapar musik keras. Salah satu
biang keladinya adalah pemakaian earphone selama
berjam-jam dengan volume keras. Para ahli
kesehatan di sana memperkirakan anak-anak
generasi "iPod" ini bakal lebih awal mengalami
presbiakusis, yakni ketulian akibat usia lanjut,
sekitar usia 40-an. Normalnya, kondisi ini baru
muncul sekitar usia 6070 tahun.
Pesan kutipan tersebut adalah ...

(A) Penggunaan earphone dapat mengganggu jam


kerja.
(B) Penggunaan earphone dapat berbahaya untuk
kesehatan telinga.
(C) Penggunaan earphone di jalan sambil menunggu
bus sangat berbahaya.
(D) Penggunaan earphone yang tidak terlalu sering
tidak berbahaya.
(E) Penggunaan
earphone
berjam-jam
mengakibatkan ketulian di usia 6070 tahun.
24. Cara penulisan kata berikut sesuai dengan EYD,
KECUALI ...

(A) khazanah.
(B) konkret.
(C) karier.

(D) jadwal.
(E) sejarahwan.

25. Makna meN-kan pada kalimat "Pemusik Jazz, Benny


Likumahuwa, mempromosikan Ambon sebagai
kota yang aman dan memiliki pesona" sama
dengan makna meN-kan pada ...

(A) Penerbit memublikasikan buku baru Dewi


Lestari.
(B) Dengan terpaksa, dia membukakan pintu
untuk pengamen itu.
(C) Nilai rupiah saat ini sangat menggembirakan.
(D) Yuanita memilih mematikan lampu.
(E) Mereka menegaskan bahwa acara tersebut akan
dihentikan.

c Universitas Indonesia

Downloaded from http://pak-anang.blogspot.com

Halaman 4 dari 12 halaman

Kode Naskah Soal:


26. Penulisan gabungan kata yang benar terdapat pada
kalimat berikut ini, KECUALI ...

(A) Kehebatan Cibaduyut sebagai sentra sepatu


kulit terdengar sampai ke mancanegara.
(B) Bus antarprovinsi JawaSumatra menjadi
salah satu alat transportasi yang diminati
penumpang.
(C) AC Milan berhasil mencapai semi nal pada
pertandingan musim panas yang lalu.
(D) Budi daya mutiara sekarang dapat dilakukan di
darat.
(E) Para peserta karnaval menggunakan busana
dan tata rias yang cerah.
27. Sejumlah kalimat berikut efektif, KECUALI ...

(A) Akan segera saya kirimkan surat ini ke alamat


yang kauinginkan.
(B) Pengembalian buku yang kamu pinjam tiga
bulan yang lalu sudah saya terima.
(C) Sudah dua minggu ini saya menunggu-nunggu
surat balasan darinya.
(D) Uang kuliah yang kamu butuhkan sudah saya
transfer.
(E) Dia akan membeberkan pesan rahasia dari istri
gelapnya di persidangan Kamis depan.
28. Kalimat berikut yang efektif adalah ...

(A) Jakarta tempatkan kesehatan sebagai prioritas.


(B) Harga sembako ditetapkan pemerintah atau
naik perlahan-lahan mendekati harga pasar.
(C) Dalam
menyampaikan
laporan
perlu
bukti-bukti konkret.
(D) Masuknya seluruh kekuatan politik dalam
kekuasaan merupakan gejala negatif dalam
demokrasi.
(E) Dikarenakan persoalan sepele,
terjadi
pertengkaran yang melibatkan beberapa
pemuda warga desa.

c Universitas Indonesia

206

29. Kalimat yang tidak efektif adalah ...

(A) Tindak kekejaman, kekerasan, dan menindas


orang kecil merupakan perbuatan tidak terpuji.
(B) Ketika saya datang, mereka sudah berkumpul
di halaman sekolah untuk menanti inspektur
upacara.
(C) Tata tertib ini tidak boleh diubah sampai ada
tata tertib baru yang disahkan oleh pimpinan.
(D) Sebagai mahasiswa, Anda diharapkan dapat
memberi keteladanan yang baik.
(E) Diperlukan orang yang sanggup berpikir kritis
dan tidak ekstrem.
30. Kalimat yang baik ditinjau dari segi ejaannya
adalah ...

(A) Perkembangan agama Islam pada abad 16


ditandai oleh munculnya faham tasawuf.
(B) "Kecermatan memilih kata sangat diperlukan
dalam kegiatan mengarang," begitu nasihat Pak
Is sewaktu saya belajar di Depok.
(C) Karena belum ada yang dirubah, peraturan ini
masih berlaku.
(D) Kreatitas harus diselaraskan dengan esiensi
dan efektitas.
(E) Sikap atau pandangan yang terlalu ekstrim
tidak semestinya mendapat tempat di sini.
31. Aku bagaikan kejatuhan bulan.
Kata kejatuhan dalam kalimat di atas berstruktur
sama dengan kata berimbuhan ke-an dalam kalimat
berikut, KECUALI ...

(A) Pak Dahlan kebakaran jenggot mendengar


namanya disebut-sebut.
(B) Tetangga saya tidak mengira kedatangan tamu
tak diundang tadi malam.
(C) Mata adikku kemasukan debu sehingga
memerah.
(D) Tiada yang risau akan kematian koruptor yang
menyengsarakan negara.
(E) Baru saja ia kepergian suaminya.

Downloaded from http://pak-anang.blogspot.com

Halaman 5 dari 12 halaman

Kode Naskah Soal:


32. Saat berbicara di depan kader Partai Demokrat di
Cikeas, Senin lalu, Presiden juga menyebutkan ada
orang yang mencoba mengirim lamaran melalui
ibu mertuanya.
Inti kalimat dari kalimat di atas adalah ...

(A) Presiden berbicara.


(B) Saat berbicara, Presiden menyebutkan ada
orang.
(C) Ada orang yang mencoba mengirim lamaran
melalui ibu mertuanya.
(D) Kader Partai Demokrat mencoba mengirim
lamaran melalui ibu mertuanya.
(E) Presiden menyebutkan ada orang.
33. Penelitian ini dilakukan terhadap tiga lm kartun
Jepang, yaitu Sailormoon, Dragon Ball, dan Magic
Knight Ray Earth, mengungkapkan bahwa 58,4%
anak yang telah menonton tayangan tersebut
menunjukkan sikap antisosial.
Kalimat di atas menjadi efektif jika ...

(A) Kata ini diganti dengan yang.


(B) Kata yang dihilangkan.
(C) Kata mengungkapkan dihilangkan.
(D) Kata terhadap diganti dengan pada.
(E) Kata tersebut diganti dengan itu.
34. Kendati demikian, kebiasaan ini masih kurang
efektif dibandingkan dengan cuci tangan
menggunakan sabun. Hal ini karena sabun dapat
meluruhkan lemak dan kotoran yang mengandung
kuman. Dengan penggunaan yang benar, semua
sabun memiliki efektivitas yang sama dalam
meluruhkan kuman-kuman penyebab penyakit.
Kalimat yang dapat mengisi bagian awal paragraf
adalah ...

206

35. (1) Bayi yang dilahirkan tidak dengan berat badan


rendah, pada umumnya mempunyai status gizi
saat lahir yang kurang lebih sama dengan status
gizi bayi di Amerika. (2) Akan tetapi, seiring
dengan bertambahnya umur ditambah
faktor-faktor lainnya, sebagian besar bayi tersebut
terus mengalami penurunan status gizi. (3) Puncak
penurunan terjadi pada umur kurang lebih 1824
bulan. (4) Pada kelompok umur inilah prevalensi
balita kurus (wasting) dan balita pendek (stunting)
mencapai titik tertinggi (Hadi, 2001). (5) Setelah
melewati umur 24 bulan, status gizi balita
umumnya mengalami perbaikan meskipun tidak
sempurna.
Kalimat yang memiliki kesalahan ejaan adalah ...

(A) kalimat 5.
(B) kalimat 4.
(C) kalimat 3.

(D) kalimat 2.
(E) kalimat 1.

36. Situ Gunung adalah salah satu lokasi wisata tertua


di Jawa Barat. Di zaman Belanda, para noni dan
sinyo kerap berlibur di lokasi ini. Pada era
keemasan kegiatan pecinta alam 1970-an hingga
awal 1990, Situ Gunung merupakan salah satu
tempat favorit berkemping. Letaknya yang
tersembunyi di lembah di antara bukit-bukit itulah
yang menarik perhatian orang.
Gagasan pokok paragraf tersebut adalah ...

(A) Letak Situ Gunung tersembunyi.


(B) Situ Gunung merupakan salah satu tempat
favorit berkemping.
(C) Para noni dan sinyo kerap berlibur di lokasi ini.
(D) Keindahan alam Situ Gunung mengundang
orang datang ke sana.
(E) Situ Gunung adalah salah satu lokasi wisata
tertua di Jawa Barat.

(A) Cuci tangan adalah kebiasaan yang sudah


berlaku umum.
(B) Kebiasaan cuci tangan sudah ada sejak masa
dahulu.
(C) Cuci tangan dengan menggunakan air telah
menjadi hal yang umum.
(D) Cuci tangan harus dibiasakan sejak masa kecil.
(E) Kebiasaan cuci tangan berdampak positif bagi
kesehatan.

c Universitas Indonesia

Downloaded from http://pak-anang.blogspot.com

Halaman 6 dari 12 halaman

Kode Naskah Soal:


37. Kemarau terjadi di berbagai teritori dengan mulai
meningginya temperatur. Kecenderungan
pengeringan dalam 30 tahun terakhir terlihat pada
hutan-hutan tropis basah Afrika dan Amerika
Selatan. Selain itu, pengeringan juga terjadi di
wilayah-wilayah yang telah kering sebelumnya,
seperti Eropa bagian selatan dan Amerika Utara
bagian barat. Di daerah-daerah basah, penguapan
terus muncul dalam curahan hujan yang deras. Hal
ini meningkatkan risiko banjir. Hujan musim
dingin menggantikan salju. Hal ini menimbulkan
suatu pertanda buruk bagi ratusan juta orang yang
bergantung pada lelehan salju di musim semi
untuk pasokan air mereka.
Pernyataan yang tidak sesuai dengan bacaan
tersebut adalah ...

(A) Temperatur bumi menjadi tinggi, menyebabkan


kekeringan di Afrika dan Amerika Selatan.
(B) Sebagian Eropa dan Amerika mengalami
kekeringan.
(C) Daerah-daerah basah mulai menghadapi
ancaman banjir.
(D) Ancaman kekurangan air mulai membayangi
daerah yang mengandalkan salju sebagai
sumber air.
(E) Salju di belahan bumi bagian selatan mulai
menipis.

206

39. (1) Penularan demam berdarah (DBD) terjadi


melalui gigitan nyamuk Aedes aegypti/Aedes
albopictus betina. (2) Nyamuk ini sebelumnya telah
membawa virus dalam tubuhnya dari penderita
DBD lain. Nyamuk tersebut berasal dari Brazil dan
Ethiopia dan sering menggigit manusia pada
waktu pagi dan siang. (4) Orang yang berisiko
terkena demam berdarah adalah anak-anak yang
berusia di bawah 15 tahun. (5) Orang yang tinggal
di lingkungan lembap serta daerah pinggiran
kumuh juga berpotensi terkena demam berdarah.
(6) Penyakit DBD sering terjadi di daerah tropis dan
muncul pada musim penghujan. (7) Virus ini
kemungkinan muncul akibat pengaruh
musim/alam serta perilaku manusia.
Kalimat yang tidak sesuai adalah ...

(A) kalimat 3.
(B) kalimat 4.
(C) kalimat 5.

(D) kalimat 6.
(E) kalimat 7.

38. (1) Sesaat sebelum tintir kehabisan cahaya, Tamin


masih dapat menikmati bentuk tubuh seorang
perawan yang terlentang di atas tikar bawah, tidur
pulas oleh kepayahan kerja sehari-hari. (2) Ia ingat
kembali, masih kecil gadis itu ketika ia pergi. (3) Ia
tidak pernah mengira, adiknya akan semanis itu
akhirnya. (4) Alangkah larasnya garis-garis wajah
itu dalam cahaya yang bertambah lemah. (5) Ia
pernah menjelajahi tanah yang jauh; ia tidak
pernah menemui gadis semanis Sumi.
Makna kata laras kalimat (4) dalam konteks wacana
tersebut adalah ...

(A) pantas
(B) cocok
(C) sesuai

c Universitas Indonesia

(D) sepadan
(E) serasi

Downloaded from http://pak-anang.blogspot.com

Halaman 7 dari 12 halaman

Kode Naskah Soal:

206

40. Indonesia tengah menjajaki kemungkinan kerja


sama di bidang bioteknologi dengan beberapa
negara, di antaranya, Iran, Korea Selatan, Belanda,
serta dengan sejumlah negara ASEAN. Kerja sama
tersebut meliputi bidang pertanian, energi, dan
kesehatan. "Negara-negara tersebut yang paling
berminat untuk bekerja sama dengan peneliti kita," kata
Ketua Konsorsium Bioteknologi Indonesia (KBI),
Dr. Ir. Bambang Prasetyo, M.Sc. di sela Konferensi
Internasional Bioteknologi di Bogor, Selasa (5/8).
Belanda dan Korea Selatan ingin bekerja sama
dalam bidang bioteknologi energi. Sementara, Iran
tertarik bekerja sama dalam bidang bioteknologi
kelautan. "Untuk bioteknologi bidang nonkelautan
mereka sudah menguasai dan sekarang ingin
bekerja sama dalam bidang kelautan," ujarnya.
Pernyataan berikut tidak sesuai dengan maksud
kalimat yang bercetak miring dalam kutipan di
atas, KECUALI ...

(A) Tidak semua negara mau bekerja sama dalam


bidang bioteknologi dengan peneliti Indonesia.
(B) Negara Iran, Korea Selatan, Belanda mau
bekerja sama dalam bidang pertanian, energi,
dan kesehatan.
(C) Negara Belanda dan Korea Selatan akan
mengadakan kerja sama bioteknologi dalam
bidang energi.
(D) Negara Iran, Belanda, dan Korea Selatan
berminat bekerja sama bioteknologi di bidang
kelautan.
(E) Negara Iran, Belanda, dan Korea Selatan
berminat bekerja sama bioteknologi di bidang
kelautan.

c Universitas Indonesia

Downloaded from http://pak-anang.blogspot.com

Halaman 8 dari 12 halaman

Kode Naskah Soal:

206

BAHASA INGGRIS
Insects, such as some moths and fruit ies, react to microbial infection in the same way as mammals; hence, they can
be used to test the eciency of new drugs and to reduce the need for animal testing. Neutrophils, a type of white blood
cell and part of the mammalian immune system, and haematocytes, cells that carry out a similar function in insects,
react in the same way to infecting microbes. Both the insect and mammalian cells produce chemicals with a similar
structure which move to the surface of the cells to kill the invading microbe. The immune cells then enclose the microbe
and release enzymes to break it down. Insects such as fruit ies, Greater Wax Moths and a type of Hawkmoth can be
used to test the ecacy of new antimicrobial drugs or to judge how virulent fungal pathogens are. It is now routine
practice to use insect larvae to perform initial testing of new drugs and then to use mice for conrmation tests.
Gunakan Petunjuk A dalam menjawab soal nomor 41 sampai
nomor 42.
41. The topic of the paragraph is _______.

(A) using insects for drug safety tests


(B) using fruit ies to check blood cells
(C) the dangers of insect testing
(D) the benets of moths
(E) reducing animal testing
42. The sentence that best ends the paragraph is
_______.

(A) Researchers will continue to explore the


similarities between insect and mammalian
responses so that insects can be used as models
to study dierent disease states in humans
(B) Dr.
Kevin Kavanagh from the National
University
of
Ireland
presented
his
research ndings at the Society for General
Microbiologys meeting at Heriot Watt
University, Edinburgh
(C) As well as reducing by up to 90% the number of
mice required, this method of testing is quicker
as tests with insects yield results in 48-72 hours
whereas tests with mice usually take 4-6 weeks
(D) Immune cells in insects and mammals are
structurally and functionally similar despite
being separated by over 400 million years of
evolution
(E) Some scientists, especially those who have
never been involved in the experiment,
nevertheless, refuse to believe that human
reaction can be substituted by moth reaction

c Universitas Indonesia

Downloaded from http://pak-anang.blogspot.com

Halaman 9 dari 12 halaman

Kode Naskah Soal:

206

In a basement lab at Stanford University Medical School, Iret-net Hor-irws mummy lay tightly wrapped in tattered
linen as a handful of scientists looked on. Starting with his feet, the scanner rotated around the mummy, snapping
X-ray-type images that appeared on nearby computer screens. The pictures, showing well-preserved bone structure,
were then mathematically manipulated to generate 3-D images that give a fuller picture of the skeleton.
The highly sophisticated scanning technology allows scientists to learn about the 5-foot-4-inch (163-centimeter-tall)
mummy in remarkable detail without doing invasive or damaging procedures. The digital images will show features
that relate to paleopathology, diseases that may have been suered by the individual, also mummication style and
patterns - how they may change through time. It will also be useful for teaching anatomy to everyone from small
children through medical school.
Iret-net Hor-irw is believed to be an ancient Egyptian priest, and his mummy belongs to the Fine Arts Museums of
San Francisco. It was dug up from a cemetery in Akhmim, on the east bank of the Nile. Akhmim, from which the
maternal relatives of the famous King Tut come, was an important provincial capital and the site of one of Egypts major
temples.
Scientists have not been able to pinpoint Iret-net Hor-irws age when he died or his cause of death. The scanning
tests may help them get a little closer. For now, they can only date him to around 500 B.C., just before the Persian
conquest, when the last native Egyptian dynasty ruled. It is one of the eras which is very poorly understood.
After scientists are nished with him, Iret-net Hor-irws mummy will be the centerpiece of an exhibit at the Legion
of Honor in San Francisco. The mummy has been out on loan from the Fine Arts Museums of San Francisco since 1944,
and the exhibit, "Very Postmortem: Mummies and Medicine," is considered his homecoming.
Gunakan Petunjuk A dalam menjawab soal nomor 43 sampai
nomor 47.
43. An appropriate title for the text is _______.

(A) Stanford Computer to Help Paleopathology


(B) An Exhibition of an Egyptian Priests Mummy
(C) New Technology to Find A Mummys Cause of
Death
(D) The Homecoming of The Fine Arts Museums
Collection
(E) Scanning Technology to Help Unwrap the
Mystery of the Mummy
44. The word invasive in paragraph 2 can be best
replaced by _______.

46. Which of the following statements about the text is


FALSE?

(A) The mummies bone structure is in ne


condition.
(B) The mummy can serve educational purposes.
(C) The mummy was thought to be one of the
Egyptian kings.
(D) The mummy belongs to an American museum.
(E) The scientists are able to tell the mummies
medical history.
47. The text is most probably found in _______.

(A) a leaet of a museum exhibition


(B) an article in a medical journal

(A) consistent

(C) a preface of an exhibition programme book

(B) surgical

(D) an article in a newspaper

(C) serious

(E) a preface of a medical book

(D) comprehensive
(E) invading
45. What can be learnt from the computer images
about the mummy?

(A) How long he had lived.


(B) How he died.
(C) His hierarchy in Egyptian temples.
(D) How he was mummied.
(E) How he had served King Tut.

c Universitas Indonesia

Downloaded from http://pak-anang.blogspot.com

Halaman 10 dari 12 halaman

Kode Naskah Soal:

206

A soak in a steamy tub, surrounded by candles sure seems like a great way to relax. But new research suggests that
burning certain kinds of candles may ____(48)____ indoor air pollutants. Researchers at South Carolina State University
studied petroleum-based and vegetable-source candles to determine their emissions. They let candles from dierent
manufacturers burn for up to six hours in a small box, and then ____(49)____ and analyzed substances released into the
air.
They found that paran-based candles the most popular kind emitted toxic chemicals like toluene and benzene.
Soybean and beeswax candles did not, according to the study. Lighting a paran candle once in a while is unlikely to
pose a health threat, but frequently lighting many candles in an unventilated space could ____(50)____ to problems, and
may aggravate asthma, cause allergy-like symptoms, or irritate the respiratory tract.
____(51)____, candles are not the only one potential source of indoor air pollution. Central heating and cooling
systems, as well as gas replaces, can generate indoor air pollution, too. People with respiratory problems such as
asthma and chronic ____(52)____ pulmonary disease should make an extra eort to cut down on indoor air pollution by
ventilating their home as naturally as possible by opening the windows.
Gunakan Petunjuk A dalam menjawab soal nomor 48 sampai
nomor 52.
48. ....

(A) reduce
(B) generate
(C) increase

(D) destroy
(E) irritate

49. ....

(A) collected
(B) collection
(C) collecting

(D) collectable
(E) collector

50. ....

(A) refer
(B) incline
(C) indicate

(D) lead
(E) reect

51. ....

(A) Hence
(B) Moreover
(C) Besides

(D) Although
(E) However

52. ....

(A) obstructive
(B) obstruct
(C) obstructing
(D) obstructively
(E) obstructed

c Universitas Indonesia

Downloaded from http://pak-anang.blogspot.com

Halaman 11 dari 12 halaman

Kode Naskah Soal:


Gunakan Petunjuk A dalam menjawab soal nomor 53 sampai
nomor 60.
53. "Will you please hurry, our school bus leaves in ve
minutes."
"_______, I wont nish in time."

(A) Why dont you hurry


(B) Its about time
(C) You go ahead
(D) Youre pulling my leg
(E) Just forget it
54. "The earthquake happening at about 3 oclock
yesterday caused many people to panic."
"Including me, as I _______ on the third-oor of my
oce at that time."

(A) worked

206

57. Having seized some of the people related to the


bombing of the J.W. Marriot and Ritz-Charlton
hotels, _______.

(A) police investigation was continued to destroy


the network of the terrorists
(B) the terrorists network would be destroyed by
the police
(C) the police were determined to destroy the
network of the terrorists
(D) a continued investigation was conducted to
destroy the terrorists network
(E) destroying the tourists network was the polices
main objective
58. Tchaikovskys music, _______ with immense
technical skill and marked by emotional warmth,
lyrical melody, and colorful orchestration, has long
had wide appeal for the general public.

(B) have worked

(A) was written

(C) have been working

(B) whose written

(D) was working

(C) which it was written

(E) had worked

(D) written

55. _______ fund raising for helping the victims of


earthquake succeeded in collecting large amount of
money, many refugees still live in tents.

(A) Because
(B) As
(C) Before

(D) Even though


(E) Since

56. "Wheres the report, Ben? You told me it would be


ready by now. I need it for the board meeting this
afternoon."
"Dont worry, Jim. _______ before the board
meeting."

(A) I have it nished


(B) I have had it nished
(C) I will have it nished
(D) The report will nish
(E) The report has nished

c Universitas Indonesia

(E) it was written


59. "Did you receive our inquiry? When will we receive
your conrmation?"
"My apology. It seems that _______. Could you
possibly resend it?"

(A) we mislay your letter


(B) your letter is mislaid
(C) we had mislaid your letter
(D) your letter has been mislaid
(E) your letter was mislaid
60. Sharon would leave the key on the doorknob.
What does the sentence mean?

(A) There is a possibility that Sharon will leave the


key on the doorknob.
(B) Sharon will probably accidentally leave the key
on the doorknob.
(C) The speaker is certain that Sharon will leave the
key on the doorknob.
(D) It is typical of Sharon to leave the key on the
doorknob.
(E) Sharon left the key on the doorknob in the past.

Downloaded from http://pak-anang.blogspot.com

Halaman 12 dari 12 halaman

SELEKSI MASUK
UNIVERSITAS INDONESIA

SIMAK UI
KEMAMPUAN DASAR
Matematika Dasar
Bahasa Indonesia
Bahasa Inggris

207
Universitas Indonesia
2010

PETUNJUK UMUM
1. Sebelum mengerjakan ujian, periksalah terlebih
dulu, jumlah soal dan nomor halaman yang terdapat
pada naskah soal.
Naskah soal ini terdiri dari 13 halaman.
2. Tulislah nomor peserta Anda pada lembar jawaban
di tempat yang disediakan.
3. Tulislah kode naskah soal ini, pada lembar jawaban
di tempat yang disediakan. Kode naskah soal ini:

207

4. Bacalah dengan cermat setiap petunjuk yang


menjelaskan cara menjawab soal.
5. Pikirkanlah sebaik-baiknya sebelum menjawab tiap
soal, karena setiap jawaban yang salah akan
mengakibatkan pengurangan nilai (penilaian: benar
+4, kosong 0, salah -1).
6. Jawablah lebih dulu soal-soal yang menurut Anda
mudah, kemudian lanjutkan dengan menjawab
soal-soal yang lebih sukar sehingga semua soal
terjawab.

7. Tulislah jawaban Anda pada lembar jawaban ujian


yang disediakan.
8. Untuk keperluan coret-mencoret, harap
menggunakan tempat yang kosong pada naskah soal
ini dan jangan pernah menggunakan lembar
jawaban karena akan mengakibatkan jawaban Anda
tidak dapat terbaca.
9. Selama ujian, Anda tidak diperkenankan bertanya
atau meminta penjelasan mengenai soal-soal yang
diujikan kepada siapapun, termasuk kepada
pengawas ujian.
10. Setelah ujian selesai, Anda diharapkan tetap duduk
di tempat Anda sampai pengawas ujian datang ke
tempat Anda untuk mengumpulkan lembar jawaban.
11. Perhatikan agar lembar jawaban ujian tidak kotor,
tidak basah, tidak terlipat, dan tidak sobek.

PETUNJUK KHUSUS
PETUNJUK A:
Pilih satu jawaban yang paling tepat.

PETUNJUK B:
Soal terdiri dari 3 bagian, yaitu PERNYATAAN, kata SEBAB, dan ALASAN yang disusun berurutan.
Pilihlah:
(A) Jika pernyataan benar, alasan benar, dan keduanya menunjukkan hubungan sebab dan akibat
(B) Jika pernyataan benar, alasan benar, tetapi keduanya tidak menunjukkan hubungan sebab dan
akibat
(C) Jika pernyataan benar dan alasan salah
(D) Jika pernyataan salah dan alasan benar
(E) Jika pernyataan dan alasan keduanya salah

PETUNJUK C:
Pilihlah:
(A) Jika (1), (2), dan (3) yang benar
(B) Jika (1) dan (3) yang benar
(C) Jika (2) dan (4) yang benar
(D) Jika hanya (4) yang benar
(E) Jika semuanya benar

Kode Naskah Soal:


MATA UJIAN
TANGGAL UJIAN
WAKTU
JUMLAH SOAL

:
:
:
:

Matematika Dasar, Bahasa Indonesia, dan Bahasa Inggris


11 APRIL 2010
120 MENIT
60

Keterangan

207

Mata Ujian MATEMATIKA DASAR nomor 1 sampai nomor 20


Mata Ujian BAHASA INDONESIA nomor 21 sampai nomor 40
Mata Ujian BAHASA INGGRIS
nomor 41 sampai nomor 60

MATEMATIKA DASAR
Gunakan Petunjuk A dalam menjawab soal nomor 1 sampai
nomor 18.
1. Nomor pegawai pada suatu pabrik terdiri atas tiga
angka dengan angka pertama bukan nol.
Banyaknya nomor pegawai yang ganjil adalah ....

(A) 648
(B) 475
(C) 450

(D) 425
(E) 324

2. A adalah himpunan penyelesaian persamaan


2 cos 3x = 1, dengan 0 x 2. Banyaknya
himpunan bagian A dengan 3 anggota adalah ....

(A) 20
(B) 15
(C) 6

(D) 3
(E) 1

3. x1 dan x2 adalah bilangan bulat yang merupakan


akar-akar persamaan kuadrat
x2 (2p + 4)x + (3p + 4) = 0, di mana p adalah
suatu konstanta. Jika x1 , p, x2 merupakan tiga
suku pertama dari suatu deret geometri, maka
suku ke-12 dari deret geometri tersebut adalah ....

(C) 6 + 2 5

(D) 6 2 5
(E) 4
4. Banyaknya bilangan bulat negatif yang memenuhi
pertidaksamaan (x + 1)(x2 + 2x 7) x2 1
adalah ....

(A) 0
(B) 1
(C) 2

c Universitas Indonesia

(A)
(B)
(C)
(D)
(E)

k2
16
3k 2
16
4k 2
17
3k 2
8
2k 2
3

6. Banyaknya penyelesaian dari persamaan


sin x sin 1 x = 1 cos x dengan 0 x 2 adalah ....
2

(A) 0
(B) 1
(C) 2
7. Jika 81ab .45a+b =
....

(A) 1/2
(B) 1
(C) 3/2

(A) 1
(B) 1

5. Jika x + y + 2z = k, x + 2y + z = k dan
2x + y + z = k , k = 0, maka x2 + y 2 + z 2 jika
dinyatakan dalam k adalah ....

(D) 3
(E) 4

(D) 3
(E) 4

135, maka nilai dari 7a b =

(D) 2
(E) 3

8. Pada suatu penangkaran terdapat burung pipit dan


burung dara. Ketika 5 burung pipit dilepaskan,
jumlah burung dara dua kali burung pipit yang
tersisa. Kemudian, ketika 25 ekor burung dara
dilepaskan, burung pipit yang tersisa adalah 3 kali
burung dara yang tersisa. Jumlah burung pipit
semula adalah ....

(A) 20
(B) 25
(C) 30

Downloaded from http://pak-anang.blogspot.com

(D) 35
(E) 40

Halaman 1 dari 13 halaman

Kode Naskah Soal:

207

12. Jika jumlah 100 suku pertama suatu deret geometri


adalah , dan jumlah 200 suku pertamanya adalah
3, maka jumlah 700 suku pertamanya adalah ....

9.

(D) 63

(A) 8
(B) 10
(C) 15

Luas segitiga pada gambar adalah .... cm2

(A) 4(1 3)

(B) 4( 3 1)

(C) 4( 3 + 1)

(D) 2( 3 + 1)

(E) 2(1 3)
10. Jumlah nilai x yang memenuhi persamaan
x2
6
det(A) + det(B) = 0 di mana A =
2
x+3
x2 6x + 8 3
dan B =
adalah ....
4
1

(A) 3
(B) 2
(C) 0
(D) 1 1
2
(E) 2 1
2
11. Sebuah perusahaan membuat dua buah produk (X
dan Y ) dengan menggunakan dua buah mesin (A
dan B). Setiap unit X memerlukan 50 menit proses
pada mesin A dan 30 menit proses pada mesin B.
Setiap unit Y memerlukan 24 menit proses pada
mesin A dan 33 menit proses pada mesin B. Pada
kondisi awal, terdapat 30 unit X dan 90 unit Y di
dalam gudang. Mesin A dapat digunakan
maksimum 40 jam dan mesin B dapat digunakan
35 jam. Diprediksi akan ada permintaan 75 unit X
dan 95 unit Y . Sistem pertidaksamaan linier yang
mewakili situasi di atas adalah ....

(A) 50X + 24Y 40(60); 30X + 33Y 35(60);


x 0; y 5
(B) 50X + 24Y 40(60); 30X + 33Y 35(60);
x 45; y 5
(C) 50X + 24Y 40; 30X + 33Y 35(60);
x 0; y 5
(D) 50X + 24Y 40; 30X + 33Y 35;
x 45; y 5
(E) 50X + 24Y 40(60); 30X + 33Y 35(60);
x 0; y 0

c Universitas Indonesia

(E) 127

13. Jika f (x) = x dan h(x) = 2x + 1, dan

(f g h)(x) = 4x2 + 8x + 3 maka g(1) = ....

(D) 2

(A) 1
(B) 0
(C) 1

(E) 3

14. Jika p =3 log 4, maka nilai x yang memenuhi


2
2
persamaan 32x 7x+3 = 4x x6 apabila
dinyatakan dalam p adalah ....

(A)
(B)

1 + 2p
2p
1+

2
p

2p
1 2p
(C)
2p
1 + 2p
(D)
2+p
1 + 2p
(E)
1
2 p
15. Nilai minimum dari kuadrat jarak titik P (0, 3) ke Q
yang terletak pada parabola y = 2x2 + 1 adalah ....

(A) 4
(B)
(C)
(D)

15
16
49
64
3
4

(E) 0
16. Diketahui sebuah barisan mempunyai urutan suku
sebagai berikut: 120, 60, 40, 30, .... Suku berikutnya
adalah ....

(A) 24
(B) 25
(C) 30

Downloaded from http://pak-anang.blogspot.com

(D) 35
(E) 36

Halaman 2 dari 13 halaman

Kode Naskah Soal:

207

1 1 0
17. Jika A = 0 1 0 , maka jumlah dari semua
0 0 1
elemen pada matriks A2010 adalah ....

(D) 2013

(A) 2010
(B) 2011
(C) 2012

(E) 2014

18. Dua buah dadu dilempar secara bersamaan. x


adalah angka yang keluar dari dadu pertama, y
adalah angka yang keluar dari dadu kedua. Jika
A = {x, y|x + y < 2y < y + 2x, di mana sisa hasil
bagi (x + y) oleh 2 adalah 0}, maka nilai P (A)= ....

(A)
(B)
(C)
(D)
(E)

1
36
1
24
1
18
1
12
1
8

Gunakan Petunjuk C dalam menjawab soal nomor 19 sampai


nomor 20.
19. Diberikan sepasang persamaan 2x 3y = 13 dan
3x + 2y = b dengan 1 b 100, dan b bilangan
bulat. Misalkan n2 = x + y , dengan x dan y adalah
solusi dari persamaan di atas, yang berupa
bilangan bulat, maka nilai n yang memenuhi
adalah ....
(1) 4
(2) 3
(3) 1
(4) 2

ba
0
0
ab
a tidak
20. Matriks A = 0
0
a
a+b
mempunyai invers jika memenuhi ....
(1) a = 0, b = 0, dan a = b
(2) a = 0, b = 0, dan a = b
(3) a = 0 dan b sembarang
(4) b = 0 dan a sembarang

c Universitas Indonesia

Downloaded from http://pak-anang.blogspot.com

Halaman 3 dari 13 halaman

Kode Naskah Soal:

207

BAHASA INDONESIA
Gunakan Petunjuk A dalam menjawab soal nomor 21 sampai
nomor 40.
21. Michael Phelps, yang hiperaktif, selalu meminta
perhatian, tidak bisa fokus, dan bermasalah sejak
kecil, menjadi atlet terbesar Olimpiade sepanjang
masa.
Gagasan utama kalimat tersebut adalah ...

(A) Michael Phelps bermasalah, namun menjadi


atlet terbesar Olimpiade.
(B) Michael Phelps hiperaktif, meminta perhatian,
tidak bisa fokus, dan bermasalah.
(C) Michael Phelps menjadi atlet terbesar
Olimpiade.
(D) Michael Phelps selalu meminta perhatian dan
tidak bisa fokus.
(E) Michael Phelps yang hiperaktif dan
bermasalah.
22. Kalimat yang tidak efektif adalah ...

(A) Tindak kekejaman, kekerasan, dan menindas


orang kecil merupakan perbuatan tidak terpuji.
(B) Ketika saya datang, mereka sudah berkumpul
di halaman sekolah untuk menanti inspektur
upacara.
(C) Tata tertib ini tidak boleh diubah sampai ada
tata tertib baru yang disahkan oleh pimpinan.
(D) Sebagai mahasiswa, Anda diharapkan dapat
memberi keteladanan yang baik.
(E) Diperlukan orang yang sanggup berpikir kritis
dan tidak ekstrem.

c Universitas Indonesia

23. Miliaran burung dari belahan bumi utara


berpindah dalam kelompok besar ke bagian selatan
setiap musim dingin sebagai fenomena alamiah.
Perpindahan tersebut dipicu oleh perubahan cuaca
yang tajam dan berkurangnya makanan.
Burung-burung tersebut mengembara dari tempat
asal mereka di Eropa Utara, Asia Utara, bahkan
Alaska menuju wilayah yang hangat, daerah
tropika atau belahan bumi selatan. Akan tetapi,
tidak semua jenis burung melakukan perjalanan
panjang dan berbahaya ini. Ada sebagian burung
yang tidak melanjutkan migrasinya setelah
menempuh jalur yang singkat.
Burung-burung tersebut memasuki Indonesia dari
dua arah, yaitu utara dan selatan khatulistiwa. Dari
bagian utara, burung-burung tersebut masuk
melalui Semenanjung Malaya, Sumatera, dan
biasanya langsung ke bagian timur serta dari
Jepang, Taiwan lalu ke Filipina dan masuk wilayah
Indonesia.
Ringkasan dari kutipan tersebut adalah ...

(A) Miliaran burung dari belahan bumi utara


berpindah dalam kelompok besar ke bagian
selatan setiap musim dingin sebagai fenomena
alamiah. Burung yang datang dari arah selatan
berasal dari Australia.
(B) Perpindahan burung dari belahan bumi
utara merupakan sebuah fenomena alamiah.
Burung-burung tersebut berpindah untuk
mencari wilayah yang hangat.
(C) Burung-burung tersebut memasuki Indonesia
melalu dua arah, yaitu utara dan selatan.
(D) Miliaran burung dari belahan bumi utara
berpindah dalam kelompok besar ke bagian
selatan setiap musim dingin sebagai fenomena
alamiah. Burung-burung tersebut memasuki
Indonesia dari dua arah, yaitu utara dan selatan
khatulistiwa.
(E) Tidak semua burung melakukan migrasi
dengan jalur yang panjang dan berbahaya. Ada
burung yang menyudahi migrasinya setelah
menempuh jalur yang singkat.

Downloaded from http://pak-anang.blogspot.com

Halaman 4 dari 13 halaman

Kode Naskah Soal:


24. Kiper nomor satu Qatar dapat menangkap bola
dari penyerang Vietnam. Akan tetapi,
tangkapannya lemah sehingga bola pun menyusur
masuk ke dalam gawang sendiri. Pada penghujung
babak pertama, sebenarnya Qatar memiliki
peluang untuk menyamakan kedudukan melalui
kaki Adel. Namun, tendangannya masih lemah.
Tendangan itu sempat mengenai bek Vietnam
sehingga hanya menghasilkan tendangan penjuru.
Hingga babak pertama berakhir, kedudukan tetap
tidak berubah. Usai turun minum, tempo
permainan berjalan kian menarik. Penyerang Qatar
kembali mengancam gawang Vietnam. Tendangan
bebas dari Waleed Abdulla sangat terukur ke arah
gawang, namun Duong lebih cepat dalam
mengantipasi gerakan bola.
Gagasan utama pada bacaan di atas adalah ...

(A) Penyerang Qatar sering mengancam gawang


Vietnam.
(B) Tendangan bebas penyerang Qatar mampu
diantsipasi Duong.
(C) Tendangan bebas Waleed Abdulla sangat
terukur.
(D) Tempo permainan berjalan kian menarik.
(E) Duong lebih cepat mengantisipasi gerakan
bola.
25. Ruly, seorang TKW di Taiwan asal Ponorogo,
memberikan keterangannya lewat SMS bahwa gaji
bersih yang diterimanya setiap bulan sebagai
pekerja rumah tangga dengan bekal ijazah SMA
adalah 10.000 dolar Taiwan (TWD) atau jika
dirupiahkan senilai 2,5 juta rupiah.
Gagasan utama kalimat di atas adalah ...

(A) Ruly seorang TKW di Taiwan asal Ponorogo.


(B) Seorang TKW di Taiwan asal Ponorogo
memberikan keterangan.
(C) Ruly memberikan keterangannya.
(D) Gaji bersih yang diterimanya setiap bulan.
(E) Gaji bersih pekerja rumah tangga adalah 10.000
dolar Taiwan (TWD).

c Universitas Indonesia

207

26. Kalimat-kalimat berikut tidak efektif, KECUALI ...

(A) Dalam membangun kelompok tertinggal


ini, biasanya banyak kendala sehingga perlu
dirumuskan model pendekatan yang tepat.
(B) Kajian ini masih berupa catatan sehingga
masih perlu disempurnakan lagi baik mengenai
materi dan cara pendekatannya.
(C) Maka orang-orang desa itu jumlahnya tidak
dihitung sesuai dengan jumlah mereka tetapi
dihitung jumlah tenaganya yang digunakan
untuk mengerjakan tanah.
(D) Kontrol tenaga kerja pada bidang desa dan
pertanian menuntut prasyarat terselenggaranya
kontrol atas tanah.
(E) Supermal Karawaci beroperasi setiap hari
mulai pukul 09.00 sampai dengan pukul 22.00.
27. Kalimat luas berikut yang efektif adalah ...

(A) Meskipun pernah tinggal di Indonesia pada


tahun 19611963 untuk melakukan riset
tentang minoritas Tionghoa, saya belum
pernah menginjakkan kaki di kawasan Bangka
Belitung, sebuah kawasan yang merupakan
pemukiman etnik Tionghoa.
(B) Saya pernah tinggal di Indonesia pada tahun
19611963 untuk melakukan riset tentang
minoritas Tionghoa, tetapi saya belum pernah
menginjakkan kaki di kawasan Bangka
Belitung, yaitu kawasan pemukiman etnik
Tionghoa.
(C) Sebenarnya, saya pernah tinggal di Indonesia
pada tahun 19611963 untuk melakukan riset
tentang minoritas Tionghoa, tetapi saya belum
pernah menginjakkan kaki di kawasan Bangka
Belitung, padahal kawasan ini merupakan
pemukiman etnik Tionghoa.
(D) Meskipun pernah tinggal di Indonesia pada
tahun 19611963 untuk melakukan riset
tentang minoritas Tionghoa, tetapi saya
belum pernah menginjakkan kaki di kawasan
Bangka Belitung, kawasan yang merupakan
pemukiman etnik Tionghoa.
(E) Bahkan, saya pernah tinggal di Indonesia
pada tahun 19611963 untuk melakukan
riset tentang minoritas Tionghoa, tetapi saya
belum pernah menginjakkan kaki di kawasan
Bangka Belitung, kawasan yang merupakan
pemukiman etnik Tionghoa.

Downloaded from http://pak-anang.blogspot.com

Halaman 5 dari 13 halaman

Kode Naskah Soal:


28. Imron patuh kepada orang tuanya senakal-nakalnya ia.
Bentuk se-nya pada kalimat berikut mempunyai
fungsi yang sama dengan se-nya dalam kalimat ...

(A) Jawablah pertanyaan ini sesanggup-sanggupnya kamu.


(B) Setinggi-tingginya bangau terbang, surutnya ke
kubangan jua.
(C) Agar tugas ini segera selesai, bekerja
secepat-cepatnya yang engkau mampu.
(D) Para siswa belajar serajin-rajinnya menjelang
ujian seleksi masuk perguruan tinggi.
(E) Jangan bersikap seenak-enaknya di sini.
29. Besarnya jumlah siswa/mahasiswa Indonesia di
Singapura dan Malaysia tentu tidak lepas dari
berbagai faktor. Pertama, siswa/mahasiswa
Indonesia tidak akan mengalami kejutan budaya
bila belajar di Singapura atau Malaysia. Dari segi
geogras, jarak tempuh dari Indonesia ke
Singapura dan Malaysia tidak terlalu jauh. Ketiga,
cuaca di Singapura, Malaysia, dan Indonesia sama.
Keempat, makanan yang ada di Malaysia,
Singapura, dan Indonesia juga tidak jauh berbeda.
Kalimat yang tepat untuk bagian awal paragraf
adalah ...

(A) Negeri favorit mahasiswa Indonesia untuk


studi adalah Singapura dan Malaysia.
(B) Observasi menunjukkan bahwa jumlah
siswa/mahasiswa Indonesia yang studi di
Singapura dan Malaysia banyak.
(C) Banyak mahasiswa Indonesia bercita-cita untuk
belajar di Singapura dan Malaysia.
(D) Kenyataan membuktikan bahwa Singapura dan
Malaysia adalah dua negara yang diminati
untuk tujuan studi.
(E) Jumlah siswa/mahasiswa Indonesia di
Singapura dan Malaysia menarik untuk
diamati.

207

30. Iklim sedang berubah lebih cepat daripada yang


pernah terjadi sebelumnya. Kegiatan manusia
adalah penyebab utamanya. ..., pembakaran bahan
bakar fosil-minyak bumi, gas, batu bara-membanjiri
atmosfer dengan karbon dioksida yang menjebak
panas. Hal ini memicu kenaikan tajam pada
temperatur global rata-rata dalam abad terakhir.
Dampaknya adalah pada pola penguapan yang
berubah, gletser-gletser yang meleleh, badai-badai
yang menguat, dan kenaikan permukaan laut.
Kata sambung yang diletakkan pada bagian yang
kosong adalah ...

(A) akibatnya.
(B) oleh karena itu.
(C) misalnya.
(D) akhirnya.
(E) di sisi lain.
31. (1) Reduce berarti mengurangi atau mereduksi
sampah yang akan terbentuk. (2) Hal ini dapat
dilakukan apabila ibu-ibu rumah tangga kembali
ke pola lama, yaitu membawa keranjang belanja ke
pasar. (3) Dengan demikian, jumlah kantong plastik
yang dibawa ke rumah akan berkurang (tereduksi).
(4) Selain itu, bila setiap orang lebih memilih
menggunakan saputangan daripada tisu, terjadi
penghematan terhadap bahan baku untuk tisu. (5)
Jika setiap orang melakukan hal tersebut, beberapa
ton sampah yang akan tereduksi per bulan dan
beberapa hasil hutan dapat diselamatkan.
Kalimat topik pada paragraf tersebut adalah ...

(A) Kalimat 5.
(B) Kalimat 4.
(C) Kalimat 3.

(D) Kalimat 2.
(E) Kalimat 1.

32. Seorang ahli hukum tata negara menyebut hal ini


sebagai bentuk "kudeta redaksional".
Makna kata kudeta pada kalimat di atas adalah ...

(A) penggalangan kekuasaan.


(B) penggulingan kekuasaan.
(C) pengambilalihan kekuasaan.
(D) perebutan kekuasaan dengan paksa.
(E) pengakuan kedaulatan.

c Universitas Indonesia

Downloaded from http://pak-anang.blogspot.com

Halaman 6 dari 13 halaman

Kode Naskah Soal:


33. Penulisan gabungan kata berikut baku, KECUALI ...

(A) mitrabestari; matahati; tatabahasa.


(B) bilamana; saputangan; darmabakti.
(C) peribahasa; sumber daya; syahbandar.
(D) halalbihalal; acapkali; citarasa.
(E) dukacita; sukaria; tunarungu.
34. Penulisan kata dalam kalimat-kalimat berikut
benar, KECUALI ...

(A) Banyak pengguna komputer sudah memahami


copy dan paste.
(B) Ibu baru membeli baju bayi di toko My Baby.
(C) Saat melakukan drag, teks yang dikopi akan
berwarna menyala (highlight).
(D) Tiras Harian Kompas paling tinggi.
(E) Berikutnya, klik kanan di area yang sudah
disorot itu.
35. Kata berimbuhan yang dipakai pada konteks yang
tidak tepat ditemukan dalam kalimat ...

(A) Burung gereja beterbangan mendengar suara


lonceng berdentang.
(B) Anak-anak kejar-mengejar di halaman sekolah.
(C) Para penonton berlari-larian turun ke tengah
lapangan tatkala melihat api berkobar.
(D) Amran dan Rio selalu surat-menyurat melalui
e-mail.
(E) Pemandangan indah terhalang karena lembah
berselimutkan kabut.

207

37. Duvurger dalam bukunya berhasil memperjelas


konik-konik teoretis antara Marxisme dengan
Demokrasi Liberal. Ia berhasil menyoroti sesuatu
yang selama berabad-abad ini menjadi isu
intelektual dan praktis yang paling penting dalam
bidang politik. Isu ini masih relevan dengan situasi
politik yang ada pada sebagian negara di dunia.
Arti ungkapan yang paling sesuai untuk kata
relevan dalam paragraf di atas adalah ...

(A) sesuai.
(B) cocok.
(C) tepat.

(D) sepadan.
(E) bergayut.

38. Tindakan itu sudah tentu menangguk reaksi keras


dari semua kalangan.
Kata yang bermakna sama dengan kata menangguk
dalam kalimat di atas adalah ...

(A) menuai.
(B) mengundang.
(C) memanen.

(D) memperoleh.
(E) memancing.

39. Cara penulisan kata berikut sesuai dengan EYD,


KECUALI ...

(A) khazanah.
(B) konkret.
(C) karier.

(D) jadwal.
(E) sejarahwan.

36. Pemakaian tanda garis miring (/) berikut ini sesuai


dengan EYD, KECUALI ...

(A) Tahun akademik 2010/2011 akan segera kita


masuki.
(B) Acara akan berlangsung dari pukul 13.00 s/d
14.30.
(C) Nomor surat keterangan tersebut 21
PFL/XI/2008.
(D) Honornya Rp850.000,00/minggu.
(E) Jalan Kamboja X/4.

c Universitas Indonesia

Downloaded from http://pak-anang.blogspot.com

Halaman 7 dari 13 halaman

Kode Naskah Soal:

207

40. Vietnam gagal mencatatkan diri sebagai tim


pertama yang lolos ke babak perempat nal setelah
pada pertandingan kedua dipaksa bermain imbang
1-1 oleh Qatar di Stadion My Dinh Hanoi. Vietnam
yang memiliki nilai sempurna setelah mengalahkan
Uni Emirat Arab 2-0 di partai perdana harus
tertekan oleh Qatar sejak 10 menit paruh pertama.
Pada menit ke-20, penyerang naturalisasi Qatar,
Sebastian Andres Quintana, memiliki peluang
emas untuk mencetak gol pembuka. Namun
sayangnya, tendangan itu masih dapat dibaca
dengan baik oleh Duong Hong Son. Delapan menit
kemudian, Quintana kembali menjadi momok bagi
lini pertahanan Vietnam. Meskipun selalu
diserang, justru Vietnam yang mampu membuka
keunggulan terlebih dulu. Mereka mencetak gol 10
menit kemudian.
Pernyataan yang sesuai dengan bacaan tersebut
adalah ...

(A) Vietnam dikalahkan Uni Emirat Arab 2-0


sehingga harus mengalahkan Qatar.
(B) Pertandingan antara Vietnam dan Qatar
berakhir seri.
(C) Vietnam terus menyerang Qatar dalam
pertandingan tersebut.
(D) Qatar berhasil mencetak gol lebih dahulu
daripada Vietnam.
(E) Qatar lebih banyak menyerang Vietnam.

c Universitas Indonesia

Downloaded from http://pak-anang.blogspot.com

Halaman 8 dari 13 halaman

Kode Naskah Soal:

207

BAHASA INGGRIS
You have invited your friend over for dinner. Your child sees your friend reach for some cookies and says, "Better
not take those, or youll get even bigger." Youre embarrassed that your child could speak so rudely. However, you
should consider that your child may not know how to use language appropriately in social situations and did not mean
harm by the comment. An individual may say words clearly and use long, complex sentences with correct grammar, but
still have a communication problem if he or she has not mastered the rules for social language known as pragmatics.
Adults may also have diculty with pragmatics, for example, as a result of a brain injury or stroke. An individual with
pragmatic problems may say inappropriate or unrelated things during conversations, tell stories in a disorganized way,
or have little variety in language use. It is not unusual for children to have pragmatic problems in only a few situations.
However, if problems in social language use occur often and seem inappropriate considering the childs age, a
pragmatic disorder may exist. Pragmatic disorders often coexist with other language problems such as vocabulary
development or grammar.
Gunakan Petunjuk A dalam menjawab soal nomor 41 sampai
nomor 42.
41. The topic of this paragraph is _______.

(A) pragmatic disorder


(B) pragmatic aspect of language
(C) pragmatics, logic and semantics
(D) children with pragmatics diculties
(E) what causes pragmatics problems
42. The paragraph which follows this one most likely
discusses _______.

(A) pragmatic language tips


(B) how to test social pragmatic skills
(C) pragmatic disorder and language inhibition
(D) semantic-pragmatic disorder in children
(E) pragmatic
impairment
behavioral problems

c Universitas Indonesia

and

associated

Downloaded from http://pak-anang.blogspot.com

Halaman 9 dari 13 halaman

Kode Naskah Soal:

207

The thought of a car or truck that can drive itself is at once both exciting and frightening. Autonomous vehicle
navigation, as the technology is known, may make life more convenient if it allows people to kick back and enjoy a good
book or movie while their cars guide themselves through rush-hour trac. However, what happens if it starts to rain or
if trac suddenly picks up?
If the technology is to work at all, it will have to be completely safe on all roads, under all speeds, and in all weather.
Therein lies the challenge: if cars and trucks are to drive autonomously, they will need futuristic sensors and advanced
computing capabilities to respond to ever-changing road conditions.
Perhaps the most extreme example of ever-changing conditions is a war zone, where roads may be reduced to
rubble and vehicles are natural targets of attack. Rolling out eets of self-navigating vehicles for the military is an
enticing idea because it could keep thousands of troops out of harms way. Nevertheless, will it be possible for these
vehicles to operate in war zones? This question was the inspiration for a recent Defense Advanced Research Projects
Agency (DARPA) contest aimed at spurring the development of such technologies.
Held at a former air force base in Victorville, Calif. in late 2007, the DARPA Urban Challenge oered a $3.5 million
purse to competitors who could design the fastest and safest vehicles that could traverse a 60-mile urban course in
moving trac in less than six hours. The contestant vehicles were unmanned and had to complete a simulated military
supply mission, maneuvering through a mock city environment, avoiding obstacles, merging into moving trac,
navigating trac circles, and negotiating intersections - all while conforming to California driving rules. Of the 89
international teams that entered the challenge, only six nished in the allotted time.
Wende Zhang of General Motors was part of the team that designed the winning vehicle, which nished with the
fastest time - an average speed of approximately 13 miles per hour. The GM team drew upon existing technology
already oered in some of their vehicles that can assist in parking or detect lane markers and trigger alarms if the
drivers are coming too close to the shoulder of the road. For the DARPA challenge, they developed a more sophisticated
package of sensors that included GPS coupled with a camera and a laser-ranging LIDAR system to guide and correct the
vehicles route through the city. In Baltimore, Zhang will present GMs patented new methods for detecting lanes and
correcting a vehicles route, which helped them win the challenge.
Though they won, people should not look for robotic chaueurs immediately. The technology must prove reliable
in many dierent road, weather and lighting conditions. Still, says Zhang, a commercially-viable autonomous driving
product may be available in the next decade.
Gunakan Petunjuk A dalam menjawab soal nomor 43 sampai
nomor 47.

(A) It cannot function if it runs out of electricity.

43. The best title for this passage is _______.

(A) Future War Zone Vehicles


(B) A Giant Leap in automotive industry
(C) Research on the development of unmanned
vehicles
(D) Defense Advanced Research Projects Agency
(E) The Triumph
competition

of

General

Motors

44. What is stated about autonomous vehicle


navigation in the passage?

in

(B) Bad weather and trac jam are minor problems


for autonomous vehicles.
(C) Sixty teams managed to design autonomous
vehicles in the DARPA competition.
(D) Autonomous vehicles will be equipped with a
state-of-the art navigating system.
(E) Robotic chaueurs will be ready in the near
future.
45. It can be inferred from the passage that the DARPA
Urban Challenge _______.

(A) meant to search for an advanced manned


navigation system
(B) was only open for American teams
(C) gave a trivial prize to the winning team
(D) is to develop self-navigating vehicles for all sorts
of road conditions
(E) was not very competitive

c Universitas Indonesia

Downloaded from http://pak-anang.blogspot.com

Halaman 10 dari 13 halaman

Kode Naskah Soal:

207

46. The word "obstacles" in paragraph 4 is closest in


meaning to which of the following?

(A) Problems
(B) Diculties
(C) Obstructions

(D) Burdens
(E) Nuisances

47. Which of the following statements is FALSE?

(A) Futuristic sensors and advanced computing


abilities will be assential feature of autonomous
vehicles.
(B) Using self-navigating vehicles in the army will
reduce the number of injured soldiers in wars.
(C) The DARPA Urban Challenge looked for the
fastest and safest autonomous vehicle.
(D) The GM teams new detecting and navigating
methods played an important role in its victory.
(E) The U.S. Army is already well-prepared and
ready to launch unmanned vehicles in current
wars.

c Universitas Indonesia

Downloaded from http://pak-anang.blogspot.com

Halaman 11 dari 13 halaman

Kode Naskah Soal:

207

Doctors have recently started to study the causes of a medical disorder which they have appropriately named SAD,
or seasonal aective disorder. People who suer from SAD become very ____(48)____ during the winter months. Their
depression appears to be the result of a decrease in the amount of sunlight they are exposed to. Doctors theorize that
decreased sunlight aects the ____(49)____ of melatonin, a hormone manufactured in the brain, and serotonin, a
chemical that helps transmit nerve impulses. Depression may result from the ensuing imbalance of these two
substances in the body. ____(50)____, doctors believe that a decrease in the amount of sunlight the body receives may
cause a ____(51)____ in the bodys natural clock, which could, in turn, result in symptoms such as listlessness,
oversleeping, weight gain, anxiety, and irritability all symptoms of depression.
____(52)____ absence of light seems to be the cause of this disorder, a daily dose of light appears to be the cure.
Doctors advise patients to sit in front of a special light box which simulates natural light for a few hours every day.
Gunakan Petunjuk A dalam menjawab soal nomor 48 sampai
nomor 52.
48. ....

(A) depressed
(B) clear
(C) doubtful

(D) mad
(E) anxious

49. ....

(A) produce
(B) production
(C) productive

(D) productively
(E) produced

50. ....

(A) Therefore
(B) However
(C) Despite

(D) In addition
(E) Thus

51. ....

(A) disturbance
(B) danger
(C) result

(D) disease
(E) decrease

52. ....

(A) In fact
(B) When
(C) Besides

c Universitas Indonesia

(D) Since
(E) Moreover

Downloaded from http://pak-anang.blogspot.com

Halaman 12 dari 13 halaman

Kode Naskah Soal:


Gunakan Petunjuk A dalam menjawab soal nomor 53 sampai
nomor 60.
53. "Id like to nish entering the data today but I have
to catch the twelve-oclock train to Bandung."
"_______ ."

207

57. "Did you receive our inquiry? When will we receive


your conrmation?"
"My apology. It seems that _______. Could you
possibly resend it?"

(A) we mislay your letter


(B) your letter is mislaid

(A) You have it done

(C) we had mislaid your letter

(B) Intan has done it

(D) your letter has been mislaid

(C) Have you done it?

(E) your letter was mislaid

(D) It has to be done


(E) Have Intan do it?
54. "The earthquake happening at about 3 oclock
yesterday caused many people to panic."
"Including me, as I _______ on the third-oor of my
oce at that time."

(A) worked
(B) have worked

58. ______ a new policy related to the establishment of


business was released, entrepreneurs had found it
dicult to get a license for their new businesses.

(A) When
(B) After
(C) Since

(D) Until
(E) While

59. "Excuse me. Could you tell me which bus I should


take to get to Plaza Indonesia?"
"Sorry, I dont know. You _______ ask the driver."

(C) have been working


(D) was working

(A) could have

(E) had worked


55. "Wheres the report, Ben? You told me it would be
ready by now. I need it for the board meeting this
afternoon."
"Dont worry, Jim. _______ before the board
meeting."

(B) had better


(C) ought to have
(D) must have
(E) would rather
60. By the time Mr. Arin retires, his youngest child
will have graduated from university.
This means that Mr. Arin _______ when his
youngest son graduates.

(A) I have it nished


(B) I have had it nished
(C) I will have it nished
(D) The report will nish

(A) will still work

(E) The report has nished

(B) will retire

56. Last summer, he visited Nasau, a small but vibrant


city, with a great range of designer outlets
________ luxury goods at tax-free prices.

(A) that sells


(B) sold
(C) they sold

c Universitas Indonesia

(C) doesnt want to retire


(D) wants to work
(E) doesnt want to work

(D) selling
(E) to sell

Downloaded from http://pak-anang.blogspot.com

Halaman 13 dari 13 halaman

SELEKSI MASUK
UNIVERSITAS INDONESIA

SIMAK UI
KEMAMPUAN DASAR
Matematika Dasar
Bahasa Indonesia
Bahasa Inggris

208
Universitas Indonesia
2010

PETUNJUK UMUM
1. Sebelum mengerjakan ujian, periksalah terlebih
dulu, jumlah soal dan nomor halaman yang terdapat
pada naskah soal.
Naskah soal ini terdiri dari 12 halaman.
2. Tulislah nomor peserta Anda pada lembar jawaban
di tempat yang disediakan.
3. Tulislah kode naskah soal ini, pada lembar jawaban
di tempat yang disediakan. Kode naskah soal ini:

208

4. Bacalah dengan cermat setiap petunjuk yang


menjelaskan cara menjawab soal.
5. Pikirkanlah sebaik-baiknya sebelum menjawab tiap
soal, karena setiap jawaban yang salah akan
mengakibatkan pengurangan nilai (penilaian: benar
+4, kosong 0, salah -1).
6. Jawablah lebih dulu soal-soal yang menurut Anda
mudah, kemudian lanjutkan dengan menjawab
soal-soal yang lebih sukar sehingga semua soal
terjawab.

7. Tulislah jawaban Anda pada lembar jawaban ujian


yang disediakan.
8. Untuk keperluan coret-mencoret, harap
menggunakan tempat yang kosong pada naskah soal
ini dan jangan pernah menggunakan lembar
jawaban karena akan mengakibatkan jawaban Anda
tidak dapat terbaca.
9. Selama ujian, Anda tidak diperkenankan bertanya
atau meminta penjelasan mengenai soal-soal yang
diujikan kepada siapapun, termasuk kepada
pengawas ujian.
10. Setelah ujian selesai, Anda diharapkan tetap duduk
di tempat Anda sampai pengawas ujian datang ke
tempat Anda untuk mengumpulkan lembar jawaban.
11. Perhatikan agar lembar jawaban ujian tidak kotor,
tidak basah, tidak terlipat, dan tidak sobek.

PETUNJUK KHUSUS
PETUNJUK A:
Pilih satu jawaban yang paling tepat.

PETUNJUK B:
Soal terdiri dari 3 bagian, yaitu PERNYATAAN, kata SEBAB, dan ALASAN yang disusun berurutan.
Pilihlah:
(A) Jika pernyataan benar, alasan benar, dan keduanya menunjukkan hubungan sebab dan akibat
(B) Jika pernyataan benar, alasan benar, tetapi keduanya tidak menunjukkan hubungan sebab dan
akibat
(C) Jika pernyataan benar dan alasan salah
(D) Jika pernyataan salah dan alasan benar
(E) Jika pernyataan dan alasan keduanya salah

PETUNJUK C:
Pilihlah:
(A) Jika (1), (2), dan (3) yang benar
(B) Jika (1) dan (3) yang benar
(C) Jika (2) dan (4) yang benar
(D) Jika hanya (4) yang benar
(E) Jika semuanya benar

Kode Naskah Soal:


MATA UJIAN
TANGGAL UJIAN
WAKTU
JUMLAH SOAL

:
:
:
:

Matematika Dasar, Bahasa Indonesia, dan Bahasa Inggris


11 APRIL 2010
120 MENIT
60

Keterangan

208

Mata Ujian MATEMATIKA DASAR nomor 1 sampai nomor 20


Mata Ujian BAHASA INDONESIA nomor 21 sampai nomor 40
Mata Ujian BAHASA INGGRIS
nomor 41 sampai nomor 60

MATEMATIKA DASAR
Gunakan Petunjuk A dalam menjawab soal nomor 1 sampai
nomor 17.

5.

1. Peluang Kris mendapat nilai A untuk matematika


adalah 0,6 dan untuk bahasa Inggris 0,7. Peluang
Kris hanya mendapatkan satu nilai A adalah ....

(A) 0,12
(B) 0,18
(C) 0,28

(D) 0,42
(E) 0,46

2. Jika (g f )(x) = 9x2 6x dan g(x) = x2 + 1,


maka f (2x + 3)= ....

(A) 6x + 4
(B) 6x + 10
(C) 2x + 4

(D) 2x + 1
(E) 3x + 1

3. x1 dan x2 adalah bilangan bulat yang merupakan


akar-akar persamaan kuadrat
x2 (2p + 4)x + (3p + 4) = 0, di mana p adalah
suatu konstanta. Jika x1 , p, x2 merupakan tiga
suku pertama dari suatu deret geometri, maka
suku ke-12 dari deret geometri tersebut adalah ....

(B) 1

1
2

(A) 125
(B) 25
(C) 5

50
105

, maka det(A3 ) = ....

(D) 25
(E) 125
3x2 7x+8
x2 +1

(A) {x |x 2 atau x 3}

(C) 6 + 2 5

(D) 6 2 5

(B) {x |2 x 3}
(C) {x |x 1 atau x 6}

(E) 4
4. A dan B pergi menonton konser musik di suatu
stadion yang mempunyai 8 pintu. Mereka masuk
dari pintu yang sama, tetapi keluar dari pintu yang
berbeda. Banyaknya cara yang dapat mereka
lakukan adalah ....

c Universitas Indonesia

6. Diketahui A =

7. Himpunan penyelesaian dari 1 <


adalah ....

(A) 1

(A) 28
(B) 224
(C) 448

Luas segitiga pada gambar adalah .... cm2

(A) 4(1 3)

(B) 4( 3 1)

(C) 4( 3 + 1)

(D) 2( 3 + 1)

(E) 2(1 3)

(D) {x |1 x 6}
(E) {x }

(D) 484
(E) 896

Downloaded from http://pak-anang.blogspot.com

Halaman 1 dari 12 halaman

Kode Naskah Soal:


8. Jika x + y + 2z = k, x + 2y + z = k dan
2x + y + z = k , k = 0, maka x2 + y 2 + z 2 jika
dinyatakan dalam k adalah ....

(A)
(B)
(C)
(D)
(E)

k2
16
3k 2
16
4k 2
17
3k 2
8
2k 2
3

(B)
(C)
(D)
(E)

3
2
5
3

(D) 2
(E) 3
13. Diketahui (f g)(x) = x2 4x + 2 dan g(x) = x 3.
Jika x1 dan x2 adalah nilai-nilai yang memenuhi
f (x) = 2, maka nilai x1 + x2 adalah ....

(D) a > 3
(E) a > 12

(D) 4

(A) 4
(B) 2
(C) 2

(E) 10

14. Sebuah kotak obat tanpa tutup alasnya berbentuk


persegi dan mempunyai volume 4000 cm3 . Luas
permukaan kotak obat minimum adalah ....

(A) 1800 cm2


(B) 1240 cm2

10. Jika a log 81b =

(A)

(A) 1
(C)

(A) a < 0
(B) a > 0
(C) a < 3
1
(a) 3a

12. Jika diketahui a log b (a log b) + (a log b) + ... = 2,


+
3
maka a log b + b log a2 = ....

(B)

9. Diketahui x1 dan x2 adalah akar-akar persamaan


x1 + 1 x2 + 1
2x2 + 6x + a = 0. Jika
+
< 2, maka
x2
x1
nilai a yang memenuhi adalah ....

dari

208

log 27a dengan a, b > 0, maka nilai


1

log (b) b

1
2

adalah ....

(D) 1100 cm2


(E) 1000 cm2
15. Diketahui sebuah barisan 2, 3, 4, 6, 6, 6, 10, 9, 8, 14,
12, 10,.... Jumlah 3n suku pertama, untuk n = 1, 2, 3,
4,...., dari barisan di atas adalah ....

2a
3b
3a
4b
a
b
3a
2b
4a
3b

(A) S3n = n (9 9n)


2
(B) S3n = n (9 + 9n)
2
(C) S3n =

11. Jika a dan b adalah bilangan riil dengan 0 < a < b


ab
dan a2 + b2 = 8ab, maka
= ....
a+b

(A) 1 15
3

(B) 1 15
5

(C) 1 10
6

(D) 1 15
5

(E) 1 15
3

c Universitas Indonesia

(C) 1200 cm2

(D) S3n =
(E) S3n =

3n
2 (9 + 9n)
n
6 (9 + 9n)
n
2 (3 + 3n)

16. y = sin(sin(sin(sin .... sin(sin(x))....))) Tentukan


pada x = 0.

(A)
(B) 1
(C) 0

Downloaded from http://pak-anang.blogspot.com

dy
dx

(D) 1
(E)

Halaman 2 dari 12 halaman

Kode Naskah Soal:

208

17. Diketahui fungsi kuadrat f (x) = x2 4x + 5. Dua


buah garis singgung di titik yang merupakan
perpotongan antara f (x) dan garis y = 5
membentuk sebuah segitiga dengan garis y = 5.
Maka titik potong kedua garis singgung tersebut
adalah ....

(A) (3,2)
(B) (2,3)
(C) (2,3)

(D) (3,2)
(E) (3,2)

Gunakan Petunjuk C dalam menjawab soal nomor 18 sampai


nomor 20.
18. Pernyataan berikut yang BENAR berkaitan dengan
1
1

deret 1 + 21 3 + (23)2 + (23)3 + .... adalah


(1) deret tersebut
merupakan deret geometri
dengan rasio 2 + 3
(2) deret tersebut merupakan deret turun

(3) suku ke-5 nilainya sama dengan 97 + 56 3


(4) jumlah semua sukunya sama dengan

3
2

19. Suatu papan segiempat dibuat dengan spesikasi


sebagai berikut: 12 dm keliling < 20 dm dan
rasio dari sisi-sisi yang berdekatan (r) adalah
1 < r < 2. Jika ukuran papan diinginkan dalam
bilangan bulat, maka luas papan yang mungkin
adalah ....
(1) 12
(2) 15
(3) 20
(4) 16
20. Diberikan sepasang persamaan 2x 3y = 13 dan
3x + 2y = b dengan 1 b 100, dan b bilangan
bulat. Misalkan n2 = x + y , dengan x dan y adalah
solusi dari persamaan di atas, yang berupa
bilangan bulat, maka nilai n yang memenuhi
adalah ....
(1) 4
(2) 3
(3) 1
(4) 2

c Universitas Indonesia

Downloaded from http://pak-anang.blogspot.com

Halaman 3 dari 12 halaman

Kode Naskah Soal:

208

BAHASA INDONESIA
Gunakan Petunjuk A dalam menjawab soal nomor 21 sampai
nomor 40.
21. Tahukah Anda, Amerika Serikat dapat
mengalahkan Jepang dalam Perang Asia Timur
Raya bukan karena bom nuklir yang
menghanguskan Kota Hiroshima dan Nagasaki,
tetapi karena bisa memecahkan kode sandi rahasia
pesan-pesan yang digunakan oleh Jepang? Ya, bom
nuklir hanya senjata pamungkas untuk perang itu.
Namun, titik balik kemenangan justru karena Unit
Kombat Intelijen AS dapat memecahkan sandi
rahasia pesan yang digunakan oleh tentara Jepang.
Sejak itu, segala kekuatan pasukan, gerakan, tipu
daya, dan strategi Jepang diketahui persis oleh AS.
Paragraf di atas membicarakan ...

(A) AS dapat mengalahkan Jepang dalam Perang


Asia Timur Raya.
(B) AS menggunakan bom nuklir untuk
menghancurkan Kota Hiroshima dan Nagasaki.
(C) AS berhasil memecahkan sandi rahasia pesan
yang digunakan tentara Jepang dalam Perang
Asia Timur Raya.
(D) Titik balik kemenangan AS atas Jepang saat
Perang Asia Timur Raya.
(E) AS mengetahui kekuatan dan strategi Jepang
setelah dapat memecahkan sandi rahasia pesan
tentara Jepang.
22. 4 Februari 2004, Mark Zuckerberglulusan
Harvard dan mantan murid Ardsley High
Schoolmeluncurkan Facebook. Situs jejaring
sosial itu dengan cepat diminati penduduk dunia.
Kesuksesan situs itu menarik masyarakat dan
membuat sejumlah pemilik perusahaan teknologi
ingin memilikinya. Facebook itu juga yang
membuat Mark, pria kelahiran New York 14 Mei
1984 itu, menjadi orang kaya baru di Amerika
Serikat.
Kutipan di atas membicarakan ...

23. Sarkofagus adalah sejenis peti kubur dari batu utuh


yang terdiri atas wadah dan tutupnya. Jenis
makam tradisi megalitik ini bisa digunakan untuk
penguburan primer (langsung) atau sekunder
(dengan meletakkan tulang-belulang si mati),
penguburan tunggal atau ganda (satu kubur
dipakai bersama-sama). Sarkofagus banyak
ditemukan di Sumatra Utara, mulai dari Parapak
sampai ke Dolok Sanggul di tepian Danau Toba.
Ciri sarkofagus di daerah ini adalah bentuk
dasarnya yang melengkung, yang menyerupai
perahu, yang mirip dengan atap rumah di Sumatra
Utara.
Paragraf di atas menjelaskan ...

(A) sarkofagus sebagai makam yang berasal dari


tradisi megalitik.
(B) fungsi sarkofagus sebagai makam tradisi
megalitik.
(C) sarkofagus sebagai peninggalan dari tradisi
megalitik.
(D) keberadaan sarkofagus di Sumatra Utara.
(E) ciri sarkofagus yang berada di Sumatra Utara.
24. Kalimat yang tidak efektif adalah ...

(A) Tindak kekejaman, kekerasan, dan menindas


orang kecil merupakan perbuatan tidak terpuji.
(B) Ketika saya datang, mereka sudah berkumpul
di halaman sekolah untuk menanti inspektur
upacara.
(C) Tata tertib ini tidak boleh diubah sampai ada
tata tertib baru yang disahkan oleh pimpinan.
(D) Sebagai mahasiswa, Anda diharapkan dapat
memberi keteladanan yang baik.
(E) Diperlukan orang yang sanggup berpikir kritis
dan tidak ekstrem.

(A) Mark Zuckerberg meluncurkan Facebook.


(B) Facebook membuat Mark Zuckerberg menjadi
orang kaya baru di Amerika Serikat.
(C) Mark Zuckerberg sebagai seorang lulusan dari
Harvad.
(D) Facebook diminati oleh semua orang.
(E) Sejumlah pemilik perusahaan Facebook ingin
memiliki situs tersebut.

c Universitas Indonesia

Downloaded from http://pak-anang.blogspot.com

Halaman 4 dari 12 halaman

Kode Naskah Soal:


25. Penulisan huruf kapital yang benar terdapat pada
kalimat ...

(A) Tidak ada yang dapat membantah kebenaran


kata-kata Bu Lurah sebab memang yang
disampaikannya benar.
(B) Ya, Tuhan, atas karuniaMu-lah aku dapat
bertahan di tempat yang gersang ini.
(C) Salah satu hari penting bagi orang Bali yang
umumnya beragama Hindu adalah Hari
Galungan.
(D) Bagi orang Jawa, khususnya yang suka masak,
gula Jawa merupakan barang penting seperti
halnya bawang putih, bawang merah, dan
bumbu-bumbu lainnya.
(E) Bagi orang Jawa, R. Ng. Ranggawarsita bukan
seorang sastrawan biasa, melainkan Pujangga
yang dianggap Waskita.

208

28. Penulisan tanda baca yang benar terdapat pada


kalimat ...

(A) Saya akan datang, jika ada yang menjemput


saya dengan mobil hari ini.
(B) Dengan demikian dapat disimpulkan, bahwa
penyebab bencana ini terkait dengan ulah
manusia yang terlalu rakus, sehingga pohon di
hutan habis dibabat semua.
(C) Dia masih sempat membungkukkan badannya
untuk menghormati kita, meskipun hatinya
kesal sekali.
(D) Oleh karena itu, saya sempatkan untuk
mendatangi dan menyalaminya walaupun
badan saya hari ini belum sepenuhnya sehat.
(E) Ia tidak berani dan tidak sanggup menatap
matamu, karena merasa amat malu, terutama
setelah niatnya untuk juga mengawini
perempuan lain kamu ketahui.

26. Hal ini disebabkan karena banyak orang memilih


kereta rel listrik sebagai sarana transportasinya.
Kalimat yang tepat untuk memperbaiki kalimat di
atas adalah ...

(A) Hal ini disebabkan banyak orang memilih


kereta rel listrik sebagai sarana transportasinya.
(B) Hal ini dikarenakan oleh banyaknya orang
yang memilih kereta rel listrik sebagai sarana
transportasinya.
(C) Hal ini disebabkan oleh banyaknya orang
memilih kereta rel listrik sebagai sarana
transportasinya.
(D) Hal ini disebabkan oleh banyaknya orang
yang memilih kereta rel listrik sebagai sarana
transportasinya.
(E) Hal ini dikarenakan banyak orang memilih
kereta rel listrik sebagai sarana transportasinya.
27. Kata berimbuhan yang dipakai pada konteks yang
tidak tepat ditemukan dalam kalimat ...

(A) Burung gereja beterbangan mendengar suara


lonceng berdentang.
(B) Anak-anak kejar-mengejar di halaman sekolah.
(C) Para penonton berlari-larian turun ke tengah
lapangan tatkala melihat api berkobar.
(D) Amran dan Rio selalu surat-menyurat melalui
e-mail.
(E) Pemandangan indah terhalang karena lembah
berselimutkan kabut.

c Universitas Indonesia

Downloaded from http://pak-anang.blogspot.com

Halaman 5 dari 12 halaman

Kode Naskah Soal:


29. Temperatur permukaan bumi sedang memanas
dengan laju yang menentukan. Berbagai
perubahan iklim terkait dengan zaman penting
sebelumnya, yang disetel secara aktif oleh berbagai
penyebab alamiah, seperti variasi pada orbit bumi
yang memengaruhi jumlah sinar matahari. Dalam
kasus-kasus itu, siklus pendinginan dan
pemanasan berlangsung lambat selama
bermilenium-milenium. Saat ini, hal itu berbeda.
Iklim sedang berubah lebih cepat daripada yang
pernah terjadi sebelumnya. Kegiatan manusia
adalah penyebab utamanya. Sebagai contoh,
pembakaran bahan bakar fosilminyak bumi, gas,
batu baramembanjiri atmosfer dengan karbon
dioksida. Hal ini memicu kenaikan tajam pada
temperatur global rata-rata dalam abad terakhir.
Dampaknya adalah pada pola penguapan yang
berubah, gletser-gletser yang meleleh, badai-badai
yang menguat, dan kenaikan permukaan laut.
Ringkasan yang dapat dibuat dari bacaan tersebut
adalah ...

(A) Temperatur permukaan bumi menjadi panas


dengan laju yang menentukan, sedangkan
iklim berubah lebih cepat daripada yang
pernah terjadi sebelumnya.
(B) Temperatur
permukaan
bumi
sedang
memanas. Kegiatan manusia adalah penyebab
utama terjadi perubahan iklim.
(C) Pembakaran bahan bakar fosil mengakibatkan
naiknya temperatur permukaan bumi yang
mengakibatkan perubahan iklim.
(D) Kenaikan tajam pada temperatur global
mengakibatkan perubahan pola penguapan
yang terlihat pada melelehnya gletser,
menguatnya badai, dan menaiknya permukaan
laut.
(E) Pada masa lalu, perubahan iklim tidak
berlangsung cepat, tetapi secara alamiah.
30. Cara penulisan kata berikut sesuai dengan EYD,
KECUALI ...

(A) khazanah.
(B) konkret.
(C) karier.

c Universitas Indonesia

(D) jadwal.
(E) sejarahwan.

208

31. Melalui konser "Langit Ketujuh", penonton diajak


berpetualang menyusuri sebuah pertunjukan musik
kolosal.
Kata menyusuri bersinonim dengan ...

(A) menjelajahi.
(B) mengikuti.
(C) mengiringi.

(D) menjalani.
(E) menempuh.

32. Kalimat berikut yang efektif adalah ...

(A) Riset ini mengkaji tentang ide tentang


pemberantasan korupsi.
(B) Bukit itu amat tinggi, hijau, dan dengan
keindahan yang menakjubkan.
(C) Di Indonesia, bukan saja mempunyai kekayaan
alam yang melimpah, melainkan juga
keanekaragaman budaya.
(D) Kepada mahasiswa yang belum menyerahkan
makalah dapat menghubungi pengajar.
(E) Kegiatannya setiap hari antara lain menata
buku, membuat katalog, dan mengatur
peminjaman buku.
33. Kalimat yang efektif adalah ...

(A) Oleh karena itu, kita wajib menyantuni


orang-orang yang menderita akibat krisis
moneter.
(B) Berdasarkan
hasil
penelitian
kami
menunjukkan bahwa tidak ada penduduk di
desa ini yang kekurangan karena penghasilan
mereka cukup besar.
(C) Dengan sistem kuliah jarak jauh akan
memungkinkan
bertambahnya
jumlah
mahasiswa.
(D) Penulisan
biogra
pengarang
yang
ditempatkan di halaman terakhir buku
merupakan tanggung jawab seorang editor
selain menyunting naskah.
(E) Penyuntingan naskah membutuhkan tenaga
yang teliti, sabar, tekun, dan harus disediakan
waktu dan situasi yang kondusif untuk
menyunting naskah.

Downloaded from http://pak-anang.blogspot.com

Halaman 6 dari 12 halaman

Kode Naskah Soal:


34. Penetapan Angka Kredit (PAK) tahap I tahun 2009
yang merupakan hasil dari penilaian Daftar Usul
Penetapan Angka Kredit (DUPAK) yang diajukan
oleh 25 pustakawan dan 4 calon pustakawan sudah
diterbitkan.
Imbuhan pe-an pada kalimat berikut semakna
dengan imbuhan pe-an pada kata bercetak miring
dalam kalimat tersebut, KECUALI ...

(A) Pembukaan peringatan Hari Krida Pertanian


(HKP) yang ke-37 tahun 2009 menurut rencana
akan dilaksanakan secara serentak di seluruh
Indonesia pada hari Senin, tanggal 22 Juni 2009.
(B) Penghargaan diberikan kepada para mahasiswa
berprestasi.
(C) Penyusunan KTSP (Kurikulum Tingkat Satuan
Pendidikan) memerlukan waktu yang tidak
sedikit.
(D) Berdasarkan Kurikulum 2006, penilaian kognitif,
psikomotorik, dan afektif menjadi satu nilai
dalam rapor.
(E) Sekarang sudah tersedia penyaringan untuk
menjernihkan air sehingga kita tidak perlu
khawatir lagi.
35. Penulisan gabungan kata yang benar terdapat
dalam kalimat ...

(A) Senjata yang digunakan militer memanfaatkan


infra merah.
(B) Tawuran antarsiswa itu berhasil diselesaikan
dengan damai.
(C) Perusahaan itu membutuhkan pekerja yang
serba bisa.
(D) Tujuan daripada seminar itu mencari jalan ke
luar dari krisis.
(E) Kerjasama siswa saat ujian dilarang.

36. Dari sisi sosial, puasa melatih orang berdisiplin,


cinta keadilan, dan kedamaian. Dari sisi kejiwaan,
puasa membuat manusia pandai mengendalikan
diri. Dari sisi kesehatan, berpuasa berarti
mengistirahatkan saluran pencernaan, enzim, dan
hormon yang biasanya bekerja untuk mencerna
saluran makanan terus-menerus selama 18 jam.
Kalimat yang tepat untuk menjadi awal paragraf di
atas adalah ...

(A) Menjalankan ibadah puasa merupakan


kegiatan yang menguntungkan.
(B) Dari berbagai sisi, puasa merupakan kegiatan
yang menguntungkan.
(C) Ada beberapa manfaat yang diperoleh dalam
kegiatan berpuasa.
(D) Ibadah puasa, jika dijalankan dengan
kesadaran, memberi keuntungan dari berbagai
sisi.
(E) Dalam agama Islam, puasa wajib dilakukan
karena memberi manfaat pada manusia.
37. (1) Angka partisipasi di jenjang pendidikan tinggi
masih menjadi salah satu persoalan besar
pendidikan. (2) Sejauh ini, angka partisipasi
terbilang rendah, yakni 18 persen. (3) Dari total
jumlah penduduk berusia 1924 tahun, yakni
sekitar 25 juta orang, baru delapan belas persen
menikmati pendidikan tinggi. (4) Indonesia masih
jauh tertinggal dibandingkan dengan Malaysia
yang angka partisipasi di jenjang pendidikan
tingginya sudah 35 persen atau Thailand sekitar 45
persen. (5) Selain persoalan akses, pemerintah juga
masih bergulat dengan persoalan mutu dan
relevansi pendidikan.
Kalimat yang menyimpang dari topik kutipan di
atas terdapat pada kalimat ...

(A) kesatu.
(B) kedua.
(C) ketiga.

c Universitas Indonesia

208

Downloaded from http://pak-anang.blogspot.com

(D) keempat.
(E) kelima.

Halaman 7 dari 12 halaman

Kode Naskah Soal:


38. Vivienne Westwood (58) dikenal sebagai perancang
yang menggunakan karyanya sebagai cara
menyatakan pandangan politik atas isu yang ia
anggap penting, terutama yang menyangkut
lingkungan dan hak asasi. Dia pernah memprotes
kebijakan Presiden George Bush. Sebagai
perancang mode, Westwood berhasil
menggabungkan pandangan politiknya dalam
desain kreatif dan mewujudkannya menjadi
produk konkret.
Pernyataan yang benar tentang Vivienne Westwood
adalah ...

(A) Vivienne Westwood adalah perancang mode,


pemerhati lingkungan, sekaligus pembela hak
asasi.
(B) Vivienne Westwood selalu mengkritik aktivitas
para tokoh politik yang menyimpang.
(C) Vivienne Westwood, selain menjadi perancang
mode, juga menjadi pengamat politik.
(D) Vivienne Westwood,
perancang mode,
mewujudkan pandangan politiknya dalam
desain yang kreatif dan konkret.
(E) Vivienne Westwood,
perancang mode,
mempunyai pandangan politik yang senantiasa
diwujudkan dalam desain kreatif.

208

40. Solar sel sebagai alat yang dapat digunakan untuk


memanfaatkan energi matahari perlu ditingkatkan
pemakaiannya untuk mengatasi krisis minyak
bumi yang semakin menipis persediaannya.
Gagasan utama kalimat tersebut adalah ...

(A) Solar sel alat yang dapat digunakan untuk


memanfaatkan energi matahari.
(B) Solar sel perlu ditingkatkan pemakaiannya.
(C) Pemanfaatan solar sel perlu ditingkatkan untuk
mengatasi krisis minyak bumi.
(D) Solar sel perlu ditingkatkan pemakaiannya
untuk mengatasi krisis minyak bumi.
(E) Minyak bumi semakin menipis persediaannya.

39. Jika banyak seniman kontemporer menikmati


hidup di tengah kota besar, dia minggir dan tinggal
di kampung. Ketika para pelukis gencar
mengangkat budaya urban seraya berpameran dari
galeri ke galeri, dia berkelana di desa-desa demi
memberdayakan rakyat miskin lewat kesenian.
Saat sebagian seniman makin makmur dan
berjarak dari kehidupan nyata, dia masih setia
pada komunitas desanya.
Berdasarkan teks di atas, perumusan tentang "dia"
yang paling tepat adalah ...

(A) "Dia" adalah seniman yang senang tinggal di


kampung.
(B) "Dia" adalah pelukis yang mengangkat budaya
urban.
(C) "Dia" adalah pelukis sekaligus seniman setia
yang memperhatikan rakyat.
(D) "Dia"
adalah
seniman
setia
yang
memberdayakan rakyat miskin melalui
kesenian.
(E) "Dia" adalah seniman pengembara.

c Universitas Indonesia

Downloaded from http://pak-anang.blogspot.com

Halaman 8 dari 12 halaman

Kode Naskah Soal:

208

BAHASA INGGRIS
(1) ____________________________. (2) The rst requirement is to complete a course in a certied climbing school,
where you learn about mountain conditions and safe climbing. (3) Another requirement is to wear special clothing; a
helmet and climbing boots to complete your outt. (4) You must also carry special equipment: iron spikes to hammer
into cracks for support and safety and special climbing ropes. (5) In addition, there are also strict climbing rules for
mountaineers. (6) Mountaineers sometimes bring their mountain bikes. (7) You can climb alone or in teams of two or
more. (8) When you climb in a team, each person is tied around the waist to the same rope to protect a climber who slips
from falling. (9) At all times, you must be aware of falling rocks and when you are tired, you need to nd a ledge to rest
on. (10) Most mountain climbers would agree that mountain climbing has many rewards. (11) The higher you climb, the
more beautiful the view is of the valley oor. (12) You can see treetops, lakes, and distant mountains. (13) Above you,
the sky and moving clouds are a spectacular sight.
Gunakan Petunjuk A dalam menjawab soal nomor 41 sampai
nomor 42.
41. The paragraph should begin with _______.

(A) Mountain climbing is a dangerous sport with


special needs and strict climbing rules
(B) Mountain climbing requires not only special
clothing but also expensive equipment
(C) There are some requirements that you have to
meet before you go climbing
(D) Mountain climbing is risky; yet, many people
do it because of the rewards afterwards
(E) Mountain climbing is not a popular sport as it
needs rigorous preparation
42. The sentence which is irrelevant to the topic of the
paragraph is _______.

(A) sentence 4
(B) sentence 6
(C) sentence 9

c Universitas Indonesia

(D) sentence 11
(E) sentence 13

Downloaded from http://pak-anang.blogspot.com

Halaman 9 dari 12 halaman

Kode Naskah Soal:

208

By the age of 5, most triplets are on par with their peers in mental and emotional development, but those born at the
lowest weights may still lag behind, according to a study published in July 2009. Israeli researchers found that among
the 126 singletons, twins and triplets they followed from birth to age 5, triplets generally trailed behind their peers in
cognitive development over the rst two years of life. By age 5, however, many triplets had bridged the gaps in both IQ
and social development, the researchers report in the journal Pediatrics. The exception was triplets whod been
particularly growth-restricted in the womb those who, at birth, had weighed more than 15 percent less than the sibling
with the highest birth weight. At age 5, these children were still developmentally behind both their siblings and peers.
Until now, there had been no well-designed studies following the development of triplets over the rst few years of
life. And the catch-up seen among most triplets in this study is "excellent news," lead researcher Dr. Ruth Feldman, of
Bar-Ilan University, told Reuters Health. Parents of triplets, she said, should be aware that their children may be slower
to reach developmental milestones in infancy, but most are likely to close that gap during the preschool years. Children
who were born substantially smaller than their siblings may not catch up, however.
At age 5, the study found, these children typically scored at the lower end of the normal range for intellectual,
emotional and social development. For instance, Feldman explained, average verbal IQ was about 95, which, while
within normal range, would make it dicult for a child to get through standard schooling.
Growth restriction was common among the 21 sets of triplets in the study. In 65 percent, one sibling was born
weighing more than 15 percent less than the heaviest sibling. The ndings on development point to the importance of
giving these children extra attention from infancy onward, according to Feldman. "Knowing that these children respond
to parental investment already in the rst months of life tells parents to be especially sensitive and responsive to these
children," Feldman explained. In addition, she said, the childrens development during infancy and preschool should be
continuously monitored, and parents and children should receive extra help when needed such as interventions to
help children regulate their emotions and cultivate social skills, or to improve their attention and concentration abilities.
More studies are also needed, the researchers note, to see whether the developmental gaps persist into later childhood
and adolescence.
Gunakan Petunjuk A dalam menjawab soal nomor 43 sampai
nomor 47.
43. An appropriate title for the text is _______.

(A) Intellectual, Emotional and social development


of Triplets
(B) Triplets, Twins, and Singletons: A Comparison
Study
(C) Parental Guides to The Development of Babies
(D) The Reason for the Slow Growth of Triplets
(E) The Correlation between Birth-weight and
Intellectual Development
44. According to the text, which triplet-baby would
most probably be developmentally behind her
peers?

45. According to the text, when do triplets typically


have the biggest gap in development?

(A) During infancy


(B) When they reach the age of ve
(C) After they reach the age of ve
(D) During school years
(E) During preschool years
46. The word persist in paragraph 4 can be best
replaced by _______.

(A) vanish
(B) elapse
(C) stay

(D) change
(E) maintain

47. The text would most probably be found in _______.

(A) Born with less than 1.5 kg in weight

(A) an advertisement of a children health center

(B) Born weighing 20 percent less than her siblings

(B) an article in a medical journal

(C) Born with heaviest weight

(C) an article in a popular science magazine

(D) Born weighing 21 percent more than her


siblings
(E) Born with lowest weight

(D) an article in a Bar-Ilan University publication

c Universitas Indonesia

(E) a preface of a medical book

Downloaded from http://pak-anang.blogspot.com

Halaman 10 dari 12 halaman

Kode Naskah Soal:

208

One morning in August 1888, Bertha Benz and her two sons, Eugen (15), and Richard (13), woke up early. They got
dressed very quietly, ____(48)____ as not to waken the head of the family, Karl Benz. They left a note that said, "We are
going to visit Grandma." They crept out to Mr. Benzs workshop , opened the door, and pushed out a three-wheeled
____(49)____. It was the fruit of Mr. Benzs long and hard work - the rst engine car.
Imagine it: a wooden workshop ____(50)____, more similar to a horse carriage than to the cars we have now: no
roof, no hood, two wheels in the back but only one wheel in the front, a kind of handle instead of a steering wheel,
leather-covered seats, and, the most important part, a 2.5 horsepower single-cylinder four-stroke engine.
The car that Mrs. Benz and her sons rolled out of the workshop had gone through a great deal of development since
its registration (no. DRP 37435) two years before, but Mr. Benz felt it needed even more work and ____(51)____. He
believed that most people would not trust such a new thing enough to be willing to buy it, and so it seemed to him that
the prospects for any success on the market were not good. ____(52)____, his loving and energetic wife believed
immensely in his abilities, and was more than certain that the fruit was ripe enough to be picked.
Gunakan Petunjuk A dalam menjawab soal nomor 48 sampai
nomor 52.
48. ....

(A) so
(B) still
(C) besides

(D) however
(E) moreover

49. ....

(A) carriage
(B) bicycle
(C) wagon

(D) wheelbarrow
(E) vehicle

50. ....

(A) construct
(B) constructed
(C) constructive
(D) construction
(E) constructively
51. ....

(A) existence
(B) treatment
(C) enjoyment

(D) impression
(E) improvement

52. ....

(A) Thus
(B) Besides
(C) In contrast

c Universitas Indonesia

(D) Therefore
(E) Furthermore

Downloaded from http://pak-anang.blogspot.com

Halaman 11 dari 12 halaman

Kode Naskah Soal:


Gunakan Petunjuk A dalam menjawab soal nomor 53 sampai
nomor 60.
53. We recommend _______ information on the
internet or ocial tourism boards before you
decide where to go for your holiday.

208

57. The person recently elected headmaster of our


school is well-qualied, sociable, and _______ to his
job.

(A) his dedication is very high


(B) he is highly dedicating

(A) to search

(C) is dedicated highly

(B) search

(D) having a high dedication

(C) searching

(E) highly dedicated

(D) that we search


(E) you searching
54. "The earthquake happening at about 3 oclock
yesterday caused many people to panic."
"Including me, as I _______ on the third-oor of my
oce at that time."

58. "How do you like your new bedroom?"


"Well, it is too dark. I need _______ a brighter
colour."

(A) to have it painted


(B) to be painted
(C) painting it

(A) worked

(D) to have it paint

(B) have worked

(E) it to be painted

(C) have been working

59. _______, alternative therapies are being accepted by


more and more people, even the World Health
Organization.

(D) was working


(E) had worked
55. "Did you receive our inquiry? When will we receive
your conrmation?"
"My apology. It seems that _______. Could you
possibly resend it?"

(A) we mislay your letter


(B) your letter is mislaid
(C) we had mislaid your letter
(D) your letter has been mislaid
(E) your letter was mislaid
56. "Wheres the report, Ben? You told me it would be
ready by now. I need it for the board meeting this
afternoon."
"Dont worry, Jim. _______ before the board
meeting."

(A) I have it nished


(B) I have had it nished
(C) I will have it nished
(D) The report will nish
(E) The report has nished

c Universitas Indonesia

(A) Despite intolerant attitudes of the medical


establishment
(B) As the medical establishment has intolerant
attitudes
(C) Because of intolerant attitudes of the medical
establishment
(D) In case of intolerant attitudes of the medical
establishment
(E) Due to intolerant attitudes of the medical
establishment
60. "This cant be the way to Edinburgh."
What does the sentence mean?

(A) The speaker was told that it wasnt the way


Edinburgh.
(B) The speaker probably knows the other way
Edinburgh.
(C) The speaker doesnt know the way
Edinburgh.
(D) The speaker thinks that it isnt the way
Edinburgh.
(E) The speaker claims that it isnt the way
Edinburgh.

Downloaded from http://pak-anang.blogspot.com

to
to
to
to
to

Halaman 12 dari 12 halaman

SELEKSI MASUK
UNIVERSITAS INDONESIA

SIMAK UI
KEMAMPUAN DASAR
Matematika Dasar
Bahasa Indonesia
Bahasa Inggris

209
Universitas Indonesia
2010

PETUNJUK UMUM
1. Sebelum mengerjakan ujian, periksalah terlebih
dulu, jumlah soal dan nomor halaman yang terdapat
pada naskah soal.
Naskah soal ini terdiri dari 12 halaman.
2. Tulislah nomor peserta Anda pada lembar jawaban
di tempat yang disediakan.
3. Tulislah kode naskah soal ini, pada lembar jawaban
di tempat yang disediakan. Kode naskah soal ini:

209

4. Bacalah dengan cermat setiap petunjuk yang


menjelaskan cara menjawab soal.
5. Pikirkanlah sebaik-baiknya sebelum menjawab tiap
soal, karena setiap jawaban yang salah akan
mengakibatkan pengurangan nilai (penilaian: benar
+4, kosong 0, salah -1).
6. Jawablah lebih dulu soal-soal yang menurut Anda
mudah, kemudian lanjutkan dengan menjawab
soal-soal yang lebih sukar sehingga semua soal
terjawab.

7. Tulislah jawaban Anda pada lembar jawaban ujian


yang disediakan.
8. Untuk keperluan coret-mencoret, harap
menggunakan tempat yang kosong pada naskah soal
ini dan jangan pernah menggunakan lembar
jawaban karena akan mengakibatkan jawaban Anda
tidak dapat terbaca.
9. Selama ujian, Anda tidak diperkenankan bertanya
atau meminta penjelasan mengenai soal-soal yang
diujikan kepada siapapun, termasuk kepada
pengawas ujian.
10. Setelah ujian selesai, Anda diharapkan tetap duduk
di tempat Anda sampai pengawas ujian datang ke
tempat Anda untuk mengumpulkan lembar jawaban.
11. Perhatikan agar lembar jawaban ujian tidak kotor,
tidak basah, tidak terlipat, dan tidak sobek.

PETUNJUK KHUSUS
PETUNJUK A:
Pilih satu jawaban yang paling tepat.

PETUNJUK B:
Soal terdiri dari 3 bagian, yaitu PERNYATAAN, kata SEBAB, dan ALASAN yang disusun berurutan.
Pilihlah:
(A) Jika pernyataan benar, alasan benar, dan keduanya menunjukkan hubungan sebab dan akibat
(B) Jika pernyataan benar, alasan benar, tetapi keduanya tidak menunjukkan hubungan sebab dan
akibat
(C) Jika pernyataan benar dan alasan salah
(D) Jika pernyataan salah dan alasan benar
(E) Jika pernyataan dan alasan keduanya salah

PETUNJUK C:
Pilihlah:
(A) Jika (1), (2), dan (3) yang benar
(B) Jika (1) dan (3) yang benar
(C) Jika (2) dan (4) yang benar
(D) Jika hanya (4) yang benar
(E) Jika semuanya benar

Kode Naskah Soal:


MATA UJIAN
TANGGAL UJIAN
WAKTU
JUMLAH SOAL

:
:
:
:

Matematika Dasar, Bahasa Indonesia, dan Bahasa Inggris


11 APRIL 2010
120 MENIT
60

Keterangan

209

Mata Ujian MATEMATIKA DASAR nomor 1 sampai nomor 20


Mata Ujian BAHASA INDONESIA nomor 21 sampai nomor 40
Mata Ujian BAHASA INGGRIS
nomor 41 sampai nomor 60

MATEMATIKA DASAR
Gunakan Petunjuk A dalam menjawab soal nomor 1 sampai
nomor 19.
1. Sebuah kotak berisi 3 bola merah dan 7 bola hijau.
Secara acak, dilakukan pengambilan 2 bola tanpa
pengembalian. Peluang terambilnya 2 bola dengan
warna berbeda adalah ....

(A) 7/90
(B) 21/100
(C) 21/50

(D) 7/15
(E) 10/21

2. x1 dan x2 adalah bilangan bulat yang merupakan


akar-akar persamaan kuadrat
x2 (2p + 4)x + (3p + 4) = 0, di mana p adalah
suatu konstanta. Jika x1 , p, x2 merupakan tiga
suku pertama dari suatu deret geometri, maka
suku ke-12 dari deret geometri tersebut adalah ....

(A) 1
(B) 1

(C) 6 + 2 5

(D) 6 2 5

(E) 4
3. Nilai terbesar x
dalam 0 < x < 2 sehingga
cos 2x = 3 sin 2x adalah ....
2
2

(A)
(B)
(C)
(D)
(E)

13
12
7
6
8
6
19
12
11
6

4. Jika x + y + 2z = k, x + 2y + z = k dan
2x + y + z = k , k = 0, maka x2 + y 2 + z 2 jika
dinyatakan dalam k adalah ....

(A)
(B)
(C)
(D)
(E)

5. Suatu koloni bakteri membelah diri menjadi dua


setiap 6 jam dan pada setiap 12 jam seperempat
bagian dari koloni itu tidak bertahan hidup. Jika
pada awal pengamatan terdapat x bakteri, maka
setelah 36 jam, jumlah bakteri dalam koloni
tersebut adalah ....

(A) 6x
(B) 24x
(C) 27x

(D) 48x
(E) 64x

6. Dari huruf-huruf S, I, M, A, dan K akan disusun


kata-kata yang tidak selalu bermakna. Peluang
huruf vokal untuk selalu berdampingan adalah ....

(A)
(B)
(C)
(D)
(E)

c Universitas Indonesia

k2
16
3k 2
16
4k 2
17
3k 2
8
2k 2
3

1
5
2
5
1
2
3
5
4
5

Downloaded from http://pak-anang.blogspot.com

Halaman 1 dari 12 halaman

Kode Naskah Soal:


7. Himpunan penyelesaian dari
adalah ....

(2x7)8 (12+xx2 )3
(x2 +x2)6

(A) {x |x < 3 atau x > 4}


(B) {x | 3 < x < 4}
(C) {x | 3 < x < 2 atau 1 < x < 4}
(D)

x
x

(E)

|1 < x <
7
|2

7
2

atau x > 4

<x<4

8.

Luas segitiga pada gambar adalah .... cm

(A) 4(1 3)

(B) 4( 3 1)

(C) 4( 3 + 1)

(D) 2( 3 + 1)

(E) 2(1 3)
2

9. Jika log = a dan log 5 = b , maka


3
30
log(75 3 10) = ....

<0

209

10. Seorang ahli gizi sedang merencanakan dua buah


menu A dan B. Setiap gram dari menu A
mengandung 2 unit lemak, 1 unit karbohidrat, dan
4 unit protein. Setiap gram dari menu B
mengandung 3 unit lemak, 3 unit karbohidrat, dan
3 unit protein. Ahli gizi tersebut ingin menyiapkan
menu yang mengandung setidaknya 18 unit lemak,
12 unit karbohidrat, dan 24 unit protein dari menu
A dan B. Dengan memisalkan X adalah banyaknya
gram menu A yang dibuat dan Y adalah banyaknya
gram menu B yang dibuat, pertidaksamaan linier
yang mewakili situasi di atas adalah ....

(A) 2X + 3Y 18;
x 0; y 0
(B) 2X + 3Y 18;
x 0; y > 0
(C) 2X + 3Y 18;
x 0; y 0
(D) 2X + 3Y 18;
x 0; y 0
(E) 2X + 3Y 18;
x 0; y 0
11. Misalkan P =

p
r

X + 3Y 12; 4X + 3Y 24;
X + 3Y 12; 4X + 3Y 24;
X + 3Y 12; 4X + 3Y 24;
X + 3Y 12; 4X + 3Y 24;
X + 3Y 12; 4X + 3Y 24;
q
s

. Jika P 1 = 2P T , maka

ps qr = ....

(A) 1 atau 2

(A)

1+a+7b
3+a+b

(B)

1+3a+7b
3+a+b

(C)

1
(B) 1 atau 2
2

1
(C) 1 2 atau 2 2
2

(D) 2 atau 2

1+3a+7b
3+3a+3b

(E) 1 atau 1

(D)

1+7a+3b
3+a+b

(E)

1+7a+3b
3+3a+3b

12. Jika f (x) =

(A) 3
(B) 2
(C) 1

3
log x
12(3 log x) ,

maka f (x) + f

3
x

= ....

(D) 1
(E) 3

13. Persamaan kuadrat x2 px + q = 0 akar-akarnya m


dan n. Jika m, n, p, q merupakan barisan aritmetika,
m
= ....
maka
n
1
4
1
(B)
2
3
(C)
4
(D) 2

(A)

(E) 4

c Universitas Indonesia

Downloaded from http://pak-anang.blogspot.com

Halaman 2 dari 12 halaman

Kode Naskah Soal:


3
14. Jika cos x, sin x, dan 2 membentuk barisan
geometri, maka suku ke-5 dari barisan tersebut
adalah ....

1
(A) 18 3

(B)
(C)
(D)
(E)

1
2

1
3 3

19. Sebuah balon berbentuk bola sedang dipompa


sehingga volumenya bertambah 100 cm3 per detik.
Laju perubahan jari-jari balon ketika diameternya
mencapai 50 cm adalah ....

(A)
(B)

1
25
1
5

(C)

(D) 5

9
2

15. Jika diketahui x y = 21 3 , y z = 2+1 3 , maka


2
2
2
nilai x + y + z xy yz xz = ....

(A) 5
(B) 10
(C) 15

209

(D) 20
(E) 25

(x + 1)10
16. Jika diketahui f g h(x) =
,
(x + 1)10 + 1
x
f (x) =
dan h(x) = x + 3, maka g(x + 5)
x+1
adalah ....

(A) (x 2)10

(E) 25

Gunakan Petunjuk C dalam menjawab soal nomor 20 .


20. Diberikan sepasang persamaan 2x 3y = 13 dan
3x + 2y = b dengan 1 b 100, dan b bilangan
bulat. Misalkan n2 = x + y , dengan x dan y adalah
solusi dari persamaan di atas, yang berupa
bilangan bulat, maka nilai n yang memenuhi
adalah ....
(1) 4
(2) 3

(B) (x + 3)10

(3) 1

(C) (x + 5)

(4) 2

(D) (x 2)5
(E) (x 3)10
17. Diketahui sebuah barisan mempunyai urutan
sebagai berikut: 32,14,16,12,8,10,.... Suku ke-21 dari
barisan di atas adalah ....

(A) 6
(B) 2
(C)
(D)

1
32
1
2

(E) 2
18. Diketahui fungsi kuadratf (x) = x2 + x + 2. Dua
buah garis singgung di titik yang merupakan
perpotongan antara f (x) dan y = 2 membentuk
sebuah segitiga dengan garis y = 2. Luas dari
segitiga yang terbentuk adalah ....

(A)
(B)

1
4
1
2

(C) 1
(D)
(E)

3
2
5
2

c Universitas Indonesia

Downloaded from http://pak-anang.blogspot.com

Halaman 3 dari 12 halaman

Kode Naskah Soal:

209

BAHASA INDONESIA
Gunakan Petunjuk A dalam menjawab soal nomor 21 sampai
nomor 40.
21. Taman Wisata Candi Borobudur akan dijadikan
sebagai lembaga ... gajah dan untuk
memperbanyak ... gajah, tahun ini, Taman Wisata
Candi Borobudur akan menambah dua ekor gajah
lagi, jantan dan betina.
Kata yang tepat untuk mengisi kalimat rumpang
tersebut adalah ...

(A) preservasi, habitat.


(B) rehabilitasi, komunitas.
(C) pelestarian, spesies.
(D) konservasi, populasi.
(E) penelitian, habitat.
22. Dalam perjalanan itu kami mendatangi beberapa
tempat wisata.
Kata perjalanan pada kalimat di atas mempunyai
makna yang sama dengan kata pada kalimat
berikut, KECUALI ...

(A) Perbaikan rumah itu memakan waktu tiga


bulan.
(B) Beberapa orang warga mengikuti pertandingan
bulu tangkis.
(C) Para pendaki gunung itu membawa perbekalan
yang cukup.
(D) Persahabatan mereka sudah berjalan selama
sepuluh tahun.
(E) Pertemuan kedua sahabat itu sangat
mengharukan.
23. Sebelum konser dimulai, di backstage yang terletak
di bawah panggung ada acara pemberian
penghargaan multiplatinum pada The Black Eyed
Peas (BEP) dari Universal Music Indonesia karena
album mereka terjual lebih dari 150 ribu kopi.
Inti kalimat tersebut adalah ...

(A) Album BEP terjual lebih dari 150 ribu kopi.


(B) Universal Music Indonesia memberikan
penghargaan pada BEP.
(C) Penghargaan multiplatinum diberikan di
bawah panggung.
(D) Ada acara pemberian penghargaan.

24. Kalimat-kalimat berikut merupakan kalimat


efektif, KECUALI ...

(A) Sebelum menyerahkan tugas, Anda sebaiknya


memeriksanya secara cermat terlebih dahulu.
(B) Kami akan menyampaikan prakiraan cuaca
kota-kota besar yang berlaku besok.
(C) Panitia seminar itu belum menentukan di mana
rapat itu diselenggarakan.
(D) Mobil kami hampir kehabisan bensin ketika
sampai di Cirebon.
(E) Polisi berhasil menangkap pelaku perusakan
bantalan rel kereta api itu.
25. Sebanyak 109 spesies dalam hutan restorasi
Harapan Rainforest di perbatasan Jambi-Sumatera
Selatan terancam punah. Hal itu disebabkan
menurunnya kerapatan pepohonan dalam kawasan
hutan tersebut. Penelitian yang dilakukan selama
15 bulan di kawasan restorasi ekosistem
Jambi-Sumatera Selatan menunjukkan, keberadaan
ratusan jenis hewan dan tumbuhan terancam
punah, yaitu 293 spesies burung, 154 mamalia, 27
ambi, 42 reptilia, dan 444 spesies pohon.
Kalimat yang tepat untuk melanjutkan paragraf
tersebut adalah ...

(A) Masih banyak lagi jenis makhluk hidup lainnya


yang hampir punah.
(B) Sudah seharusnya pemerintah melindungi
spesies hewan di kawasan itu.
(C) Sangat disayangkan jika spesies yang
seharusnya dilindungi tersebut sampai punah.
(D) Sebaiknya penelitian terhadap hewan dan
tumbuhan juga dilakukan di tempat lain.
(E) Kepunahan hewan dan tumbuhan itu adalah
akibat ulah segelintir orang.
26. Cara penulisan gabungan kata yang benar adalah ...

(A) kerjasama.
(B) karya wisata.
(C) tatabahasa.
(D) anti komunis.
(E) simpang lima.

(E) Penghargaan multiplatinum diberikan sebelum


konser dimulai.

c Universitas Indonesia

Downloaded from http://pak-anang.blogspot.com

Halaman 4 dari 12 halaman

Kode Naskah Soal:


27. Kata-kata berikut yang semua ditulis secara baku
adalah ...

(A) azasi; akte; ijazah.


(B) zaman; khawatir; khusyu.
(C) lembap; antre; khaul.
(D) khazanah; khawatir; khuldi.
(E) trampil; paedagogis; energi.
28. Pada akhirnya, media hanyalah pilihan. Dari cat
minyak sampai arang, dari kayu sampai baja
antikarat, dari fotogra sampai seni rupa video,
dari cukil kayu sampai cetak digital, dan
seterusnya. Ketika sebuah karya seni rupa cat air
tak membuat kita berpikir bahwa karya ini dapat
dibuat lebih baik dengan cat minyak, ketika karya
seni gras cetak saring tak mengingatkan
sebaiknya karya ini dibuat dengan cukil nilo, dapat
dikatakan bahwa seniman memilih media yang pas
untuk dirinya.
Gagasan pokok paragraf tersebut terletak pada ...

(A) awal paragraf.


(B) akhir paragraf.
(C) awal dan akhir paragraf.
(D) seluruh paragraf.
(E) tengah paragraf.
29. Kalimat yang tidak efektif adalah ...

(A) Apabila ia ingin naik kelas, harus rajin belajar.


(B) Para tamu dipersilakan duduk.
(C) Menurut saya, masih banyak calon mahasiswa
yang bingung dalam menentukan pilihan studi
mereka.
(D) Tidak ada yang sulit dalam hidup ini, bukan?
(E) Mahasiswa yang belum lulus Bahasa Inggris
diberi kesempatan untuk mengikuti ujian
susulan.

c Universitas Indonesia

209

30. Kalimat yang tidak efektif dan ditulis dengan ejaan


yang tidak benar adalah ...

(A) Di sekitar tempat ini ada buah-buahan yang


dapat kita makan.
(B) Menurut penelitian menunjukkan bahwa
wanita tidak suka makan pisang Ambon.
(C) Konon laki-laki lebih menyukai buah durian
daripada buah kedondong.
(D) Kita membutuhkan kamus yang lengkap untuk
mendukung aktivitas kita sehari-hari yang
lebih banyak berurusan dengan kata.
(E) Sudah saatnya kita menggalakkan kegiatan
agar produktivitas dan kreativitas kita yang
selama ini menurun bisa pulih kembali.
31. Kunyit merupakan salah satu bumbu alami yang
bisa membantu mengatasi radang. Dilaporkan oleh
majalah Prevention bahwa kunyit mampu
membunuh sel-sel kanker, menunda pertumbuhan
tumor, dan mendorong efektivitas kemoterapi.
Beberapa riset menemukan bahwa kunyit efektif
jika dicampur dengan lada hitam dan minyak
zaitun. Kunyit yang sudah diolah dalam bentuk
bubuk hanya mengandung dua puluh persen dari
total kandungan aslinya. Jadi, lebih baik
mengonsumsi kunyit yang belum diolah.
Pesan berikut tidak sesuai dengan isi kutipan di
atas, KECUALI ...

(A) Untuk membunuh se-sel kanker di dalam tubuh


Anda, konsumsilah kunyit.
(B) Jika ingin merasakan kehebatan kunyit, cobalah
olahan kunyit setiap saat.
(C) Supaya kunyit mampu membunuh sel-sel
kanker, campurlah dengan lada hitam.
(D) Agar khasiat kunyit lebih baik, gunakanlah
kunyit hasil olahan sendiri.
(E) Apabila Anda ingin kepraktisan, nikmatilah
kunyit buatan sendiri.

Downloaded from http://pak-anang.blogspot.com

Halaman 5 dari 12 halaman

Kode Naskah Soal:


32. Cara penulisan kata serapan yang terdapat dalam
kalimat berikut salah, KECUALI ...

(A) Sejak dini anak-anak perlu dilatih dan


distimulir untuk mandiri agar kelak tidak
menjadi beban orang lain.
(B) Ia terlihat frustasi ketika anak yang
dibangga-banggakan selama ini ternyata
tidak lulus.
(C) Ia memiliki tipe wajah fotojenik sehingga
terpilih sebagai gadis sampul.
(D) Perundingan prelimineri diselenggarakan
sebelum nota kesepahaman ditandatangani.
(E) Tidak terlintas dalam benak saya bahwa Kadri
akan menjadi anggota kelompok penganut
kepercayaan yang ekstrem.
33. Kalimat yang efektif dan ditulis dengan ejaan yang
benar adalah ...

(A) Anak perlu dididik dengan benar agar tumbuh


menjadi anak yang kreatifnya tinggi.
(B) Jangan
mengandalkan
perkembangan
intelegensi anak semata-mata hanya pada
gurunya saja.
(C) Boleh memprotes pembangunan jalan ini, tapi
anda harus konsekuensi.
(D) Saputangan berdarah ini menjadi tanda bahwa
saya masih mencintainya.
(E) Dalam makalah singkat ini membahas ciri dan
perilaku lima etnik di Indonesia.
34. Kalimat yang tidak efektif adalah ...

(A) Tindak kekejaman, kekerasan, dan menindas


orang kecil merupakan perbuatan tidak terpuji.
(B) Ketika saya datang, mereka sudah berkumpul
di halaman sekolah untuk menanti inspektur
upacara.
(C) Tata tertib ini tidak boleh diubah sampai ada
tata tertib baru yang disahkan oleh pimpinan.
(D) Sebagai mahasiswa, Anda diharapkan dapat
memberi keteladanan yang baik.
(E) Diperlukan orang yang sanggup berpikir kritis
dan tidak ekstrem.

209

35. Kata berimbuhan yang dipakai pada konteks yang


tidak tepat ditemukan dalam kalimat ...

(A) Burung gereja beterbangan mendengar suara


lonceng berdentang.
(B) Anak-anak kejar-mengejar di halaman sekolah.
(C) Para penonton berlari-larian turun ke tengah
lapangan tatkala melihat api berkobar.
(D) Amran dan Rio selalu surat-menyurat melalui
e-mail.
(E) Pemandangan indah terhalang karena lembah
berselimutkan kabut.
36. Cara penulisan kata berikut sesuai dengan EYD,
KECUALI ...

(A) khazanah.
(B) konkret.
(C) karier.

(D) jadwal.
(E) sejarahwan.

37. Para penulis sering memilih dan menggunakan


makna kata yang berpretensi ilmiah tanpa
memperhatikan untuk siapa buku itu ditulis.
Kata berpretensi dalam kalimat tersebut dapat
diganti dengan kata ...

(A) berisi.
(B) berlagak.
(C) berpotensi.

(D) beragam.
(E) bermakna.

38. Salju abadi di Kibo Area, Kilimanjaro, terbentuk


karena glacier abadi. Namun, kini, luas glacier abadi
itu terus menyusut seiring terjadinya pemanasan
global. Saat ini lapisan glacier di Kilimanjaro tinggal
2,2 km2 dari total 19 km2 di Afrika. Padahal, pada
1982, luas glacier di Kilimanjaro masih 3,3 km2 . Hal
ini berarti ada penyusutan 33% dalam waktu 27
tahun. Oleh karena itu, diperkirakan dalam waktu
12 dekade ke depan, lapisan glacier itu akan
lenyap dan Kilimanjaro pun akan terpuruk menjadi
seonggok gunung batu berdebu.
Mengapa salju abadi di Kibo Area, Kilimanjaro,
pada sekitar 12 dekade mendatang hanya akan
menjadi gunung batu berdebu saja?

(A) Salju yang ada di tempat itu akan menyusut.


(B) Terjadi pemanasan global.
(C) Terjadi pelelehan lapisan glacier seiring dengan
pemanasan global.
(D) Hal ini disebabkan oleh terjadinya penyusutan
glacier sebanyak 33%.
(E) Terjadi fenomena alam berupa pemanasan
global selama 27 tahun di area itu.

c Universitas Indonesia

Downloaded from http://pak-anang.blogspot.com

Halaman 6 dari 12 halaman

Kode Naskah Soal:


39. Apa saja segi buruk permen? Pertama, kandungan
vitamin dalam permen sangat sedikit. Begitu pun
kandungan seratnya nyaris tidak ada. Permen juga
menyebabkan batuk, membuat gemuk,
menimbulkan sakit gigi, menghilangkan nafsu
makan, hingga membuat anak jadi hiperaktif. Riset
terbaru bahkan menyebutkan anak yang terbiasa
makan permen sejak kecil, saat dewasa cenderung
memiliki perilaku kriminal. Para peneliti dari
Universitas Cardi, Wales, Inggris, yang
melakukan riset ini mengamati data 17.415 anak
yang lahir pada bulan April 1970 di Inggris. Data
yang diambil dari British Cohort Study itu secara
terperinci menyebutkan informasi kesehatan dan
gaya hidup anak-anak tersebut pada periode
tertentu, misalnya saat usia 5, 10, dan dewasa. Di
usia 34 tahun, para responden diwawancarai
apakah mereka pernah melakukan tindakan
kriminal. Hasilnya, 69 persen yang pernah
melakukan tindakan kekerasan atau kriminal
ternyata memiliki riwayat hobi makan permen di
usia 10 tahun, dibandingkan dengan 42 persen
yang tidak punya catatan kriminal. Jadi, perlukah
orang tua mengibarkan "bendera perang" pada
permen? Tidak sepenuhnya demikian, kata Simon
Moore, dosen senior dalam studi Violence and
Society Research Group, Universitas Cardi.
Intisari dari kutipan tersebut yang benar adalah ...

209

40. Selama bertahun-tahun, isu kemiskinan menjadi


perhatian serius dan fokus kajian para sarjana dan
perumus kebijakan publik. Kemiskinan dianalisis
dari berbagai sudut pandang dan pendekatan guna
mendapatkan pemahaman yang utuh. Kemiskinan
bukan gejala sederhana, pun tidak terkait ekonomi
semata-mata, tetapi saling terkait dengan masalah
lain yang amat kompleks. Para sarjana mencoba
menerangkan denisi baru kemiskinan dengan
memberi makna melampaui pengertian
konvensional yang selama ini dipahami umum.
Kesimpulan yang dapat ditarik dari bacaan di atas
...

(A) Kemiskinan menjadi topik pembahasan para


ahli.
(B) Kemiskinan menjadi keprihatinan bagi para
ahli.
(C) Kemiskinan bersifat kompleks.
(D) Batasan kemiskinan sulit dirumuskan.
(E) Denisi kemiskinan perlu dirumuskan kembali
sesuai dengan perkembangan zaman.

(A) Orang tua jangan sering memberikan permen


kepada anaknya.
(B) Hasil riset tentang permen mengagetkan orang
tua.
(C) Permen mempunyai dampak buruk bagi
penikmatnya.
(D) Anak yang suka makan permen setelah besar
melakukan tindakan kekerasan.
(E) Kandungan vitamin pada permen sangat
sedikit.

c Universitas Indonesia

Downloaded from http://pak-anang.blogspot.com

Halaman 7 dari 12 halaman

Kode Naskah Soal:

209

BAHASA INGGRIS
(1) ____________________________. (2) However, plastic in the ocean actually decomposes as it is exposed to the
rain and sun and other environmental conditions. (3) When plastic decomposes it releases potentially toxic bisphenol A
(BPA) and PS oligomer into the water. (4) BPA and PS oligomer are sources of concern because they can disrupt the
functioning of hormones in animals and can seriously aect reproductive systems. (5) Polystyrene begins to decompose
within one year, releasing components that are detectable in the parts-per-million range. (6) However, the volume of
plastics in the ocean is increasing, so that decomposition products remain a potential problem. (7) It is estimated that 10
percent of the worlds plastic waste nds its way into the sea and slowly but surely most of it ends up in the Pacic
Ocean. (8) Each year as much as 150,000 tons of plastic debris, most notably Styrofoam, washes up on the shores of
Japan alone. (9) Vast expanses of waste, consisting mainly of plastic, oat elsewhere in the oceans.
Gunakan Petunjuk A dalam menjawab soal nomor 41 sampai
nomor 42.
41. The paragraph should begin with _______.

(A) Plastics in daily use are generally assumed to be


quite stable
(B) Scientists always believed that plastics in the
oceans were hazardous only to sea animals
(C) The UN Environmental Program estimates that
over a million seabirds, as well as more than 100
thousand marine mammals, die every year from
ingesting plastic
(D) Some researchers estimate that there are over
six kilos of plastic for every kilo of naturally
occurring plankton in the Pacic Ocean
(E) Plastics decompose with surprising speed and
release potentially toxic substances into the
water
42. The sentence which is irrelevant to the topic of the
paragraph is sentence number _______.

(A) four
(B) ve
(C) six

c Universitas Indonesia

(D) seven
(E) eight

Downloaded from http://pak-anang.blogspot.com

Halaman 8 dari 12 halaman

Kode Naskah Soal:

209

Spider-Man isnt the only person with an interest in spider silk. While Spider-Man uses the threads to zigzag from
building to building, or to snare a bad guy, scientists are investigating silk for dierent reasons. Though researchers
have learned a lot about silk by investigating spiders, insects such as caterpillars, ants and bees also have been studied
for the sticky stu. Scientists are even trying to get silk from animals such as goats.
It turns out silk might be good for weaving a lot more than shirts and ties. In the future, the silky ber might be
used to make supertough bulletproof vests and light but strong parachute cords. Silk also might work well for delicate
tasks inside the body. Researchers are experimenting with using silk to support growing cells, the same way a
construction crew builds scaolding around a building to help keep everything in its place during construction.
Spider silk is an ideal material, according to a researcher from University of Wyoming, but humans have been
gathering silk not from spiders but from silkworms for hundreds of years. But silkworm silk has its problems. A
silkworm covers its silk in sticky glue that holds the cocoon together. Sometimes humans have a bad allergic reaction to
this glue.
Spiders, on the other hand, dont use sticky glue. Most spiders have an abdomen made up of ve dierent sections.
The last two sections are where the silk-making happens. These sections of the lower belly are modied into special
structures called spinnerets, which are sort of like faucets for silk. The silk is mixed in glands and then secreted out of
the spinnerets. Spiders cant shoot silk out for long distances the way Spider-Man does. Instead, they attach the
emerging silk to something, like a tree branch, and then move away from the branch. This pulls the silk outward.
The main ingredient in spider silk is proteins, and there are many dierent kinds, depending on which spider is
spinning and which silk it wants to make. Some of the proteins are very large and complicated, and therefore hard to
make a lot of in the lab. So some scientists have put the genes that have the instructions for making silk into other
creatures, such as goats. The silk-making genes are turned on only in the goat cells that make milk, so when these goats
are milked, there is silk in the milk. However, a liter of milk may have only 15 grams of silk, which means it would take
about 600 gallons of milk to make one bulletproof vest. At higher concentrations the milk starts clumping, perhaps
because the silk proteins are sticking to milk proteins.
Gunakan Petunjuk A dalam menjawab soal nomor 43 sampai
nomor 47.
43. An appropriate title for the text is _______.

(A) The Chemical Characteristic of silk


(B) Silk Making : from Worms to Spiders
(C) Articial Spider Silk Production
(D) The Long History of Silk
(E) Silks Superpower
44. Which eld might use silk in the future?

(A) Health and automotive


(B) Agriculture and military
(C) Biochemistry and construction
(D) Military and medicine

45. Why did the writer mention building construction?

(A) To illustrate how strong and elastic silk is


(B) To describe how researchers extract silk out of
spiders
(C) To explain about the process of silk making
(D) To illustrate how growing cells can be held up
by silk
(E) To compare between silk from silkworms and
silk from spiders
46. The word clumping in paragraph 5 is best replaced
by _______.

(A) clotting
(B) cluttering
(C) spreading

(D) dispersing
(E) collecting

(E) Mining and hospitality

c Universitas Indonesia

Downloaded from http://pak-anang.blogspot.com

Halaman 9 dari 12 halaman

Kode Naskah Soal:

209

47. Which of the following statements about the text is


FALSE?

(A) The use of silk in health is still empirical.


(B) Silk making genes can be planted into animals
other than goats.
(C) Humans have long been collecting spider silk
for various purposes.
(D) Secreting silk from specially-engineered milk is
eective but not ecient.
(E) In the future, we will possibly use silk made by
bees or ants.

c Universitas Indonesia

Downloaded from http://pak-anang.blogspot.com

Halaman 10 dari 12 halaman

Kode Naskah Soal:

209

The power of music aects all of us and has long appealed to our emotions. In fact, the earliest known
music-making is in Stone Age culture, ____(48)____ we have been tapping our toes, humming along, singing and
dancing ever since. It is for this reason that UCLA researchers are using music to help children with autism spectrum
disorders (ASD), for whom understanding emotions is a very dicult task. This inability enables them of the chance to
____(49)____ eectively and make friends and can often lead to social isolation and loneliness. The researchers have
developed a music education program ____(50)____ to help children with ASD better understand emotions and learn to
recognize emotions in others.
Specically, the children are using a method of music ____(51)____ known as the Or-Schulwerk approach.
Developed by 20th -century German composer Carl Or ("schulwerk" is German for schooling), it is a unique approach
to music learning that is supported by movement and based on things that kids ____(52)____ like to do, such as sing,
chant rhymes, clap, dance and keep a beat or play a rhythm on anything near at hand. Or called this music and
movement activity "elemental" - basic, unsophisticated and concerned with the fundamental building blocks of music.
Gunakan Petunjuk A dalam menjawab soal nomor 48 sampai
nomor 52.
48. ....

(A) if
(B) and
(C) so

(D) but
(E) as

49. ....

(A) communicate
(B) communication
(C) communicating
(D) communicator
(E) communicated
50. ....

(A) discovered
(B) connected
(C) designed

(D) related
(E) improved

51. ....

(A) educate
(B) educated
(C) educating

(D) education
(E) educative

52. ....

(A) intuition
(B) intuitive
(C) intuitiveness
(D) intuit
(E) intuitively

c Universitas Indonesia

Downloaded from http://pak-anang.blogspot.com

Halaman 11 dari 12 halaman

Kode Naskah Soal:


Gunakan Petunjuk A dalam menjawab soal nomor 53 sampai
nomor 60.
53. "My computer wont boot up."
"_______"

(A) Mind if I use it?

209

57. Since dictionaries ________ to be read but to be


consulted when necessary, the lexicographer must
present his material in a concise, convenient, and
consistent style.

(A) do not mean


(B) are not meaning

(B) Have you tried it?

(C) did not mean

(C) Why dont you save it?

(D) were not meant

(D) I want to speed it up.


(E) Good thing youve backed up your data.
54. "The earthquake happening at about 3 oclock
yesterday caused many people to panic."
"Including me, as I _______ on the third-oor of my
oce at that time."

(E) are not meant


58. "Do you have tight working hours?"
"No. As long as I ________, I am allowed to have
exible working hours."

(A) did my job


(B) got my job to do

(A) worked

(C) am doing my job

(B) have worked

(D) have my job done

(C) have been working

(E) I am done

(D) was working


(E) had worked
55. "Wheres the report, Ben? You told me it would be
ready by now. I need it for the board meeting this
afternoon."
"Dont worry, Jim. _______ before the board
meeting."

(A) I have it nished

59. "I just ordered a large pizza for the two of us."
"Are you serious? You _____ hungry. Youve just
had your big lunch."

(A) shouldnt be
(B) mustnt be
(C) cant be

(D) are not


(E) may not be

60. If my brother had not gone for a vacation to


Singapore last week, he would not have been
infected with H1N1 u which has killed some
people in Indonesia. From the above sentence we
may conclude that _______.

(B) I have had it nished


(C) I will have it nished
(D) The report will nish
(E) The report has nished
56. "Did you receive our inquiry? When will we receive
your conrmation?"
"My apology. It seems that _______. Could you
possibly resend it?"

(A) we mislay your letter

(A) my brother had been to Singapore before he


became infected
(B) nobody was cured after getting H1N1
(C) my brother caught the H1N1 u some time ago
(D) People who went to singapore caught H1N1
(E) my brother died because of the H1N1 u

(B) your letter is mislaid


(C) we had mislaid your letter
(D) your letter has been mislaid
(E) your letter was mislaid

c Universitas Indonesia

Downloaded from http://pak-anang.blogspot.com

Halaman 12 dari 12 halaman

Anda mungkin juga menyukai